You are on page 1of 372

SET 3

Series : SSO/C

Code No.

56/3


- -

.
Roll No.

Candidates must write the Code on


the title page of the answer-book.

- 12
- - -
- 26
,
- 15 - 10.15
10.15 10.30 -
-

Please check that this question paper contains 12 printed pages.


Code number given on the right hand side of the question paper should be written on the
title page of the answer-book by the candidate.
Please check that this question paper contains 26 questions.
Please write down the Serial Number of the question before attempting it.
15 minutes time has been allotted to read this question paper. The question paper will be
distributed at 10.15 a.m. From 10.15 a.m. to 10.30 a.m., the students will read the
question paper only and will not write any answer on the answer-book during this period.

()
CHEMISTRY (Theory)
: 3 ]

Time allowed : 3 hours ]

: 70

[ Maximum Marks : 70

:
(i)

(ii)

- 1 5 - 1
- 6 10 - 2
- 11 22 - 3
- 23 4
- 24 26 - 5
,

(iii)
(iv)
(v)
(vi)
(vii)
56/3

[P.T.O.

General Instructions :

1.

(i)

All questions are compulsory.

(ii)

Question number 1 to 5 are very short answer questions and carry 1 mark each.

(iii)

Question number 6 to 10 are short answer questions and carry 2 marks each.

(iv)

Question number 11 to 22 are also short answer questions and carry 3 marks each.

(v)

Question number 23 is a value based question and carry 4 marks.

(vi)

Question number 24 to 26 are long answer questions and carry 5 marks each.

(vii)

Use log tables, if necessary. Use of calculators is not allowed.

What is meant by chelate effect ?

2.

(IUPAC) :
CH3 CH2 CHO
Write the IUPAC name of the following :
CH3 CH2 CHO

3.

:
, p- p-
Arrange the following in increasing order of basic strength :
Aniline, p-Nitroaniline and p-Toluidine

4.

AgCl

What type of stoichiometric defect is shown by AgCl ?

5.

?
What are emulsions ? Give an example.

56/3

6.

(i) (ii) H2S



Describe the preparation of potassium permanganate. How does the acidified
permanganate solution react with oxalic acid ? Write the ionic equations for the
reactions.
OR
Describe the oxidising action of potassium dichromate and write the ionic equations
for its reaction with (i) an iodide (ii) H2S.

7.


Write the mechanism of acid dehydration of ethanol to yield ethene.

8.

:
(i)

(x)

(ii)

(m)

Define the following terms :

56/3

(i)

Mole fraction (x)

(ii)

Molality of a solution (m)


3

[P.T.O.

9.


mol L1
Write units of rate constants for zero order and for the second order reactions if the
concentration is expressed in mol L1 and time in second.

10.

:
(i)

(ii)

NF3

NCl3

Explain the following :

11.

(i)

Nitrogen is much less reactive than phosphorus.

(ii)

NF3 is an exothermic compound but NCl3 is an endothermic compound.

?
What is meant by disproportionation ? Give one example of disproportionation
reaction in aqueous solutions.

12.

IUPAC :
(i)

[Co(NH3)6]Cl3

(ii)

[NiCl4]2

(iii) K3[Fe(CN)6]
Write the IUPAC name of the following :
(i)

[Co(NH3)6]Cl3

(ii)

[NiCl4]2

(iii) K3[Fe(CN)6]
56/3

13.

(IUPAC) :
(i)

CH3 CH CH2 CH3


|
Br
Br

(ii)
Br
(iii) CH2 = CH CH2 Cl
Give the IUPAC names of the following compounds :
(i)

CH3 CH CH2 CH3


|
Br
Br

(ii)
Br
(iii) CH2 = CH CH2 Cl

14.

?
(i)

(ii)

-1-

(iii)

-2-

How are the following conversions carried out ?


(i)

Benzyl chloride to Benzyl alcohol

(ii)

Ethyl magnesium chloride to Propan-1-ol

(iii) Propene to Propan-2-ol


56/3

[P.T.O.

15.

:
(i)

PCl5
?
CH3 CH2OH

OH
anhyd. AlCl3
+ CH3 Cl
?

(ii)

(iii) CH3 Cl + CH3CH2 ONa ?


Write the major product in the following equations :
(i)

PCl5
CH3 CH2OH
?

OH
anhyd. AlCl3
+ CH3 Cl
?

(ii)

(iii) CH3 Cl + CH3CH2 ONa ?

16.

:
(i)

(ii)

(iii)

Define the following as related to proteins :


(i)

Peptide linkage

(ii)

Primary structure

(iii) Denaturation

17.

Explain the term copolymerization and give two examples of copolymerization.


56/3

18.

fcc 4.077 108 cm ,


(r)
Silver crystallises in fcc lattice. If edge length of the unit cell is 4.077 108 cm, then
calculate the radius of silver atom.

19.

- (M.W. 342) 5 ( ) X 0.877%


X
A 5 percent solution (by mass) of cane-sugar (M.W. 342) is isotonic with 0.877%
solution of substance X. Find the molecular weight of X.

20.

60 s1 1/10
?
The rate constant for a first order reaction is 60 s1. How much time will it take to
reduce the initial concentration of the reactant to its 1/10th value ?

21.

:
(i)

(ii)

(iii)

Describe the following processes :


(i)

Dialysis

(ii)

Electrophoresis

(iii) Tyndall effect


56/3

[P.T.O.

22.

:
(i)

(ii)

(iii)


Answer the following :
(i)

What is the role of cryolite in the metallurgy of aluminium ?

(ii)

Differentiate between roasting and calcination.

(iii) What is meant by the term chromatography ?


OR
Write the reactions taking place in different zones of the blast furnace to obtain Iron.

23.


,


(i)

(ii)

(iii)

Neeraj went to the departmental store to purchase groceries. On one of the shelves he
noticed sugar free tablets. He decided to buy them for his grandfather who was a
diabetic. There were three types of sugar free tablets. He decided to buy sucrolose
which was good for his grandfathers health.
(i)

Name another sugar free tablet which Neeraj did not purchase.

(ii)

Was it right to purchase such medicines without doctors prescription ?

(iii) What quality of Neeraj is reflected above ?


56/3

24.

(a)

(b)

:
(i)

p-

(ii)

4--3--2-

:
(i)

(ii)

(iii)

(a)

(b)

:
(i)

(ii)

CH3CHO

?
(i)

CH3 CH3

(ii)

CH3 CH CH2 CHO


|
OH

(iii) CH3CH2OH
(a)

(b)

Draw the structures of the following :


(i)

p-Methylbenzaldehyde

(ii)

4-Methylpent-3-en-2-one

Give chemical tests to distinguish between the following pairs of compounds :


(i)

Benzoic acid and Ethyl benzoate.

(ii)

Benzaldehyde and Acetophenone.

(iii) Phenol and Benzoic acid.


OR
56/3

[P.T.O.

(a)

(b)

Draw the structures of the following derivatives :


(i)

Propanone oxime

(ii)

Semicarbazone of CH3CHO

How will you convert ethanal into the following compounds ? Give the chemical
equations involved.
(i)

CH3 CH3

(ii)

CH3 CH CH2 CHO


|
OH

(iii) CH3CH2OH
25.

(a)

16

15

?
(b)


(i)

H2SO4 CaF2 ?

(ii)

(iii)

Ca(OH)2 ?

(a)

(b)

:
(i)

BrF3

(ii)

XeO3

:
(i)

PH3

NH3 ?

(ii)

(iii) XeOF4
56/3


10

(a)

Elements of Gr. 16 generally show lower value of first ionization enthalpy


compared to the corresponding periods of Gr. 15. Why ?

(b)

What happens when


(i)

concentrated H2SO4 is added to CaF2 ?

(ii)

sulphur dioxide reacts with chlorine in the presence of charcoal ?

(iii) ammonium chloride is treated with Ca(OH)2 ?


OR
(a)

(b)

Draw the structure of the following :


(i)

BrF3

(ii)

XeO3

Answer the following :


(i)

Why is NH3 more basic than PH3 ?

(ii)

Why are halogens strong oxidising agents ?

(iii) Draw the structure of XeOF4.

26.

rG e.m.f.(E)

25 C :

Zn(s) | Zn2+(aq) || Sn2+(aq) | Sn(s)

: E

= 0.76 V; E 2+ = 0.14 V
Zn2+/Zn
Sn /Sn

F = 96500 C mol1

(a)

56/3

11

[P.T.O.

(b)

:
Fe2+(aq) + Ag+(aq) Fe3+(aq) + Ag(s)

rG
(E + = 0.80 V; E 3+ 2+ = 0.77 V)
Ag /Ag
Fe /Fe
Calculate rG and e.m.f. (E) that can be obtained from the following cell under the
standard conditions at 25 C :
Zn(s) | Zn2+(aq) || Sn2+(aq) | Sn(s)
Given : E

= 0.76 V; E 2+ = 0.14 V
Zn2+/Zn
Sn /Sn

and

F = 96500 C mol1.
OR

(a)

Define conductivity and molar conductivity for the solution of an electrolyte.


Discuss their variation with concentration.

(b)

Calculate the standard cell potential of the galvanic cell in which the following
reaction takes place :
Fe2+(aq) + Ag+(aq) Fe3+(aq) + Ag(s)
Calculate the rG and equilibrium constant of the reaction also.
(E + = 0.80 V; E 3+ 2+ = 0.77 V)
Ag /Ag
Fe /Fe
____________

56/3

12

CHEMISTRY MARKING SCHEME


SET -56/3
Compt. July, 2015
Qu
es.

Value points

Marks

Formation of stable complex by polydentate ligand.

Propanal

p-Nitroaniline < Aniline < p-Toluidine

Frenkel defect

Emulsions are liquid liquid colloidal systems.


For example milk, cream (or any other one correct example)
Potassium permanganate is prepared by fusion of MnO2 with an alkali metal hydroxide and an
oxidising agent like KNO3. This produces the dark green K2MnO4 which disproportionates in a
neutral or acidic solution to give permanganate.

Oxalate ion or oxalic acid is oxidised at 333 K:


1

OR
6

i)
1

ii)
1

i)

ii) Molality (m) is defined as the number of moles of the solute per kilogram (kg) of the

solvent.

Or

Zero order : mol L-1s-1


Second order : L mol-1s-1

10

1
Due to high bond dissociation enthalpy of N N
1
ii)
Due to low bond dissociation enthalpy of F2 than Cl2 and strong bond formation
between N and F
Disproportionation : The reaction in which an element undergoes self-oxidation and self- 1

11

1
1

i)

reduction simultaneously. For example


2Cu+ (aq)

Cu2+ (aq) + Cu(s)

(Or any other correct equation)


12

13

i)

Hexaamminecobalt(III) chloride

ii)

Tetrachlorido nickelate(II)

iii)

Potassium hexacyanoferrate(III)

i)

2-bromobutane

ii)

1, 3-dibromobenzene

iii)

3-choloropropene

1
1
1
1
1
1

14

i)
1
ii)

1
15

1
i)

ii)

1
iii)

16

17

Peptide linkage in proteins, -amino acids are connected to each other by peptide
bond or peptide linkage (-CONH- bond).
ii)
Primary structure - each polypeptide in a protein molecule having amino acids which
are linked with each other in a specific sequence.
iii)
Denaturation - When a protein is subjected to physical change like change in
temperature or chemical change like change in pH, protein loses its biological activity.
Copolymerisation is a polymerisation reaction in which a mixture of more than one monomeric
species is allowed to polymerise and form a copolymer.
i)

1
1
1
1

(or any other correct example)


18

r = 1.44 x

1
1

r=
r=

19

cm

cane sugar = X
Therefore, ccane sugar = cX

(where c is molar concentration)


1

gmol-1

MX =

20

MX = 59.9 or 60 gmol-1

k=

log

60 s-1 =
t=

log
log 10

t=

t= 0.0384 s
21

It is a process of removing the dissolved substance from a colloidal solution by means


of diffusion through a semi - permeable membrane.
The movement of colloidal particles under an applied electric potential towards
oppositely charged electrode is called electrophoresis.
Colloidal particles scatter light in all directions in space. This scattering of light
illuminates the path of beam in the colloidal dispersion.
It lowers the melting point of alumina / acts as a solvent.

i)
ii)
iii)

22

i)
ii)

Roasting
Ore is heated in a regular supply of air

Calcination
Heating in a limited supply or
absence of air.

1
1
1

1
1
1

(Or with equation)


It is a process of separation of different components of a mixture which are differently

iii)

adsorbed on a suitable adsorbent.


OR
22

6x
=3

(any 6 correct equations)


Aspartame, Saccharin (any one)
No
Social concern, empathy, concern, social awareness (any 2 )

23

i)
ii)
iii)

24

a) i)

1
1
2

1
ii)

1
b) i)Add NaHCO3, benzoic acid will give brisk effervescence of CO2 whereas ethylbenzoate
4

will not.
ii)Add NaOH and I2, acetophonone forms yellow ppt of iodoform on heating whereas
benzaldehyde will not.
iii)Add neutral FeCl3, phenol gives violet colouration whereas benzoic acid does not.
(or any other correct test)

1
1
1

OR
24

a) i)

ii)

b) i)

ii)

1
iii)
1
25

a) Due to relatively stable half filled p-orbitals of group 15 elements


b) i) CaF2 + H2SO4 CaSO4 + 2HF
ii)

2
1
1
1

iii)
OR
25

a) i)

ii)

b) i)Due to small size of nitrogen, the lone pair of electron on nitrogen is localized/ easily
available for donation.
ii)Because they need only one electron to attain stable/noble gas configuration.

1
iii)
26

26

E0cell = E0Sn2+ / Sn - E0Zn2+ / Zn


= - 0.14V (- 0.76V)
= 0.62V
0
rG = -n F E0cell
= - 2 x 96500 C mol-1 x 0.62 V
= - 119660 J mol-1

Ecell = E0cell -

log

Ecell = 0.62 -

log

1
1
1
1

OR
a) The conductivity of a solution at any given concentration is the conductance of one unit
volume of solution kept between two platinum electrodes with unit area of cross section
and at a distance of unit length.
Molar conductivity of a solution at a given concentration is the conductance of the volume
V of solution containing one mole of electrolyte kept between two electrodes with area of
cross section A and distance of unit length.
Molar conductivity increases with decrease in concentration.
b)E0cell = E0C - E0A
= 0.80V 0.77V
= 0.03V
rG0 = -n F E0cell
= - 1 x 96500 C mol-1 x 0.03 V
= - 2895 J mol-1

Log Kc=
6

Log Kc=
Log Kc= 0.508

Dr. Sangeeta Bhatia

Sh. S.K. Munjal

Ms. Garima Bhutani

Sh. D.A. Mishra

SET 3
.

Series : SSO/1/C

Code No.

56/1/3


- -

.
Roll No.

Candidates must write the Code on


the title page of the answer-book.

- 11
- - -
- 26
,
- 15 - 10.15
10.15 10.30 -
-

Please check that this question paper contains 11 printed pages.


Code number given on the right hand side of the question paper should be written on the
title page of the answer-book by the candidate.
Please check that this question paper contains 26 questions.
Please write down the Serial Number of the question before attempting it.
15 minutes time has been allotted to read this question paper. The question paper will be
distributed at 10.15 a.m. From 10.15 a.m. to 10.30 a.m., the students will read the
question paper only and will not write any answer on the answer-book during this period.

()
CHEMISTRY (Theory)
: 3 ]

Time allowed : 3 hours ]

: 70

[ Maximum Marks : 70

:
(i)

(ii)

- 1 5 - 1
- 6 10 - 2
- 11 22 - 3
- 23 4
- 24 26 - 5

(iii)
(iv)
(v)
(vi)
(vii)
56/1/3

[P.T.O.

General Instructions :
(i)

All questions are compulsory.

(ii)

Q. No. 1 to 5 are very short answer questions and carry 1 mark each.

(iii) Q. No. 6 to 10 are short answer questions and carry 2 marks each.
(iv) Q. No. 11 to 22 are also short answer questions and carry 3 marks each.
(v)

Q. No. 23 is a value based question and carry 4 marks.

(vi) Q. No. 24 to 26 are long answer questions and carry 5 marks each.
(vii) Use log tables if necessary, use of calculator is not allowed.

[Ni(NH3)6]Cl2 (IUPAC)

1.

What is the IUPAC name of the complex [Ni(NH3)6]Cl2 ?

2.

3-
Draw the structure of 3-methylpentanal.

3.

C6H5N2Cl + H3PO2 + H2O - - Complete the following reaction equation :


C6H5N2Cl + H3PO2 + H2O - - -

: (z) ?

4.

What is the no. of atoms per unit cell (z) in a body-centred cubic structure ?

5.

In reference to surface chemistry, define dialysis.


56/1/3

:-

6.

H+
CH3CH2OH CH2 = CH2 + H2O
443 K
Explain the mechanism of dehydration steps of ethanol :H+
CH3CH2OH CH2 = CH2 + H2O
443 K

7.

Define osmotic pressure of a solution. How is the osmotic pressure related to the
concentration of a solute in a solution ?
8.

:
(i) (t)
(ii) (k)
Define the following terms :
(i) Half-life of a reaction (t)
(ii)

Rate constant (k)

9.

(i)

H2SO4

(ii)

XeF2

Draw the structures of the following :


(i) H2SO4
(ii)
10.

XeF2

:
(i)
(ii)
What is meant by disproportionation ? Give an example of a disproportionation
reaction in aqueous solution.
OR
Suggest reasons for the following features of transition metal chemistry :
(i) The transition metals and their compounds are usually paramagnetic.
(ii) The transition metals exhibit variable oxidation states.

56/1/3

[P.T.O.

11.

(i)

-2-

(ii)

(iii)

p-

How are the following conversions carried out ?


(i)

Propene to propane-2-ol

(ii)

Benzyl chloride to Benzyl alcohol

(iii) Anisole to p-Bromoanisole

12.

A B
Br2 KOH C C C6H7N
A, B C (IUPAC)

An aromatic compound A on treatment with aqueous ammonia and heating forms
compound B which on heating with Br2 and KOH forms a compound C of
molecular formula C6H7N. Write the structures and IUPAC names of compounds A, B
and C.

13.

How are vitamins classified ? Name the vitamin responsible for the coagulation of
blood.

14.

(i)

-S

(ii)

(iii)

Write the names and structures of the monomers of the following polymers :
(i)

Buna-S

(ii)

Neoprene

(iii) Teflon
56/1/3

15.

:
(i)
(ii)
(iii) F-
Define the following :
(i)

Schottky defect

(ii)

Frenkel defect

(iii) F-centre

(C2H4O2) 45 g 600 g

16.

(i)

(ii)


( : Kf = 1.86 K kg mol1)

45 g of ethylene glycol (C2H4O2) is mixed with 600 g of water. Calculate


(i)

the freezing point depression and

(ii)

the freezing point of the solution


(Given : Kf of water = 1.86 K kg mol1)

17.

700 K :
, Ea (R = 8.314 J K1 mol1)

500 K

0.02 s1

0.07 s1

The rate constants of a reaction at 500 K and 700 K are 0.02 s1 and 0.07 s1
respectively. Calculate the value of activation energy, Ea. (R = 8.314 J K1 mol1)
18.

:
(i)
(ii)
(iii) - ( )
Define the following terms :
(i)

Electrophoresis

(ii)

Adsorption

(iii) Shape selective catalysis


56/1/3

[P.T.O.

19.

(i)

(ii)

(iii)


(Pig) ?
Outline the principles of refining of metals by the following methods :
(i)

Distillation

(ii)

Zone refining

(iii) Electrolysis
OR
Write down the reactions taking place in different zones in the blast furnace during the
extraction of iron. How is pig iron different from cast iron ?

? ?

20.

What is lanthanoid contraction ? What are the consequences of lanthanoid contraction ?

21.

(i)

[Co(NH3)5(NO2)]2+

(ii)

[Co(en)3]Cl3 (en =

(iii) [Pt(NH3)2Cl2]
Indicate the types of isomerism exhibited by the following complexes :
(i)

[Co(NH3)5(NO2)]2+

(ii)

[Co(en)3]Cl3 (en = ethylene diamine)

(iii) [Pt(NH3)2Cl2]
56/1/3

(IUPAC) :

22.

(i)

CH3 CH CH2 CH3


|
OH

(ii)

(iii) CH3

CH3
|
C CH2 Cl
|
CH3

Name the following according to IUPAC system :


(i) CH3 CH CH2 CH3
|
OH

(ii)

(iii) CH3

CH3
|
C CH2 Cl
|
CH3

23.

(i)
(ii)
(iii)


?
?

Ramesh went to a departmental store to purchase groceries. On one of shelves he


noticed sugar-free tablets. He decided to buy them for his grandfather who was a
diabetic. There were three types of sugar-free tablets. Ramesh decided to buy
sucrolose which was good for his grandfathers health.
(i)

Name another sugar free tablet which Ramesh did not buy.

(ii)

Was it right to purchase such medicines without doctors prescription ?

(iii) What quality of Ramesh is reflected above ?


56/1/3

[P.T.O.

24.

(a)

(b)

:
(i)

(ii)

?
(i)

(ii)

m-

(iii)

3-

(a)

(b)

:
(i)

(ii)

:
(i)

LiAlH4
CH3 C CH3
?
||
O

CHO
(ii)

HNO3 / H2SO4
?
273 283 K

PCl5
(iii) CH3 COOH
?
(a)

(b)

Describe the following giving chemical equations :


(i)

De-carboxylation reaction

(ii)

Friedel-Crafts reaction

How will you bring about the following conversions ?


(i)

Benzoic acid to Benzaldehyde

(ii)

Benzene to m-Nitroacetophenone

(iii) Ethanol to 3-Hydroxybutanal


OR
56/1/3

(a)

Describe the following actions :


(i)

(b)

Acetylation

(ii)

Aldol condensation

Write the main product in the following equations :


(i)

LiAlH4
?
CH3 C CH3
||
O

CHO
(ii)

HNO3 / H2SO4
?
273 283 K

PCl5
(iii) CH3 COOH
?

25.

(a)

(b)

:
(i)

Cu + HNO3()

(ii)

P4 + NaOH+ H2O

(i)

R3P = O R3N = O ? (R = )

(ii)

(iii)

(a)

:
(i)

(ii)

H2SO4

(b)

, ,

(a)

Complete the following chemical reaction equations :

56/1/3

(i)

Cu + HNO3(dilute)

(ii)

P4 + NaOH+ H2O
9

[P.T.O.

(b)

(i)

Why does R3P = O exist but R3N = O does not ? (R = alkyl group)

(ii)

Why is dioxygen a gas but sulphur a solid ?

(iii) Why are halogens coloured ?


OR
(a)

26.

Write balanced equations for the following reactions :


(i)

Chlorine reacts with dry slaked lime.

(ii)

Carbon reacts with concentrated H2SO4.

(b)

Describe the contact process for the manufacture of sulphuric acid with special
reference to the reaction conditions, catalysts used and the yield in the process.

(a)

(b)

(i)

(^m)

(ii)

(iii)

:
(i)

(ii)

(a)

(b)


Ni(s) | Ni2+(aq) || Ag+(aq) | Ag(s)

25 C
?
E

56/1/3

= 0.25 V, E + = 0.80 V.
Ni2+/Ni
Ag /Ag
10

(a)

Define the following terms :


(i)

Molar conductivity (^m)

(ii)

Secondary batteries

(iii) Fuel cell


(b)

State the following laws :


(i)

Faraday first law of electrolysis

(ii)

Kohlrauschs law of independent migration of ions


OR

(a)

Define the term degree of dissociation. Write an expression that relates the molar
conductivity of a weak electrolyte to its degree of dissociation.

(b)

For the cell reaction


Ni(s) | Ni2+(aq) || Ag+(aq) | Ag(s)
Calculate the equilibrium constant at 25 C. How much maximum work would
be obtained by operation of this cell ?
E

= 0.25 V and E + = 0.80 V.


Ni2+/Ni
Ag /Ag

___________

56/1/3

11

[P.T.O.

56/1/3

12

CHEMISTRY MARKING SCHEME


SET -56/1/3
Compt. July, 2015
Qu
es.

Marks

Value points
Hexaamninenickel (II) chloride

4
5

(where Ar is C6H5)
2

It is a process of removing a dissolved substance from a colloidal solution by means of diffusion 1


through a suitable membrane.

The external pressure which is applied on solution side to stop the flow of solvent across the
semi-permeable membrane.

The osmotic pressure is directly proportional to concentration of the solution. /

= CRT

The half-life of a reaction is the time in which the concentration of a reactant is reduced to one- 1
half of its initial concentration.
Rate constant is the rate of reaction when the concentration of the reactant is unity.
1

1+1

i)
10

ii)

Disproportionation : The reaction in which an element undergoes self-oxidation and self- 1


reduction simultaneously. For example
2Cu+ (aq)

Cu2+ (aq) + Cu(s)

(Or any other correct equation)


OR
10

i)
ii)
i)

11

Due to presence of unpaired electrons in d-orbitals.


Due to incomplete filling of d-orbitals.

1
1
1

ii)

1
iii)

1
+

12

Benzoic acid
+

Benzamide

13

14

CAniline
Fat soluble vitamin- Vitamin A, D
Water soluble vitamin-Vitamin B,C
Vitamin K
i)

+
+
1
+

ii)
+

iii)

15

16

i)
The defect in which equal number of cations and anions are missing from the lattice.
ii)
Due to dislocation of smaller ion from its normal site to an interstitial site.
iii)
Anionic vacancies are occupied by unpaired electron.
i) Tf = Kf m
Tf = Kf

1
1
1

Tf =
1

Tf =2.325K or 2.3250 C
ii) Tf0- Tf = 2.3250 C
O0C - Tf = 2.3250 C
Tf = - 2.3250 C or 270.675 K

17

1
18

i) The movement of colloidal particles under an applied electric potential towards oppositely
charged electrode is called electrophoresis.
ii) The accumulation of molecular species at the surface rather than in the bulk of a solid or liquid
3

1
1

iii)

19 i)
ii)
iii)

is termed adsorption.
The catalytic reaction that depends upon the pore structure of the catalyst and the size of the
reactant and product molecules is called shape-selective catalysis.
The impure metal is evaporated to obtain the pure metal as distillate.
This method is based on the principle that the impurities are more soluble in the melt than in
the solid state of the metal.
The impure metal is made to act as anode. A strip of the same metal in pure form is used as
cathode. They are put in a suitable electrolytic bath containing soluble salt of the same metal.
The more basic metal remains in the solution and the less basic ones go to the anode mud.
OR

19

21

22

23

24

1
1

x4
=2

( any four correct equations)


Cast iron has lower carbon content (about 3%) than pig iron / cast iron is hard & brittle whereas
pig iron is soft.

20

The steady decrease in atomic radii from La to Lu due to imperfect shielding of 4f orbital.
Consequences
i)
Members of third transition series have almost identical radii as coresponding members
of second transition series.
ii)
Difficulty in separation.

a) Linkage isomerism
b) Optical isomerism
c) Cis - trans / Geometrical isomerism
a) Butan 2 ol
b) 2 bromotoluene
c) 2, 2-dimethylchlorpropane
i)
Aspartame, Saccharin (any one)
ii)
No
iii)
Social concern, empathy, concern, social awareness (any 2 )
a) i) Carboxylic acids lose carbon dioxide to form hydrocarbons when their sodium salts are
heated with sodalime (NaOH and CaO).

1
1
1
1
1
1
1
1
2
1

ii) When the alkyl / acyl group is introduced at ortho and para positions by reaction
with alkyl halide / acyl halide in the presence of anhydrous aluminium chloride (a Lewis
acid) as catalyst.

1+1

(Note : Award full marks if correct equation is given )


b) i)

1
ii)

1
iii)

1
(or any other correct method)
OR
a) i) When the acyl groups are introduced at ortho and para positions by reaction with acyl halide in the
presence of anhydrous aluminium chloride (a Lewis acid) as catalyst.

24

-hydrogen undergo a reaction in the presence of


dilute alkali as catalyst to form -hydroxy aldehydes (aldol) or
hydroxy ketones (ketol),
respectively.

ii) Aldehydes and ketones having at least one

(Note : Award full marks if correct equation is given )


b)i)
1

ii)

iii)

25

CH3COCl

1
1

a) i)
ii)

b) i) Due to absence of d-orbital, nitrogen cannot expand its valency beyond four.
ii) Because of p p multiple bonding in dioxygen which is absent in sulphur.
iii) Due to excitation of electron by absorption of radiation from visible region.

1
1
1

OR

25

1
1

a) i)
ii)
b) It is manufactured by Contact Process which involves following steps:

i) burning of sulphur or sulphide ores in air to generate SO2.


ii) conversion of SO2 to SO3 by the reaction with oxygen in the presence of a catalyst (V2O5)
iii) absorption of SO3 in H2SO4 to give Oleum (H2S2O7). The oleum obtained is diluted to give
sulphuric acid

26

26

Reaction condition pressure of 2 bar and temperature of 720 K


Catalyst used is V2O5
Yield 96 98% pure
a)i)Molar conductivity of a solution at a given concentration is the conductance of the volume V
of solution containing one mole of electrolyte kept between two electrodes with area of cross
section A and distance of unit length.
ii) Secondary battery- can be recharged by passing current through it in opposite direction so that
it can be used again.
iii) Galvanic cells that are designed to convert the energy of combustion of fuels like hydrogen,
methane, methanol, etc. directly into electrical energy are called fuel cells.
b)i) The amount of chemical reaction which occurs at any electrode during electrolysis by a
current is proportional to the quantity of electricity passed through the electrolyte (solution or
melt).
ii) Limiting molar conductivity of an electrolyte can be represented as the sum of the individual
contributions of the anion and cation of the electrolyte.
OR
a) Degree of dissociation is the extent to which electrolyte gets dissociated into its constituent
ions.

b) E0cell = E0Ag+ / Ag - E0Ni2+ / Ni


= 0.80V 0.25V
6

1
1

1
1

1
1

1
1

= 0.55V

1og Kc =
=
log Kc = 18.644
G0 = - nFE0cell
= -2x96500 Cmol-1 x 0.55V
= -106,150 Jmol-1
Max.work =+106150 Jmol-1 or 106.150k Jmol-1

Dr. Sangeeta Bhatia

Sh. S.K. Munjal

Ms. Garima Bhutani

Sh. D.A. Mishra

SET 2
.

Series : SSO/C

Code No.

56/2


- -

.
Roll No.

Candidates must write the Code on


the title page of the answer-book.

- 12
- - -
- 26
,
- 15 - 10.15
10.15 10.30 -
-

Please check that this question paper contains 12 printed pages.


Code number given on the right hand side of the question paper should be written on the
title page of the answer-book by the candidate.
Please check that this question paper contains 26 questions.
Please write down the Serial Number of the question before attempting it.
15 minutes time has been allotted to read this question paper. The question paper will be
distributed at 10.15 a.m. From 10.15 a.m. to 10.30 a.m., the students will read the
question paper only and will not write any answer on the answer-book during this period.

()
CHEMISTRY (Theory)
: 3 ]

Time allowed : 3 hours ]

: 70

[ Maximum Marks : 70

:
(i)

(ii)

- 1 5 - 1
- 6 10 - 2
- 11 22 - 3
- 23 4
- 24 26 - 5
,

(iii)
(iv)
(v)
(vi)
(vii)
56/2

[P.T.O.

General Instructions :

1.

(i)

All questions are compulsory.

(ii)

Question number 1 to 5 are very short answer questions and carry 1 mark each.

(iii)

Question number 6 to 10 are short answer questions and carry 2 marks each.

(iv)

Question number 11 to 22 are also short answer questions and carry 3 marks each.

(v)

Question number 23 is a value based question and carry 4 marks.

(vi)

Question number 24 to 26 are long answer questions and carry 5 marks each.

(vii)

Use log tables, if necessary. Use of calculators is not allowed.

?
What are emulsions ? Give an example.

2.

What is meant by chelate effect ?

3.

(IUPAC) :
CH3 CH2 CHO
Write the IUPAC name of the following :
CH3 CH2 CHO

4.

:
, p- p-
Arrange the following in increasing order of basic strength :
Aniline, p-Nitroaniline and p-Toluidine

5.

AgCl

What type of stoichiometric defect is shown by AgCl ?


56/2

6.

:
(i)

(ii)

NF3

NCl3

Explain the following :

7.

(i)

Nitrogen is much less reactive than phosphorus.

(ii)

NF3 is an exothermic compound but NCl3 is an endothermic compound.

(i) (ii) H2S



Describe the preparation of potassium permanganate. How does the acidified
permanganate solution react with oxalic acid ? Write the ionic equations for the
reactions.
OR
Describe the oxidising action of potassium dichromate and write the ionic equations
for its reaction with (i) an iodide (ii) H2S.

8.


Write the mechanism of acid dehydration of ethanol to yield ethene.

9.

:
(i)

(x)

(ii)

(m)

Define the following terms :

56/2

(i)

Mole fraction (x)

(ii)

Molality of a solution (m)


3

[P.T.O.

10.


mol L1
Write units of rate constants for zero order and for the second order reactions if the
concentration is expressed in mol L1 and time in second.

11.

:
(i) ?
(ii)
(iii) ?


Answer the following :
(i)

What is the role of cryolite in the metallurgy of aluminium ?

(ii)

Differentiate between roasting and calcination.

(iii) What is meant by the term chromatography ?


OR
Write the reactions taking place in different zones of the blast furnace to obtain Iron.

12.

?
What is meant by disproportionation ? Give one example of disproportionation
reaction in aqueous solutions.

13.

IUPAC :
(i)

[Co(NH3)6]Cl3

(ii)

[NiCl4]2

(iii) K3[Fe(CN)6]
Write the IUPAC name of the following :
(i)

[Co(NH3)6]Cl3

(ii)

[NiCl4]2

(iii) K3[Fe(CN)6]
56/2

14.

(IUPAC) :
(i)

CH3 CH CH2 CH3


|
Br
Br

(ii)
Br
(iii) CH2 = CH CH2 Cl
Give the IUPAC names of the following compounds :
(i)

CH3 CH CH2 CH3


|
Br
Br

(ii)
Br
(iii) CH2 = CH CH2 Cl

15.

?
(i)

(ii)

-1-

(iii)

-2-

How are the following conversions carried out ?


(i)

Benzyl chloride to Benzyl alcohol

(ii)

Ethyl magnesium chloride to Propan-1-ol

(iii) Propene to Propan-2-ol


56/2

[P.T.O.

16.

:
(i)

PCl5
?
CH3 CH2OH

OH
anhyd. AlCl3
+ CH3 Cl
?

(ii)

(iii) CH3 Cl + CH3CH2 ONa ?


Write the major product in the following equations :
(i)

PCl5
CH3 CH2OH
?

OH
anhyd. AlCl3
+ CH3 Cl
?

(ii)

(iii) CH3 Cl + CH3CH2 ONa ?

17.

:
(i)

(ii)

(iii)

Define the following as related to proteins :


(i)

Peptide linkage

(ii)

Primary structure

(iii) Denaturation

18.

Explain the term copolymerization and give two examples of copolymerization.


56/2

19.

fcc 4.077 108 cm ,


(r)
Silver crystallises in fcc lattice. If edge length of the unit cell is 4.077 108 cm, then
calculate the radius of silver atom.

20.

- (M.W. 342) 5 ( ) X 0.877%


X
A 5 percent solution (by mass) of cane-sugar (M.W. 342) is isotonic with 0.877%
solution of substance X. Find the molecular weight of X.

21.

60 s1 1/10
?
The rate constant for a first order reaction is 60 s1. How much time will it take to
reduce the initial concentration of the reactant to its 1/10th value ?

22.

:
(i)

(ii)

(iii)

Describe the following processes :


(i)

Dialysis

(ii)

Electrophoresis

(iii) Tyndall effect


56/2

[P.T.O.

23.


,


(i)

(ii)

(iii)

Neeraj went to the departmental store to purchase groceries. On one of the shelves he
noticed sugar free tablets. He decided to buy them for his grandfather who was a
diabetic. There were three types of sugar free tablets. He decided to buy sucrolose
which was good for his grandfathers health.
(i)

Name another sugar free tablet which Neeraj did not purchase.

(ii)

Was it right to purchase such medicines without doctors prescription ?

(iii) What quality of Neeraj is reflected above ?

24.

(a)

16

15

?
(b)


(i)

H2SO4 CaF2 ?

(ii)

(iii)

Ca(OH)2 ?

56/2

(a)

(b)

:
(i)

BrF3

(ii)

XeO3

:
(i)

PH3

NH3 ?

(ii)

(iii) XeOF4

(a)

Elements of Gr. 16 generally show lower value of first ionization enthalpy


compared to the corresponding periods of Gr. 15. Why ?

(b)

What happens when


(i)

concentrated H2SO4 is added to CaF2 ?

(ii)

sulphur dioxide reacts with chlorine in the presence of charcoal ?

(iii) ammonium chloride is treated with Ca(OH)2 ?


OR
(a)

(b)

Draw the structure of the following :


(i)

BrF3

(ii)

XeO3

Answer the following :


(i)

Why is NH3 more basic than PH3 ?

(ii)

Why are halogens strong oxidising agents ?

(iii) Draw the structure of XeOF4.


56/2

[P.T.O.

25.

(a)

(b)

:
(i)

p-

(ii)

4--3--2-

:
(i)

(ii)

(iii)

(a)

(b)

:
(i)

(ii)

CH3CHO

?
(i)

CH3 CH3

(ii)

CH3 CH CH2 CHO


|
OH

(iii) CH3CH2OH
(a)

(b)

Draw the structures of the following :


(i)

p-Methylbenzaldehyde

(ii)

4-Methylpent-3-en-2-one

Give chemical tests to distinguish between the following pairs of compounds :


(i)

Benzoic acid and Ethyl benzoate.

(ii)

Benzaldehyde and Acetophenone.

(iii) Phenol and Benzoic acid.


OR
56/2

10

(a)

(b)

Draw the structures of the following derivatives :


(i)

Propanone oxime

(ii)

Semicarbazone of CH3CHO

How will you convert ethanal into the following compounds ? Give the chemical
equations involved.
(i)

CH3 CH3

(ii)

CH3 CH CH2 CHO


|
OH

(iii) CH3CH2OH

26.

rG e.m.f.(E)

25 C :

Zn(s) | Zn2+(aq) || Sn2+(aq) | Sn(s)

: E

= 0.76 V; E 2+ = 0.14 V
Zn2+/Zn
Sn /Sn

F = 96500 C mol1

(a)

(b)

:
Fe2+(aq) + Ag+(aq) Fe3+(aq) + Ag(s)

rG
(E

Ag+/Ag

56/2

= 0.80 V; E

Fe3+/Fe2+

= 0.77 V)
11

[P.T.O.

Calculate rG and e.m.f. (E) that can be obtained from the following cell under the
standard conditions at 25 C :
Zn(s) | Zn2+(aq) || Sn2+(aq) | Sn(s)
Given : E

= 0.76 V; E 2+ = 0.14 V
Zn2+/Zn
Sn /Sn

and

F = 96500 C mol1.
OR

(a)

Define conductivity and molar conductivity for the solution of an electrolyte.


Discuss their variation with concentration.

(b)

Calculate the standard cell potential of the galvanic cell in which the following
reaction takes place :
Fe2+(aq) + Ag+(aq) Fe3+(aq) + Ag(s)
Calculate the rG and equilibrium constant of the reaction also.
(E + = 0.80 V; E 3+ 2+ = 0.77 V)
Ag /Ag
Fe /Fe
____________

56/2

12

CHEMISTRY MARKING SCHEME


SET -56/2
Compt. July, 2015
Qu
es.

Value points

Marks
+

Emulsions are liquid liquid colloidal systems.


For example milk, cream (or any other one correct example)
Formation of stable complex by polydentate ligand.

Propanal

p-Nitroaniline < Aniline < p-Toluidine

Frenkel defect

i)

Due to high bond dissociation enthalpy of N N


ii)
Due to low bond dissociation enthalpy of F2 than Cl2 and strong bond formation
between N and F
Potassium permanganate is prepared by fusion of MnO2 with an alkali metal hydroxide and an
oxidising agent like KNO3. This produces the dark green K2MnO4 which disproportionates in a
neutral or acidic solution to give permanganate.

1
1

Oxalate ion or oxalic acid is oxidised at 333 K:


1

OR
7

i)
1

ii)
1

1
1

i)

ii) Molality (m) is defined as the number of moles of the solute per kilogram (kg) of the

solvent.

10
11

Or

Zero order : mol L-1s-1


Second order : L mol-1s-1

i)
ii)

1
1

It lowers the melting point of alumina / acts as a solvent.


Roasting
Ore is heated in a regular supply of air

iii)

Calcination
Heating in a limited supply or
absence of air.

1
1

(Or with equation)


It is a process of separation of different components of a mixture which are differently
1

adsorbed on a suitable adsorbent.


OR
11

12

6x
=3

(any 6 correct equations)


Disproportionation : The reaction in which an element undergoes self-oxidation and self- 1
reduction simultaneously. For example
2Cu+ (aq)

Cu2+ (aq) + Cu(s)

(Or any other correct equation)


13

i)

Hexaamminecobalt(III) chloride

ii)

Tetrachlorido nickelate(II)

iii)

Potassium hexacyanoferrate(III)

1
1
1

14

i)

2-bromobutane

ii)

1, 3-dibromobenzene

iii)

3-choloropropene

1
1
1

15

i)
1
ii)

1
16

1
i)
ii)
1

iii)
1
17

18

Peptide linkage in proteins, -amino acids are connected to each other by peptide
bond or peptide linkage (-CONH- bond).
ii)
Primary structure - each polypeptide in a protein molecule having amino acids which
are linked with each other in a specific sequence.
iii)
Denaturation - When a protein is subjected to physical change like change in
temperature or chemical change like change in pH, protein loses its biological activity.
Copolymerisation is a polymerisation reaction in which a mixture of more than one monomeric
species is allowed to polymerise and form a copolymer.
i)

1
1
1
1

(or any other correct example)


19

r = 1.44 x

1
1

r=
r=

20

cm

cane sugar = X
Therefore, ccane sugar = cX

(where c is molar concentration)


1

gmol-1

MX =

21

MX = 59.9 or 60 gmol-1

k=

log

60 s-1 =
t=

log
log 10

t=

t= 0.0384 s
22

i)
ii)
iii)

23

24

i)
ii)
iii)
a)
b)

It is a process of removing the dissolved substance from a colloidal solution by means


of diffusion through a semi - permeable membrane.
The movement of colloidal particles under an applied electric potential towards
oppositely charged electrode is called electrophoresis.
Colloidal particles scatter light in all directions in space. This scattering of light
illuminates the path of beam in the colloidal dispersion.
Aspartame, Saccharin (any one)
No
Social concern, empathy, concern, social awareness (any 2 )
Due to relatively stable half filled p-orbitals of group 15 elements
i) CaF2 + H2SO4 CaSO4 + 2HF
ii)
iii)
OR

1
1
1

1
1
1
2
2
1
1
1

24

a) i)

ii)

b) i)Due to small size of nitrogen, the lone pair of electron on nitrogen is localized/ easily
available for donation.
ii)Because they need only one electron to attain stable/noble gas configuration.

1
1

1
iii)
25

a) i)

1
ii)

1
b) i)Add NaHCO3, benzoic acid will give brisk effervescence of CO2 whereas ethylbenzoate
will not.
ii)Add NaOH and I2, acetophonone forms yellow ppt of iodoform on heating whereas
benzaldehyde will not.
iii)Add neutral FeCl3, phenol gives violet colouration whereas benzoic acid does not.
5

1
1

(or any other correct test)

OR
25

a) i)

ii)

b) i)

ii)

1
iii)
1
26

26

E0cell = E0Sn2+ / Sn - E0Zn2+ / Zn


= - 0.14V (- 0.76V)
= 0.62V
rG0 = -n F E0cell
= - 2 x 96500 C mol-1 x 0.62 V
= - 119660 J mol-1

Ecell = E0cell -

log

Ecell = 0.62 -

log

1
1
1
1

OR
a) The conductivity of a solution at any given concentration is the conductance of one unit
volume of solution kept between two platinum electrodes with unit area of cross section
and at a distance of unit length.
Molar conductivity of a solution at a given concentration is the conductance of the volume
V of solution containing one mole of electrolyte kept between two electrodes with area of
cross section A and distance of unit length.
Molar conductivity increases with decrease in concentration.

b)E0cell = E0C - E0A


= 0.80V 0.77V
= 0.03V
0
rG = -n F E0cell
= - 1 x 96500 C mol-1 x 0.03 V
= - 2895 J mol-1

Log Kc=

Log Kc=
Log Kc= 0.508

Dr. Sangeeta Bhatia

Sh. S.K. Munjal

Ms. Garima Bhutani

Sh. D.A. Mishra

SET 2
.

Series : SSO/1/C

Code No.

56/1/2


- -

.
Roll No.

Candidates must write the Code on


the title page of the answer-book.

- 11
- - -
- 26
,
- 15 - 10.15
10.15 10.30 -
-

Please check that this question paper contains 11 printed pages.


Code number given on the right hand side of the question paper should be written on the
title page of the answer-book by the candidate.
Please check that this question paper contains 26 questions.
Please write down the Serial Number of the question before attempting it.
15 minutes time has been allotted to read this question paper. The question paper will be
distributed at 10.15 a.m. From 10.15 a.m. to 10.30 a.m., the students will read the
question paper only and will not write any answer on the answer-book during this period.

()
CHEMISTRY (Theory)
: 3 ]

Time allowed : 3 hours ]

: 70

[ Maximum Marks : 70

:
(i)

(ii)

- 1 5 - 1
- 6 10 - 2
- 11 22 - 3
- 23 4
- 24 26 - 5

(iii)
(iv)
(v)
(vi)
(vii)
56/1/2

[P.T.O.

General Instructions :
(i)

All questions are compulsory.

(ii)

Q. No. 1 to 5 are very short answer questions and carry 1 mark each.

(iii) Q. No. 6 to 10 are short answer questions and carry 2 marks each.
(iv) Q. No. 11 to 22 are also short answer questions and carry 3 marks each.
(v)

Q. No. 23 is a value based question and carry 4 marks.

(vi) Q. No. 24 to 26 are long answer questions and carry 5 marks each.
(vii) Use log tables if necessary, use of calculator is not allowed.

1.

In reference to surface chemistry, define dialysis.

[Ni(NH3)6]Cl2 (IUPAC)

2.

What is the IUPAC name of the complex [Ni(NH3)6]Cl2 ?

3.

3-
Draw the structure of 3-methylpentanal.

4.

C6H5N2Cl + H3PO2 + H2O - - Complete the following reaction equation :


C6H5N2Cl + H3PO2 + H2O - - -

: (z) ?

5.

What is the no. of atoms per unit cell (z) in a body-centred cubic structure ?
56/1/2

6.

:
(i)

(ii)

What is meant by disproportionation ? Give an example of a disproportionation


reaction in aqueous solution.
OR
Suggest reasons for the following features of transition metal chemistry :
(i)

The transition metals and their compounds are usually paramagnetic.

(ii)

The transition metals exhibit variable oxidation states.

:-

7.

H+
CH3CH2OH CH2 = CH2 + H2O
443 K
Explain the mechanism of dehydration steps of ethanol :H+
CH3CH2OH CH2 = CH2 + H2O
443 K

8.

Define osmotic pressure of a solution. How is the osmotic pressure related to the
concentration of a solute in a solution ?

9.

(i)

(t)

(ii)

(k)

Define the following terms :


(i)

Half-life of a reaction (t)

(ii)

Rate constant (k)

56/1/2

[P.T.O.

10.

(i)

H2SO4

(ii)

XeF2

Draw the structures of the following :


(i)

H2SO4

(ii)

XeF2

11.

(i)

[Co(NH3)5(NO2)]2+

(ii)

[Co(en)3]Cl3 (en =

(iii) [Pt(NH3)2Cl2]
Indicate the types of isomerism exhibited by the following complexes :
(i)

[Co(NH3)5(NO2)]2+

(ii)

[Co(en)3]Cl3 (en = ethylene diamine)

(iii) [Pt(NH3)2Cl2]

(IUPAC) :

12.

(i)

CH3 CH CH2 CH3


|
OH

(ii)

(iii) CH3

56/1/2

CH3
|
C CH2 Cl
|
CH3
4

Name the following according to IUPAC system :


(i)

CH3 CH CH2 CH3


|
OH

(ii)

(iii) CH3

CH3
|
C CH2 Cl
|
CH3

13.

(i)

-2-

(ii)

(iii)

p-

How are the following conversions carried out ?


(i)

Propene to propane-2-ol

(ii)

Benzyl chloride to Benzyl alcohol

(iii) Anisole to p-Bromoanisole

A B
Br2 KOH C C C6H7N

14.

A, B

(IUPAC)

An aromatic compound A on treatment with aqueous ammonia and heating forms


compound B which on heating with Br2 and KOH forms a compound C of
molecular formula C6H7N. Write the structures and IUPAC names of compounds A, B
and C.
56/1/2

[P.T.O.

15.

How are vitamins classified ? Name the vitamin responsible for the coagulation of
blood.

16.

:
(i) -S
(ii)
(iii)
Write the names and structures of the monomers of the following polymers :
(i)

Buna-S

(ii)

Neoprene

(iii) Teflon

17.

:
(i)
(ii)
(iii) F-
Define the following :
(i)

Schottky defect

(ii)

Frenkel defect

(iii) F-centre

(C2H4O2) 45 g 600 g

18.

(i)

(ii)


( : Kf = 1.86 K kg mol1)

45 g of ethylene glycol (C2H4O2) is mixed with 600 g of water. Calculate


(i)

the freezing point depression and

(ii)

the freezing point of the solution


(Given : Kf of water = 1.86 K kg mol1)

56/1/2

19.

700 K :
, Ea (R = 8.314 J K1 mol1)

500 K

0.02 s1

0.07 s1

The rate constants of a reaction at 500 K and 700 K are 0.02 s1 and 0.07 s1
respectively. Calculate the value of activation energy, Ea. (R = 8.314 J K1 mol1)

20.

(i)

(ii)

(iii)

- ( )

Define the following terms :


(i)

Electrophoresis

(ii)

Adsorption

(iii) Shape selective catalysis

21.

(i)

(ii)

(iii)


(Pig) ?
Outline the principles of refining of metals by the following methods :
(i)

Distillation

(ii)

Zone refining

(iii) Electrolysis
OR
Write down the reactions taking place in different zones in the blast furnace during the
extraction of iron. How is pig iron different from cast iron ?
56/1/2

[P.T.O.

? ?

22.

What is lanthanoid contraction ? What are the consequences of lanthanoid contraction ?

23.

(i)

(ii)

(iii)

Ramesh went to a departmental store to purchase groceries. On one of shelves he


noticed sugar-free tablets. He decided to buy them for his grandfather who was a
diabetic. There were three types of sugar-free tablets. Ramesh decided to buy
sucrolose which was good for his grandfathers health.
(i)

Name another sugar free tablet which Ramesh did not buy.

(ii)

Was it right to purchase such medicines without doctors prescription ?

(iii) What quality of Ramesh is reflected above ?

24.

(a)

(b)

:
(i)

Cu + HNO3()

(ii)

P4 + NaOH+ H2O

(i)

R3P = O R3N = O ? (R = )

(ii)

(iii)

56/1/2

(a)

:
(i)
(ii) H2SO4

(b)

, ,

(a)

Complete the following chemical reaction equations :

(b)

(i)

Cu + HNO3(dilute)

(ii)

P4 + NaOH+ H2O

(i)

Why does R3P = O exist but R3N = O does not ? (R = alkyl group)

(ii)

Why is dioxygen a gas but sulphur a solid ?

(iii) Why are halogens coloured ?


OR
(a)

(b)

Write balanced equations for the following reactions :


(i)

Chlorine reacts with dry slaked lime.

(ii)

Carbon reacts with concentrated H2SO4.

Describe the contact process for the manufacture of sulphuric acid with special
reference to the reaction conditions, catalysts used and the yield in the process.

25.

(a)

(b)

:
(i)

(ii)

?
(i)

(ii)

m-

(iii)

3-

56/1/2

[P.T.O.

(a)

(b)

:
(i)

(ii)

:
(i)

LiAlH4
?
CH3 C CH3
||
O

CHO
HNO3 / H2SO4
?
273 283 K

(ii)

PCl5
(iii) CH3 COOH
?
(a)

(b)

Describe the following giving chemical equations :


(i)

De-carboxylation reaction

(ii)

Friedel-Crafts reaction

How will you bring about the following conversions ?


(i)

Benzoic acid to Benzaldehyde

(ii)

Benzene to m-Nitroacetophenone

(iii) Ethanol to 3-Hydroxybutanal


OR
(a)

Describe the following actions :


(i)

(b)

Acetylation

(ii)

Aldol condensation

Write the main product in the following equations :


(i)

LiAlH4
CH3 C CH3
?
||
O

CHO
(ii)

HNO3 / H2SO4
?
273 283 K

PCl5
(iii) CH3 COOH
?
56/1/2

10

26.

(a)

:
(i) (^m)
(ii)
(iii)

(b)

:
(i)
(ii)

(a)

(b)


Ni(s) | Ni2+(aq) || Ag+(aq) | Ag(s)

25 C
?
E
(a)

= 0.25 V, E + = 0.80 V.
Ni2+/Ni
Ag /Ag

Define the following terms :


(i) Molar conductivity (^m)
(ii) Secondary batteries
(iii) Fuel cell

(b)

State the following laws :


(i) Faraday first law of electrolysis
(ii) Kohlrauschs law of independent migration of ions
OR

(a)

Define the term degree of dissociation. Write an expression that relates the molar
conductivity of a weak electrolyte to its degree of dissociation.

(b)

For the cell reaction


Ni(s) | Ni2+(aq) || Ag+(aq) | Ag(s)
Calculate the equilibrium constant at 25 C. How much maximum work would
be obtained by operation of this cell ?
E

= 0.25 V and E + = 0.80 V.


Ni2+/Ni
Ag /Ag
___________

56/1/2

11

[P.T.O.

56/1/2

12

CHEMISTRY MARKING SCHEME


SET -56/1/2
Compt. July, 2015
Qu
es.

Marks

Value points

It is a process of removing a dissolved substance from a colloidal solution by means of diffusion 1


through a suitable membrane.
Hexaamninenickel (II) chloride
1

(where Ar is C6H5)
2

Disproportionation : The reaction in which an element undergoes self-oxidation and self- 1

reduction simultaneously. For example


2Cu+ (aq)

Cu2+ (aq) + Cu(s)

(Or any other correct equation)


OR
6

i)
ii)

Due to presence of unpaired electrons in d-orbitals.


Due to incomplete filling of d-orbitals.

1
1

The external pressure which is applied on solution side to stop the flow of solvent across the
semi-permeable membrane.

The osmotic pressure is directly proportional to concentration of the solution. /

= CRT

10

The half-life of a reaction is the time in which the concentration of a reactant is reduced to one- 1
half of its initial concentration.
Rate constant is the rate of reaction when the concentration of the reactant is unity.
1
1+1

i)
11

12

13

ii)
a)
b)
c)
a)
b)
c)
i)

Linkage isomerism
Optical isomerism
Cis - trans / Geometrical isomerism
Butan 2 ol
2 bromotoluene
2, 2-dimethylchlorpropane

1
1
1
1
1
1
1

ii)

1
iii)

1
+

14

Benzoic acid
+

C-

Benzamide

Aniline
2

15

16

+
+
1
+

Fat soluble vitamin- Vitamin A, D


Water soluble vitamin-Vitamin B,C
Vitamin K
i)

ii)
+

iii)

17

18

i)
The defect in which equal number of cations and anions are missing from the lattice.
ii)
Due to dislocation of smaller ion from its normal site to an interstitial site.
iii)
Anionic vacancies are occupied by unpaired electron.
i) Tf = Kf m
Tf = Kf

1
1
1

Tf =
1

Tf =2.325K or 2.3250 C
ii) Tf0- Tf = 2.3250 C
O0C - Tf = 2.3250 C
Tf = - 2.3250 C or 270.675 K

19

1
20

i) The movement of colloidal particles under an applied electric potential towards oppositely
charged electrode is called electrophoresis.
ii) The accumulation of molecular species at the surface rather than in the bulk of a solid or liquid
is termed adsorption.
iii) The catalytic reaction that depends upon the pore structure of the catalyst and the size of the
reactant and product molecules is called shape-selective catalysis.
3

1
1
1

21 i) The impure metal is evaporated to obtain the pure metal as distillate.


ii) This method is based on the principle that the impurities are more soluble in the melt than in
the solid state of the metal.
iii) The impure metal is made to act as anode. A strip of the same metal in pure form is used as
cathode. They are put in a suitable electrolytic bath containing soluble salt of the same metal.
The more basic metal remains in the solution and the less basic ones go to the anode mud.
OR

1
1

x4
=2

( any four correct equations)


Cast iron has lower carbon content (about 3%) than pig iron / cast iron is hard & brittle whereas
pig iron is soft.

The steady decrease in atomic radii from La to Lu due to imperfect shielding of 4f orbital.
Consequences
i)
Members of third transition series have almost identical radii as coresponding members
of second transition series.
ii)
Difficulty in separation.

23

i)
ii)
iii)

24

a) i)

1
1
2
1
1

22

Aspartame, Saccharin (any one)


No
Social concern, empathy, concern, social awareness (any 2 )

1+1

ii)

b) i) Due to absence of d-orbital, nitrogen cannot expand its valency beyond four.
ii) Because of p p multiple bonding in dioxygen which is absent in sulphur.
iii) Due to excitation of electron by absorption of radiation from visible region.

1
1
1

OR

1
1

a) i)
ii)
b) It is manufactured by Contact Process which involves following steps:

i) burning of sulphur or sulphide ores in air to generate SO2.


ii) conversion of SO2 to SO3 by the reaction with oxygen in the presence of a catalyst (V2O5)
iii) absorption of SO3 in H2SO4 to give Oleum (H2S2O7). The oleum obtained is diluted to give
sulphuric acid
Reaction condition pressure of 2 bar and temperature of 720 K
Catalyst used is V2O5
4

1
1

25

Yield 96 98% pure


a) i) Carboxylic acids lose carbon dioxide to form hydrocarbons when their sodium salts are
heated with sodalime (NaOH and CaO).

ii) When the alkyl / acyl group is introduced at ortho and para positions by reaction
with alkyl halide / acyl halide in the presence of anhydrous aluminium chloride (a Lewis
acid) as catalyst.

1
(Note : Award full marks if correct equation is given )
b) i)

ii)

iii)

(or any other correct method)


OR
25

a) i) When the acyl groups are introduced at ortho and para positions by reaction with acyl halide in the
presence of anhydrous aluminium chloride (a Lewis acid) as catalyst.

-hydrogen undergo a reaction in the presence of


dilute alkali as catalyst to form -hydroxy aldehydes (aldol) or
hydroxy ketones (ketol),
respectively.

1
1

ii) Aldehydes and ketones having at least one

(Note : Award full marks if correct equation is given )


1

b)i)

ii)
1

1
iii)

26

26

CH3COCl

a)i)Molar conductivity of a solution at a given concentration is the conductance of the volume V


of solution containing one mole of electrolyte kept between two electrodes with area of cross
section A and distance of unit length.
ii) Secondary battery- can be recharged by passing current through it in opposite direction so that
it can be used again.
iii) Galvanic cells that are designed to convert the energy of combustion of fuels like hydrogen,
methane, methanol, etc. directly into electrical energy are called fuel cells.
b)i) The amount of chemical reaction which occurs at any electrode during electrolysis by a
current is proportional to the quantity of electricity passed through the electrolyte (solution or
melt).
ii) Limiting molar conductivity of an electrolyte can be represented as the sum of the individual
contributions of the anion and cation of the electrolyte.
OR
a) Degree of dissociation is the extent to which electrolyte gets dissociated into its constituent
ions.

b) E0cell = E0Ag+ / Ag - E0Ni2+ / Ni


= 0.80V 0.25V
= 0.55V

1
1

1
1

1og Kc =
=
log Kc = 18.644
G0 = - nFE0cell
= -2x96500 Cmol-1 x 0.55V
= -106,150 Jmol-1
Max.work =+106150 Jmol-1 or 106.150k Jmol-1

Dr. Sangeeta Bhatia

Sh. S.K. Munjal

Ms. Garima Bhutani

Sh. D.A. Mishra

SET 1
.

Series : SSO/C

Code No.

56/1


- -

.
Roll No.

Candidates must write the Code on


the title page of the answer-book.

- 12
- - -
- 26
,
- 15 - 10.15
10.15 10.30 -
-

Please check that this question paper contains 12 printed pages.


Code number given on the right hand side of the question paper should be written on the
title page of the answer-book by the candidate.
Please check that this question paper contains 26 questions.
Please write down the Serial Number of the question before attempting it.
15 minutes time has been allotted to read this question paper. The question paper will be
distributed at 10.15 a.m. From 10.15 a.m. to 10.30 a.m., the students will read the
question paper only and will not write any answer on the answer-book during this period.

()
CHEMISTRY (Theory)
: 3 ]

Time allowed : 3 hours ]

: 70

[ Maximum Marks : 70

:
(i)

(ii)

- 1 5 - 1
- 6 10 - 2
- 11 22 - 3
- 23 4
- 24 26 - 5
,

(iii)
(iv)
(v)
(vi)
(vii)
56/1

[P.T.O.

General Instructions :

1.

(i)

All questions are compulsory.

(ii)

Question number 1 to 5 are very short answer questions and carry 1 mark each.

(iii)

Question number 6 to 10 are short answer questions and carry 2 marks each.

(iv)

Question number 11 to 22 are also short answer questions and carry 3 marks each.

(v)

Question number 23 is a value based question and carry 4 marks.

(vi)

Question number 24 to 26 are long answer questions and carry 5 marks each.

(vii)

Use log tables, if necessary. Use of calculators is not allowed.

AgCl

What type of stoichiometric defect is shown by AgCl ?

2.

?
What are emulsions ? Give an example.

3.

What is meant by chelate effect ?

4.

(IUPAC) :
CH3 CH2 CHO
Write the IUPAC name of the following :
CH3 CH2 CHO

5.

:
, p- p-
Arrange the following in increasing order of basic strength :
Aniline, p-Nitroaniline and p-Toluidine

56/1

6.

:
(i)

(x)

(ii)

(m)

Define the following terms :

7.

(i)

Mole fraction (x)

(ii)

Molality of a solution (m)


mol L1
Write units of rate constants for zero order and for the second order reactions if the
concentration is expressed in mol L1 and time in second.

8.

:
(i)

(ii)

NF3

NCl3

Explain the following :

9.

(i)

Nitrogen is much less reactive than phosphorus.

(ii)

NF3 is an exothermic compound but NCl3 is an endothermic compound.

(i) (ii) H2S


56/1

[P.T.O.

Describe the preparation of potassium permanganate. How does the acidified


permanganate solution react with oxalic acid ? Write the ionic equations for the
reactions.
OR
Describe the oxidising action of potassium dichromate and write the ionic equations
for its reaction with (i) an iodide (ii) H2S.

10.


Write the mechanism of acid dehydration of ethanol to yield ethene.

11.

fcc 4.077 108 cm ,


(r)
Silver crystallises in fcc lattice. If edge length of the unit cell is 4.077 108 cm, then
calculate the radius of silver atom.

12.

- (M.W. 342) 5 ( ) X 0.877%


X
A 5 percent solution (by mass) of cane-sugar (M.W. 342) is isotonic with 0.877%
solution of substance X. Find the molecular weight of X.

13.

60 s1 1/10
?
The rate constant for a first order reaction is 60 s1. How much time will it take to
reduce the initial concentration of the reactant to its 1/10th value ?

56/1

14.

:
(i)

(ii)

(iii)

Describe the following processes :


(i)

Dialysis

(ii)

Electrophoresis

(iii) Tyndall effect

15.

:
(i)

(ii)

(iii)


Answer the following :
(i)

What is the role of cryolite in the metallurgy of aluminium ?

(ii)

Differentiate between roasting and calcination.

(iii) What is meant by the term chromatography ?


OR
Write the reactions taking place in different zones of the blast furnace to obtain Iron.

16.

?
What is meant by disproportionation ? Give one example of disproportionation
reaction in aqueous solutions.

56/1

[P.T.O.

17.

IUPAC :
(i)

[Co(NH3)6]Cl3

(ii)

[NiCl4]2

(iii) K3[Fe(CN)6]
Write the IUPAC name of the following :
(i)

[Co(NH3)6]Cl3

(ii)

[NiCl4]2

(iii) K3[Fe(CN)6]

18.

(IUPAC) :
(i)

CH3 CH CH2 CH3


|
Br
Br

(ii)
Br
(iii) CH2 = CH CH2 Cl
Give the IUPAC names of the following compounds :
(i)

CH3 CH CH2 CH3


|
Br
Br

(ii)
Br
(iii) CH2 = CH CH2 Cl
56/1

19.

?
(i)
(ii) -1-
(iii) -2-
How are the following conversions carried out ?
(i)

Benzyl chloride to Benzyl alcohol

(ii)

Ethyl magnesium chloride to Propan-1-ol

(iii) Propene to Propan-2-ol


20.

:
(i)

PCl5
CH3 CH2OH
?

OH
anhyd. AlCl3
+ CH3 Cl
?

(ii)

(iii) CH3 Cl + CH3CH2 ONa ?


Write the major product in the following equations :
(i)

PCl5
CH3 CH2OH
?

OH
(ii)

anhyd. AlCl3
+ CH3 Cl
?

(iii) CH3 Cl + CH3CH2 ONa ?


21.

:
(i)
(ii)
(iii)
Define the following as related to proteins :
(i)

Peptide linkage

(ii)

Primary structure

(iii) Denaturation
56/1

[P.T.O.

22.

Explain the term copolymerization and give two examples of copolymerization.

23.


,


(i)

(ii)

(iii)

Neeraj went to the departmental store to purchase groceries. On one of the shelves he
noticed sugar free tablets. He decided to buy them for his grandfather who was a
diabetic. There were three types of sugar free tablets. He decided to buy sucrolose
which was good for his grandfathers health.
(i)

Name another sugar free tablet which Neeraj did not purchase.

(ii)

Was it right to purchase such medicines without doctors prescription ?

(iii) What quality of Neeraj is reflected above ?

24.

rG e.m.f.(E)

25 C :

Zn(s) | Zn2+(aq) || Sn2+(aq) | Sn(s)

: E

= 0.76 V; E 2+ = 0.14 V
Zn2+/Zn
Sn /Sn

F = 96500 C mol1

56/1

(a)

(b)

:
Fe2+(aq) + Ag+(aq) Fe3+(aq) + Ag(s)

rG
(E + = 0.80 V; E 3+ 2+ = 0.77 V)
Ag /Ag
Fe /Fe

Calculate rG and e.m.f. (E) that can be obtained from the following cell under the
standard conditions at 25 C :
Zn(s) | Zn2+(aq) || Sn2+(aq) | Sn(s)
Given : E

= 0.76 V; E 2+ = 0.14 V
Zn2+/Zn
Sn /Sn

and

F = 96500 C mol1.
OR

(a)

Define conductivity and molar conductivity for the solution of an electrolyte.


Discuss their variation with concentration.

(b)

Calculate the standard cell potential of the galvanic cell in which the following
reaction takes place :
Fe2+(aq) + Ag+(aq) Fe3+(aq) + Ag(s)
Calculate the rG and equilibrium constant of the reaction also.
(E + = 0.80 V; E 3+ 2+ = 0.77 V)
Ag /Ag
Fe /Fe

56/1

[P.T.O.

25.

(a)

16

15

?
(b)


(i)

H2SO4 CaF2 ?

(ii)

(iii)

Ca(OH)2 ?

(a)

(b)

:
(i)

BrF3

(ii)

XeO3

:
(i)

PH3

NH3 ?

(ii)

(iii) XeOF4

(a)

Elements of Gr. 16 generally show lower value of first ionization enthalpy


compared to the corresponding periods of Gr. 15. Why ?

(b)

What happens when


(i)

concentrated H2SO4 is added to CaF2 ?

(ii)

sulphur dioxide reacts with chlorine in the presence of charcoal ?

(iii) ammonium chloride is treated with Ca(OH)2 ?


OR
56/1

10

(a)

(b)

Draw the structure of the following :


(i)

BrF3

(ii)

XeO3

Answer the following :


(i)

Why is NH3 more basic than PH3 ?

(ii)

Why are halogens strong oxidising agents ?

(iii) Draw the structure of XeOF4.

26.

(a)

(b)

:
(i)

p-

(ii)

4--3--2-

:
(i)

(ii)

(iii)

(a)

(b)

:
(i)

(ii)

CH3CHO

?
(i)

CH3 CH3

(ii)

CH3 CH CH2 CHO


|
OH

(iii) CH3CH2OH
56/1

11

[P.T.O.

(a)

(b)

Draw the structures of the following :


(i)

p-Methylbenzaldehyde

(ii)

4-Methylpent-3-en-2-one

Give chemical tests to distinguish between the following pairs of compounds :


(i)

Benzoic acid and Ethyl benzoate.

(ii)

Benzaldehyde and Acetophenone.

(iii) Phenol and Benzoic acid.


OR
(a)

(b)

Draw the structures of the following derivatives :


(i)

Propanone oxime

(ii)

Semicarbazone of CH3CHO

How will you convert ethanal into the following compounds ? Give the chemical
equations involved.
(i)

CH3 CH3

(ii)

CH3 CH CH2 CHO


|
OH

(iii) CH3CH2OH
_________

56/1

12

CHEMISTRY MARKING SCHEME


SET -56/1
Compt. July, 2015
Qu
es.

Value points

Marks

Frenkel defect

Emulsions are liquid liquid colloidal systems.


For example milk, cream (or any other one correct example)
Formation of stable complex by polydentate ligand.

Propanal

p-Nitroaniline < Aniline < p-Toluidine

i)

ii) Molality (m) is defined as the number of moles of the solute per kilogram (kg) of the

solvent.

7
8

Or

Zero order : mol L-1s-1


Second order : L mol-1s-1

1
1

i)

Due to high bond dissociation enthalpy of N N


ii)
Due to low bond dissociation enthalpy of F2 than Cl2 and strong bond formation
between N and F
Potassium permanganate is prepared by fusion of MnO2 with an alkali metal hydroxide and an
oxidising agent like KNO3. This produces the dark green K2MnO4 which disproportionates in a
neutral or acidic solution to give permanganate.

1
1

Oxalate ion or oxalic acid is oxidised at 333 K:


1

OR
9

i)
1

ii)
1

10

11

r = 1.44 x

1
1

r=
r=

12

cm

cane sugar = X
Therefore, ccane sugar = cX

(where c is molar concentration)


1

gmol-1

MX =

13

MX = 59.9 or 60 gmol-1

k=

log

60 s-1 =
t=

log
log 10

t=

t= 0.0384 s
14

i)
ii)

It is a process of removing the dissolved substance from a colloidal solution by means


of diffusion through a semi - permeable membrane.
The movement of colloidal particles under an applied electric potential towards
oppositely charged electrode is called electrophoresis.
2

1
1
1

iii)
15

i)
ii)

Colloidal particles scatter light in all directions in space. This scattering of light
illuminates the path of beam in the colloidal dispersion.
It lowers the melting point of alumina / acts as a solvent.
Roasting
Ore is heated in a regular supply of air

iii)

Calcination
Heating in a limited supply or
absence of air.

1
1
1

(Or with equation)


It is a process of separation of different components of a mixture which are differently
1

adsorbed on a suitable adsorbent.


OR
15

16

6x
=3

(any 6 correct equations)


Disproportionation : The reaction in which an element undergoes self-oxidation and self- 1
reduction simultaneously. For example
2Cu+ (aq)

Cu2+ (aq) + Cu(s)

(Or any other correct equation)


17

18

i)

Hexaamminecobalt(III) chloride

ii)

Tetrachlorido nickelate(II)

iii)

Potassium hexacyanoferrate(III)

i)

2-bromobutane

ii)

1, 3-dibromobenzene

iii)

3-choloropropene

1
1
1
1
1
1

19

i)
1
ii)

20

1
i)
ii)
1

iii)
1
21

22

Peptide linkage in proteins, -amino acids are connected to each other by peptide
bond or peptide linkage (-CONH- bond).
ii)
Primary structure - each polypeptide in a protein molecule having amino acids which
are linked with each other in a specific sequence.
iii)
Denaturation - When a protein is subjected to physical change like change in
temperature or chemical change like change in pH, protein loses its biological activity.
Copolymerisation is a polymerisation reaction in which a mixture of more than one monomeric
species is allowed to polymerise and form a copolymer.
i)

1
1
1
1

(or any other correct example)

23

24

i)
Aspartame, Saccharin (any one)
ii)
No
iii)
Social concern, empathy, concern, social awareness (any 2 )
0
0
E cell = E Sn2+ / Sn - E0Zn2+ / Zn
= - 0.14V (- 0.76V)
= 0.62V
0
rG = -n F E0cell
= - 2 x 96500 C mol-1 x 0.62 V
= - 119660 J mol-1

1
1
2
1
1
1
1

24

Ecell = E0cell -

log

Ecell = 0.62 -

log

OR
a) The conductivity of a solution at any given concentration is the conductance of one unit
volume of solution kept between two platinum electrodes with unit area of cross section
and at a distance of unit length.
Molar conductivity of a solution at a given concentration is the conductance of the volume
V of solution containing one mole of electrolyte kept between two electrodes with area of
cross section A and distance of unit length.
Molar conductivity increases with decrease in concentration.
b)E0cell = E0C - E0A
= 0.80V 0.77V
= 0.03V
rG0 = -n F E0cell
= - 1 x 96500 C mol-1 x 0.03 V
= - 2895 J mol-1

25

Log Kc=

Log Kc=
Log Kc= 0.508
a) Due to relatively stable half filled p-orbitals of group 15 elements
b) i) CaF2 + H2SO4 CaSO4 + 2HF

ii)

2
1
1
1

iii)
OR

25

a) i)

ii)

1
b) i)Due to small size of nitrogen, the lone pair of electron on nitrogen is localized/ easily
5

available for donation.


ii)Because they need only one electron to attain stable/noble gas configuration.

26

iii)
a) i)

ii)

1
b) i)Add NaHCO3, benzoic acid will give brisk effervescence of CO2 whereas ethylbenzoate
will not.
ii)Add NaOH and I2, acetophonone forms yellow ppt of iodoform on heating whereas
benzaldehyde will not.
iii)Add neutral FeCl3, phenol gives violet colouration whereas benzoic acid does not.
(or any other correct test)

1
1
1

OR

26

a) i)

ii)

b) i)

ii)

iii)
1

Dr. Sangeeta Bhatia

Sh. S.K. Munjal

Ms. Garima Bhutani

Sh. D.A. Mishra

SET 1
.

Series : SSO/1/C

Code No.

56/1/1


- -

.
Roll No.

Candidates must write the Code on


the title page of the answer-book.

- 11
- - -
- 26
,
- 15 - 10.15
10.15 10.30 -
-

Please check that this question paper contains 11 printed pages.


Code number given on the right hand side of the question paper should be written on the
title page of the answer-book by the candidate.
Please check that this question paper contains 26 questions.
Please write down the Serial Number of the question before attempting it.
15 minutes time has been allotted to read this question paper. The question paper will be
distributed at 10.15 a.m. From 10.15 a.m. to 10.30 a.m., the students will read the
question paper only and will not write any answer on the answer-book during this period.

()
CHEMISTRY (Theory)
: 3 ]

Time allowed : 3 hours ]

: 70

[ Maximum Marks : 70

:
(i)

(ii)

- 1 5 - 1
- 6 10 - 2
- 11 22 - 3
- 23 4
- 24 26 - 5

(iii)
(iv)
(v)
(vi)
(vii)
56/1/1

[P.T.O.

General Instructions :
(i)

All questions are compulsory.

(ii)

Q. No. 1 to 5 are very short answer questions and carry 1 mark each.

(iii) Q. No. 6 to 10 are short answer questions and carry 2 marks each.
(iv) Q. No. 11 to 22 are also short answer questions and carry 3 marks each.
(v)

Q. No. 23 is a value based question and carry 4 marks.

(vi) Q. No. 24 to 26 are long answer questions and carry 5 marks each.
(vii) Use log tables if necessary, use of calculator is not allowed.

: (z) ?

1.

What is the no. of atoms per unit cell (z) in a body-centred cubic structure ?

2.

In reference to surface chemistry, define dialysis.

[Ni(NH3)6]Cl2 (IUPAC)

3.

What is the IUPAC name of the complex [Ni(NH3)6]Cl2 ?

4.

3-
Draw the structure of 3-methylpentanal.

5.

C6H5N2Cl + H3PO2 + H2O - - Complete the following reaction equation :


C6H5N2Cl + H3PO2 + H2O - - 56/1/1

6.

Define osmotic pressure of a solution. How is the osmotic pressure related to the
concentration of a solute in a solution ?

7.

(i)

(t)

(ii)

(k)

Define the following terms :


(i)

Half-life of a reaction (t)

(ii)

Rate constant (k)

8.

(i)

H2SO4

(ii)

XeF2

Draw the structures of the following :

9.

(i)

H2SO4

(ii)

XeF2

:
(i)

(ii)

What is meant by disproportionation ? Give an example of a disproportionation


reaction in aqueous solution.
OR
Suggest reasons for the following features of transition metal chemistry :
(i)

The transition metals and their compounds are usually paramagnetic.

(ii)

The transition metals exhibit variable oxidation states.

56/1/1

[P.T.O.

10.

:H+
CH3CH2OH CH2 = CH2 + H2O
443 K
Explain the mechanism of dehydration steps of ethanol :H+
CH3CH2OH CH2 = CH2 + H2O
443 K

11.

(i)

(ii)

(iii) F-
Define the following :
(i)

Schottky defect

(ii)

Frenkel defect

(iii) F-centre

(C2H4O2) 45 g 600 g

12.

(i)

(ii)


( : Kf = 1.86 K kg mol1)

45 g of ethylene glycol (C2H4O2) is mixed with 600 g of water. Calculate


(i)

the freezing point depression and

(ii)

the freezing point of the solution


(Given : Kf of water = 1.86 K kg mol1)

13.

700 K :
, Ea (R = 8.314 J K1 mol1)

500 K

0.02 s1

0.07 s1

The rate constants of a reaction at 500 K and 700 K are 0.02 s1 and 0.07 s1
respectively. Calculate the value of activation energy, Ea. (R = 8.314 J K1 mol1)
56/1/1

14.

(i)

(ii)

(iii)

- ( )

Define the following terms :


(i)

Electrophoresis

(ii)

Adsorption

(iii) Shape selective catalysis

15.

(i)

(ii)

(iii)


(Pig) ?
Outline the principles of refining of metals by the following methods :
(i)

Distillation

(ii)

Zone refining

(iii) Electrolysis
OR
Write down the reactions taking place in different zones in the blast furnace during the
extraction of iron. How is pig iron different from cast iron ?

? ?

16.

What is lanthanoid contraction ? What are the consequences of lanthanoid contraction ?


56/1/1

[P.T.O.

17.

(i)

[Co(NH3)5(NO2)]2+

(ii)

[Co(en)3]Cl3 (en =

(iii) [Pt(NH3)2Cl2]
Indicate the types of isomerism exhibited by the following complexes :
(i)

[Co(NH3)5(NO2)]2+

(ii)

[Co(en)3]Cl3 (en = ethylene diamine)

(iii) [Pt(NH3)2Cl2]

(IUPAC) :

18.

(i)

CH3 CH CH2 CH3


|
OH

(ii)

(iii) CH3

CH3
|
C CH2 Cl
|
CH3

Name the following according to IUPAC system :


(i)

CH3 CH CH2 CH3


|
OH

(ii)

(iii) CH3

56/1/1

CH3
|
C CH2 Cl
|
CH3
6

19.

(i)

-2-

(ii)

(iii)

p-

How are the following conversions carried out ?


(i)

Propene to propane-2-ol

(ii)

Benzyl chloride to Benzyl alcohol

(iii) Anisole to p-Bromoanisole

20.

A B
Br2 KOH C C C6H7N
A, B C (IUPAC)

An aromatic compound A on treatment with aqueous ammonia and heating forms
compound B which on heating with Br2 and KOH forms a compound C of
molecular formula C6H7N. Write the structures and IUPAC names of compounds A, B
and C.

21.

How are vitamins classified ? Name the vitamin responsible for the coagulation of
blood.

22.

(i)

-S

(ii)

(iii)

Write the names and structures of the monomers of the following polymers :
(i)

Buna-S

(ii)

Neoprene

(iii) Teflon
56/1/1

[P.T.O.

23.

(i)

(ii)

(iii)

Ramesh went to a departmental store to purchase groceries. On one of shelves he


noticed sugar-free tablets. He decided to buy them for his grandfather who was a
diabetic. There were three types of sugar-free tablets. Ramesh decided to buy
sucrolose which was good for his grandfathers health.
(i)

Name another sugar free tablet which Ramesh did not buy.

(ii)

Was it right to purchase such medicines without doctors prescription ?

(iii) What quality of Ramesh is reflected above ?

24.

(a)

(b)

:
(i)

(^m)

(ii)

(iii)

:
(i)

(ii)

(a)

(b)


Ni(s) | Ni2+(aq) || Ag+(aq) | Ag(s)

25 C
?
E
56/1/1

= 0.25 V, E + = 0.80 V.
Ni2+/Ni
Ag /Ag
8

(a)

Define the following terms :


(i)

Molar conductivity (^m)

(ii)

Secondary batteries

(iii) Fuel cell


(b)

State the following laws :


(i)

Faraday first law of electrolysis

(ii)

Kohlrauschs law of independent migration of ions


OR

(a)

Define the term degree of dissociation. Write an expression that relates the molar
conductivity of a weak electrolyte to its degree of dissociation.

(b)

For the cell reaction


Ni(s) | Ni2+(aq) || Ag+(aq) | Ag(s)
Calculate the equilibrium constant at 25 C. How much maximum work would
be obtained by operation of this cell ?
E

= 0.25 V and E + = 0.80 V.


Ni2+/Ni
Ag /Ag

25.

(a)

(b)

:
(i)

Cu + HNO3()

(ii)

P4 + NaOH+ H2O

(i)

R3P = O R3N = O ? (R = )

(ii)

(iii)

(a)

(b)

56/1/1

:
(i)

(ii)

H2SO4

, ,

9

[P.T.O.

(a)

(b)

Complete the following chemical reaction equations :


(i)

Cu + HNO3(dilute)

(ii)

P4 + NaOH+ H2O

(i)

Why does R3P = O exist but R3N = O does not ? (R = alkyl group)

(ii)

Why is dioxygen a gas but sulphur a solid ?

(iii) Why are halogens coloured ?


OR
(a)

26.

Write balanced equations for the following reactions :


(i)

Chlorine reacts with dry slaked lime.

(ii)

Carbon reacts with concentrated H2SO4.

(b)

Describe the contact process for the manufacture of sulphuric acid with special
reference to the reaction conditions, catalysts used and the yield in the process.

(a)

(b)

(i)

(ii)

?
(i)

(ii)

m-

(iii)

3-

(a)

56/1/1

:
(i)

(ii)


10

(b)

:
(i)

LiAlH4
?
CH3 C CH3
||
O

CHO
HNO3 / H2SO4
?
273 283 K

(ii)

PCl5
(iii) CH3 COOH
?
(a)

(b)

Describe the following giving chemical equations :


(i)

De-carboxylation reaction

(ii)

Friedel-Crafts reaction

How will you bring about the following conversions ?


(i)

Benzoic acid to Benzaldehyde

(ii)

Benzene to m-Nitroacetophenone

(iii) Ethanol to 3-Hydroxybutanal


OR
(a)

Describe the following actions :


(i)

(b)

Acetylation

(ii)

Aldol condensation

Write the main product in the following equations :


(i)

LiAlH4
CH3 C CH3
?
||
O

CHO
(ii)

HNO3 / H2SO4
?
273 283 K

PCl5
(iii) CH3 COOH
?
___________
56/1/1

11

[P.T.O.

56/1/1

12

CHEMISTRY MARKING SCHEME


SET -56/1/1
Qu
es.

Marks

Value points

2
3

It is a process of removing a dissolved substance from a colloidal solution by means of diffusion 1


through a suitable membrane.
Hexaamninenickel (II) chloride
1

6.

7.

8.

(where Ar is C6H5)
The external pressure which is applied on solution side to stop the flow of solvent across the
semi-permeable membrane.

The osmotic pressure is directly proportional to concentration of the solution. /

The half-life of a reaction is the time in which the concentration of a reactant is reduced to one- 1
half of its initial concentration.
Rate constant is the rate of reaction when the concentration of the reactant is unity.
1
1+1

i)
9

= CRT

ii)

Disproportionation : The reaction in which an element undergoes self-oxidation and self- 1


reduction simultaneously. For example
2Cu+ (aq)

Cu2+ (aq) + Cu(s)

(Or any other correct equation)


OR
9

i)
ii)

Due to presence of unpaired electrons in d-orbitals.


Due to incomplete filling of d-orbitals.

1
1

10

11

12

i)
The defect in which equal number of cations and anions are missing from the lattice.
ii)
Due to dislocation of smaller ion from its normal site to an interstitial site.
iii)
Anionic vacancies are occupied by unpaired electron.
i) Tf = Kf m
Tf = Kf

1
1
1

Tf =
1

Tf =2.325K or 2.3250 C
ii) Tf0- Tf = 2.3250 C
O0C - Tf = 2.3250 C
Tf = - 2.3250 C or 270.675 K

13

1
14

i) The movement of colloidal particles under an applied electric potential towards oppositely
charged electrode is called electrophoresis.
ii) The accumulation of molecular species at the surface rather than in the bulk of a solid or liquid
is termed adsorption.
iii) The catalytic reaction that depends upon the pore structure of the catalyst and the size of the
reactant and product molecules is called shape-selective catalysis.
15 i) The impure metal is evaporated to obtain the pure metal as distillate.
ii) This method is based on the principle that the impurities are more soluble in the melt than in
the solid state of the metal.
iii) The impure metal is made to act as anode. A strip of the same metal in pure form is used as
cathode. They are put in a suitable electrolytic bath containing soluble salt of the same metal.
The more basic metal remains in the solution and the less basic ones go to the anode mud.
OR

1
1
1

1
1

15

x4
=2

( any four correct equations)


Cast iron has lower carbon content (about 3%) than pig iron / cast iron is hard & brittle whereas
pig iron is soft.

16

17

18

19

The steady decrease in atomic radii from La to Lu due to imperfect shielding of 4f orbital.
Consequences
i)
Members of third transition series have almost identical radii as coresponding members
of second transition series.
ii)
Difficulty in separation.

a)
b)
c)
a)
b)
c)
i)

1
1
1
1
1
1

Linkage isomerism
Optical isomerism
Cis - trans / Geometrical isomerism
Butan 2 ol
2 bromotoluene
2, 2-dimethylchlorpropane

1+1

ii)

1
iii)

1
+

20

Benzoic acid
3

C-

21

22

Benzamide

Aniline

Fat soluble vitamin- Vitamin A, D


Water soluble vitamin-Vitamin B,C
Vitamin K
i)

+
+
1
+

ii)
+

iii)

23

24

i)
Aspartame, Saccharin (any one)
ii)
No
iii)
Social concern, empathy, concern, social awareness (any 2 )
a)i)Molar conductivity of a solution at a given concentration is the conductance of the volume V
of solution containing one mole of electrolyte kept between two electrodes with area of cross
section A and distance of unit length.
ii) Secondary battery- can be recharged by passing current through it in opposite direction so that
it can be used again.
iii) Galvanic cells that are designed to convert the energy of combustion of fuels like hydrogen,
methane, methanol, etc. directly into electrical energy are called fuel cells.
b)i) The amount of chemical reaction which occurs at any electrode during electrolysis by a
current is proportional to the quantity of electricity passed through the electrolyte (solution or
melt).
ii) Limiting molar conductivity of an electrolyte can be represented as the sum of the individual
contributions of the anion and cation of the electrolyte.
OR
4

1
1
2
1

1
1

24

a) Degree of dissociation is the extent to which electrolyte gets dissociated into its constituent

ions.

1
1

b) E0cell = E0Ag+ / Ag - E0Ni2+ / Ni


= 0.80V 0.25V
= 0.55V

1og Kc =
=
log Kc = 18.644
G0 = - nFE0cell
= -2x96500 Cmol-1 x 0.55V
= -106,150 Jmol-1
Max.work =+106150 Jmol-1 or 106.150k Jmol-1
25

1
1
1

a) i)
ii)

b) i) Due to absence of d-orbital, nitrogen cannot expand its valency beyond four.
ii) Because of p p multiple bonding in dioxygen which is absent in sulphur.
iii) Due to excitation of electron by absorption of radiation from visible region.

1
1
1

OR

25

1
1

a) i)
ii)
b) It is manufactured by Contact Process which involves following steps:

i) burning of sulphur or sulphide ores in air to generate SO2.


ii) conversion of SO2 to SO3 by the reaction with oxygen in the presence of a catalyst (V2O5)
iii) absorption of SO3 in H2SO4 to give Oleum (H2S2O7). The oleum obtained is diluted to give
sulphuric acid

26

Reaction condition pressure of 2 bar and temperature of 720 K


Catalyst used is V2O5
Yield 96 98% pure
a) i) Carboxylic acids lose carbon dioxide to form hydrocarbons when their sodium salts are
heated with sodalime (NaOH and CaO).

ii) When the alkyl / acyl group is introduced at ortho and para positions by reaction
with alkyl halide / acyl halide in the presence of anhydrous aluminium chloride (a Lewis
acid) as catalyst.

1
1

1
1

(Note : Award full marks if correct equation is given )


b) i)

ii)
1

iii)

(or any other correct method)


OR
a) i) When the acyl groups are introduced at ortho and para positions by reaction with acyl halide in the
presence of anhydrous aluminium chloride (a Lewis acid) as catalyst.

26

-hydrogen undergo a reaction in the presence of


dilute alkali as catalyst to form -hydroxy aldehydes (aldol) or
hydroxy ketones (ketol),
respectively.

ii) Aldehydes and ketones having at least one

(Note : Award full marks if correct equation is given )


b)i)
1

ii)

iii)

CH3COCl

SET-3

H$moS> Z.

Series SSO/2

Code No.

amob Z.

56/2/3/F

narjmWu H$moS >H$mo Cma-nwpVH$m Ho$ _wI-n


>na Ad` {bIo &

Roll No.

Candidates must write the Code on the


title page of the answer-book.

H$n`m OmM H$a b| {H$ Bg Z-n _o _w{V n> 15 h &


Z-n _| Xm{hZo hmW H$s Amoa {XE JE H$moS >Z~a H$mo N>m Cma -nwpVH$m Ho$ _wI-n> na
{bI| &
H$n`m OmM H$a b| {H$ Bg Z-n _| >26 Z h &
H$n`m Z H$m Cma {bIZm ew$ H$aZo go nhbo, Z H$m H$_mH$ Ad` {bI| &
Bg Z-n H$mo nT>Zo Ho$ {bE 15 {_ZQ >H$m g_` {X`m J`m h & Z-n H$m {dVaU nydm
_| 10.15 ~Oo {H$`m OmEJm & 10.15 ~Oo go 10.30 ~Oo VH$ N>m Ho$db Z-n H$mo nT>|Jo
Ama Bg Ad{Y Ho$ XmamZ do Cma-nwpVH$m na H$moB Cma Zht {bI|Jo &
Please check that this question paper contains 15 printed pages.
Code number given on the right hand side of the question paper should be
written on the title page of the answer-book by the candidate.
Please check that this question paper contains 26 questions.
Please write down the Serial Number of the question before
attempting it.
15 minute time has been allotted to read this question paper. The question
paper will be distributed at 10.15 a.m. From 10.15 a.m. to 10.30 a.m., the
students will read the question paper only and will not write any answer on
the answer-book during this period.

agm`Z {dkmZ (gmpVH$)


CHEMISTRY (Theory)

{ZYm[aV g_` : 3 KQ>o

A{YH$V_ AH$ : 70

Time allowed : 3 hours


56/2/3/F

Maximum Marks : 70
1

P.T.O.

gm_m` {ZX}e :
(i)

g^r Z A{Zdm` h &

(ii)

Z g`m 1 go 5 VH$ A{V bKw-Cmar` Z h Ama `oH$ Z Ho$ {bE 1 AH$ h &

(iii)

Z g`m 6 go 10 VH$ bKw-Cmar` Z h Ama `oH$ Z Ho$ {bE 2 AH$ h &

(iv)

Z g`m 11 go 22 VH$ ^r bKw-Cmar` Z h Ama `oH$ Z Ho$ {bE 3 AH$ h &

(v)

Z g`m 23 _y`mYm[aV Z h Ama BgHo$ {bE 4 AH$ h &

(vi)

Z g`m 24 go 26 VH$ XrK-Cmar` Z h Ama `oH$ Z Ho$ {bE 5 AH$ h &

(vii)

`{X Amd`H$Vm hmo, Vmo bmJ Q>o~bm| H$m `moJ H$a| & H$Hw$boQ>am| Ho$ Cn`moJ H$s AZw_{V
Zht h &

General Instructions :
(i)

All questions are compulsory.

(ii)

Questions number 1 to 5 are very short answer questions and carry


1 mark each.

(iii)

Questions number 6 to 10 are short answer questions and carry 2 marks


each.

(iv)

Questions number 11 to 22 are also short answer questions and carry


3 marks each.

(v)

Question number 23 is a value based question and carry 4 marks.

(vi)

Questions number 24 to 26 are long answer questions and carry 5 marks


each.

(vii)

Use log tables, if necessary. Use of calculators is not allowed.

1.

Cg `m{JH$ H$m gy `m h {Og_| Vd Y


AQ>\$bH$s` [a>{$ H$m 2/3dm ^mJ KoaVo h ?

ccp

OmbH$ ~ZmVm h Ama

Ho$ na_mUw

What is the formula of a compound in which the element Y forms ccp


lattice and atoms of X occupy 2/3rd of octahedral voids ?
56/2/3/F

2.

{XE JE `m{JH$ H$m AmB.`y.nr.E.gr. Zm_ {b{IE :

HO CH2 CH = C CH3
CH3
Write the IUPAC name of the given compound :
HO CH2 CH = C CH3
CH3

3.

^m{VH$emofU CH$_Ur` h O~{H$ amgm`{ZH$emofU AZwH$_Ur` hmoVm h & `m| ?

Physisorption is reversible while chemisorption is irreversible. Why ?


4.

{ZZ{b{IV `w_ _| H$mZ

A{^{H$`m A{YH$ VrdVm go H$aoJm Ama `m| ?


Ama CH3 CH2 I

SN2

CH3 CH2 Br

Which would undergo SN2 reaction faster in the following pair and why ?
CH3 CH2 Br and CH3 CH2 I
5.

gm_m` Vmn_mZ na g\$a H$m H$mZ-gm Ana$n (EbmoQ>mon) D$_r` $n go Wm`r h ?

Which allotrope of sulphur is thermally stable at room temperature ?

6.

(a)

Obr` H$mna(II) bmoamBS> {db`Z Ho$ {dwV-AnKQ>Z Ho$ XmamZ H$WmoS> na


{ZZ{b{IV A{^{H$`mE hmoVr h :
Cu2+ (aq) + 2e
Cu(s)
H+ (aq) + e

1
H (g)
2 2

E0 = + 034 V
E0 = 000 V

CZHo$ _mZH$ AnM`Z BboQ>moS> {d^d (E0) Ho$ _mZm| Ho$ AmYma na H$WmoS> na {H$g
A{^{H$`m H$s g^mdZm (gwgJVVm) h Ama `m| ?
(b)

56/2/3/F

Am`Zm| Ho$ dV A{^J_Z Ho$ H$mobamD$e {Z`_ H$m H$WZ H$s{OE & BgH$m EH$
AZw`moJ {b{IE &
3

P.T.O.

(a)

Following reactions occur at cathode during the electrolysis of


aqueous copper(II) chloride solution :
Cu2+ (aq) + 2e
Cu(s)
H+ (aq) + e

1
H (g)
2 2

E0 = + 034 V
E0 = 000 V

On the basis of their standard reduction electrode potential (E0)


values, which reaction is feasible at the cathode and why ?
(b)

State Kohlrausch law of independent migration of ions. Write its


one application.

7.

gH$_U Vd `m| n[adVu CnM`Z AdWmAmo H$mo X{eV H$aVo h


H$mZ-gm Vd gdm{YH$ CnM`Z AdWmE XemVm h Ama `m| ?

? 3d loUr

_|

(Sc go Zn)
2

Why do transition elements show variable oxidation states ? In 3d series


(Sc to Zn), which element shows the maximum number of oxidation
states and why ?

8.

(i)

{ZZ{b{IV H$mbog H$m AmB.`y.nr.E.gr. Zm_ {b{IE

[Cr (en)3]Cl3
(ii)

(i)

{ZZ{b{IV H$mbog H$m gy {b{IE :


nmoQ>{e`_ Q>mB AmgbQ>mo H$mo_oQ>(III)
Write down the IUPAC name of the following complex :
[Cr (en)3]Cl3

(ii)

Write the formula for the following complex :


Potassium tri oxalato chromate(III)

56/2/3/F

9.

{ZZ{b{IV A{^{H$`mAm| _| `moJ AmZo dmbo A{^H$maH$m| Ho$ Zm_ Xr{OE :


(i)

?
CH3 CHO CH3 CH CH3
|

OH
(ii)

?
CH3 COOH CH3 COCl

Name the reagents used in the following reactions :


(i)

(ii)
10.

?
CH3 CHO CH3 CH CH3
|
OH
?
CH3 COOH CH3 COCl

amCQ> Ho$ {Z`_ go G$Um_H$ {dMbZ go `m Vmn` h ? EH$ CXmhaU Xr{OE & G$Um_H$
{dMbZ Ho$ {bE mixH H$m `m {M hmoVm h ?
AWdm
E{OAmoQ>mon H$mo n[a^m{fV H$s{OE & amCQ> Ho$ {Z`_ go G$Um_H$ {dMbZ mam ~ZZo dmbm
E{OAmoQ>mon {H$g H$ma H$m hmoVm h ? EH$ CXmhaU Xr{OE &

2>

What is meant by negative deviation from Raoults law ? Give an


example. What is the sign of mixH for negative deviation ?
OR

11.

Define azeotropes. What type of azeotrope is formed by negative


deviation from Raoults law ? Give an example.
(a)
EpH$b hbmBS>| Ob _| KwbZerb Zht h & `m| ?
(b)
(c)
(a)
(b)
(c)

56/2/3/F

`wQ>Z-1-Amb H$m{eH$s` {ZpH$` (YwdU AKyUH$) h naVw `wQ>Z-2-Amb


H$m{eH$s` g{H$` (YwdU KyUH$) h & `m| ?
`{n bmoarZ BboQ>mZ H$mo AmH${fV H$aZo dmbm Jwn h {\$a ^r `h BboQ>mZZohr
Eamo_{Q>H$ {VWmnZ A{^{H$`mAm| _| AmWm}- VWm nam- {ZX}eH$ h & `m| ?

Why are alkyl halides insoluble in water ?


Why is Butan-1-ol optically inactive but Butan-2-ol is optically
active ?
Although chlorine is an electron withdrawing group, yet it is
ortho-, para- directing in electrophilic aromatic substitution
reactions. Why ?
5

P.T.O.

12.

{ZZ{b{IV H$m $nmVaU Amn H$go H$a|Jo :


(i)
~oOmoBH$ E{gS> H$mo ~oOpS>hmBS> _|
(ii)
EWmBZ H$mo EWZb _|
(iii) Eogr{Q>H$ E{gS> H$mo _rWoZ _|
AWdm
{ZZ{b{IV A{^{H$`mAm| go g~pYV g_rH$aUm| H$mo {b{IE :
(i)
Q>r\$Z A{^{H$`m
(ii)
dmo\$-{H$Za AnM`Z
(iii) EQ>mS> A{^{H$`m

How do you convert the following :


(i)

Benzoic acid to Benzaldehyde

(ii)

Ethyne to Ethanal

(iii)

Acetic acid to Methane


OR

Write the equations involved in the following reactions :

13.

(i)

Stephen reaction

(ii)

Wolff-Kishner reduction

(iii)

Etard reaction

Ob _| NaCl (_mob `_mZ = 585 g mol1) H$s {H$VZr _mm KwbmB OmE {H$
{h_mH$ 2C KQ> OmE, `h _mZVo hE {H$ NaCl nyU $n go {dK{Q>V hmoVm h ?
(Kf Ob Ho$ {bE = 186 K kg mol1)
372 g

Calculate the mass of NaCl (molar mass = 585 g mol1) to be dissolved in


372 g of water to lower the freezing point by 2C, assuming that NaCl
undergoes complete dissociation. (Kf for water = 186 K kg mol1)

56/2/3/F

14.

{ZZ{b{IV ~hbH$m| Ho$ EH$bH$m| Ho$ Zm_ Ama CZH$s gaMZmE {b{IE :
(i)
Q>oarbrZ
(ii)
~Ho$bmBQ>
(iii) ~wZm-S
Write the names and structures of the monomers of the following
polymers :

15.

(i)

Terylene

(ii)

Bakelite

(iii)

Buna-S

(i)

{ZZ{b{IV _o H$mZ _moZmogHo$amBS> h :


Q>mM, _mQ>mog, \$Q>mog, gobwbmog
Abr` Eo{_Zmo Eo{gS>m| Ama jmar` Eo{_Zmo Eo{gS>m| Ho$ ~rM$ `m AVa hmoVm h ?
Cg {dQ>m{_Z H$m Zm_ {b{IE {OgH$s H$_r Ho$ H$maU _gyS>m| _| IyZ AmZo bJVm h &

(ii)
(iii)

16.

(i)

Which one of the following is a monosaccharide :


starch, maltose, fructose, cellulose

(ii)

What is the difference between acidic amino acids and basic amino
acids ?

(iii)

Write the name of the vitamin whose deficiency causes bleeding of


gums.

(i)

H$mbog [Pt(en)2Cl2]2+ Ho$ `m{_Vr` g_md`dm| H$mo Amao{IV H$s{OE &


{H$Q>b \$sS> {gmV Ho$ AmYma na `{X o > P h, Vmo d4 Am`Z H$m BboQ>m{ZH$
{d`mg {b{IE &
H$mbog [Ni(CN)4]2 H$m gH$aU H$ma Ama Mw~H$s` `dhma {b{IE &
(Ni H$m na_mUw H$_mH$ = 28)

(ii)
(iii)
(i)

Draw the geometrical isomers of complex [Pt(en)2Cl2]2+.

(ii)

On the basis of crystal field theory, write the electronic


configuration for d4 ion, if o P.

(iii)

Write the hybridization type and magnetic behaviour of the


complex [Ni(CN)4]2. (Atomic number of Ni = 28)

56/2/3/F

P.T.O.

17.

18.

(i)
(ii)

{ZH$b Ho$ n[aH$aU _| H$m_ AmZo dmbr {d{Y Ho$ nrN>o Omo {gmV hmoVm h CgH$m
CoI H$s{OE &
gmoZo Ho$ {ZH$fU _| VZw NaCN H$s `m ^y{_H$m hmoVr h ?

(iii)

H$mna _Q>o `m hmoVm h

(i)

Indicate the principle behind the method used for the refining of
Nickel.

(ii)

What is the role of dilute NaCN in the extraction of gold ?

(iii)

What is copper matte ?

25C

na {ZZ gob H$m {dwV-dmhH$ ~b (B.E_.E\$.) n[aH${bV H$s{OE

Zn | Zn2+ (0.001 M) | | H+ (0.01 M) | H2(g) (1 bar) | Pt(s)

E0

(Zn 2 / Zn )

= 0.76 V,

E0

(H / H )
2

= 0.00 V

Calculate the emf of the following cell at 25C :


Zn | Zn2+ (0.001 M) | | H+ (0.01 M) | H2(g) (1 bar) | Pt(s)

E0

(Zn 2 / Zn )

19.

= 0.76 V,

E0

(H / H )
2

= 0.00 V

{ZZ{b{IV A{^{H$`mAm| Ho$ CnmXm| H$s mJw{$ H$s{OE :


(i)

i) B H
CH3 CH = CH2 2 6 ?
ii) 3 H 2O 2 / OH

(ii)

Br2 (aq)
C6H5 OH
?

(iii)

Cu / 573 K
CH3CH2OH ?

Predict the products of the following reactions :


(i)

i) B H
CH3 CH = CH2 2 6 ?
ii) 3 H 2O 2 / OH

(ii)

Br2 (aq)
?
C6H5 OH

(iii)

Cu / 573 K
CH3CH2OH ?

56/2/3/F

20.

EH$ Vd X (_moba `_mZ = 60 g mol1) H$m KZd 623 g cm3 h & `{X `y{ZQ> gob
Ho$ H$moa H$s b~mB 4 108 cm h, Vmo `y{~H$ `y{ZQ> gob Ho$ H$ma H$s `m nhMmZ
hmoJr ?

An element X (molar mass = 60 g mol1) has a density of 6.23 g cm3.


Identify the type of cubic unit cell, if the edge length of the unit cell is
4 108 cm.
21.

(a)

(b)

{ZZ{b{IV H$mo Amn H$maU XoVo hE H$go g_PmEJo :


(i)
Mn H$m CV_ bwAmoamBS> MnF4 h O~{H$ CV_ AmgmBS>
h &
(ii) gH$_U YmVwE Ama CZHo$ `m{JH$ CoaH$ JwUY_ XemVo h &
{ZZ{b{IV g_rH$aU H$mo nyU H$s{OE
2

3 MnO 4
(a)

(b)

+ 4H+

How would you account for the following :


(i)

Highest fluoride of Mn is MnF4 whereas the highest oxide is


Mn2O7.

(ii)

Transition metals and their compounds show catalytic


properties.

Complete the following equation :


2

3 MnO 4

22.

Mn2O7

+ 4H+

{ZZ{b{IV AdbmoH$Zm| Ho$ {bE H$maUm| H$mo Xr{OE :


(i)

g_wr Ob Ama ZXr H$m Ob Ohm {_bVo h dhm EH$ S>oQ>m ~Z OmVm h &

(ii)

MmaH$mob H$s gVh na


hmoVr h &

(iii)

MyU {H$E hE nXmW A{YH$ ^mdembr A{YemofH$ hmoVo h &

56/2/3/F

N2

Jg H$s Anojm

NH3

Jg A{YH$ erKVm go A{Yemo{fV

P.T.O.

Give reasons for the following observations :

23.

(i)

A delta is formed at the meeting point of sea water and river


water.

(ii)

NH3 gas adsorbs more readily than N2 gas on the surface of


charcoal.

(iii)

Powdered substances are more effective adsorbents.

~m| _| _Yw_oh Ama CXmgr Ho$ ~T>Vo Ho$gm| H$mo XoIZo Ho$ ~mX EH$ {g Hy$b Ho$ q{gnb
lr MmonS>m Zo EH$ go{_Zma H$m Am`moOZ {H$`m {Og_| Chm|Zo ~m| Ho$ A{^^mdH$m| VWm A`
Hy$bm| Ho$ q{gnbm| H$mo Am_{V {H$`m & Chm|Zo Hy$bm| _| gS>o hE ^mo` nXmWm] (OH$ \y$S>)
na {V~Y bJmZo H$m {ZU` {b`m, gmW hr `h {ZU` {b`m {H$ Hy$bm| _| dm`dYH$
nXmW Ogo gyn, bgr, XY Am{X H$Q>rZm| _| CnbY H$amB OmE & Chm|Zo `h ^r {ZU`
{b`m {H$ mV:H$mbrZ Ego~br Ho$ g_` ~m| H$mo {V{XZ AmYo KQ>o H$s emar[aH$ H$gaV
^r H$amB OmE & N>: _mh nMmV lr MmonS>m Zo ~m| Ho$ dm` H$m A{YH$V_ {dmb`m| _|
nwZ: {ZarjU H$adm`m Ama ~m| Ho$ dm` _| AZwn_ gwYma nm`m J`m &
Cn`w$ H$aU H$mo nT>Zo Ho$ ~mX, {ZZ{b{IV Zm| Ho$ Cma Xr{OE

(i)

lr MmonS>m mam {H$Z _y`m| (H$_-go-H$_ Xmo) H$mo Xem`m J`m h

(ii)

EH$ {dmWu Ho$ $n _|, Amn Bg {df` _| H$go OmJ$H$Vm \$bmEJo

(iii)

AdZ_Z-{damoYr S>J {~Zm S>mQ>a H$s gbmh Ho$ `m| Zht boZo Mm{hE

(iv)

H${_ _YwaH$ Ho$ Xmo CXmhaU Xr{OE &

?
?

Seeing the growing cases of diabetes and depression among children,


Mr. Chopra, the principal of one reputed school organized a seminar in
which he invited parents and principals. They all resolved this issue by
strictly banning the junk food in schools and by introducing healthy
snacks and drinks like soup, lassi, milk etc. in school canteens. They also
decided to make compulsory half an hour physical activities for the
students

in

the

morning

assembly

daily.

After

six

months,

Mr. Chopra conducted the health survey in most of the schools and
discovered a tremendous improvement in the health of students.
56/2/3/F

10

After reading the above passage, answer the following questions :


(i)

What are the values (at least two) displayed by Mr. Chopra ?

(ii)

As a student, how can you spread awareness about this issue ?

(iii)

Why should antidepressant drugs not be taken without consulting


a doctor ?

24.

(iv)

Give two examples of artificial sweeteners.

(a)

`oH$ Ho$ {bE Cn`w$ CXmhaU XoVo hE {ZZ{b{IV A{^{H$`mAm| H$mo X{eV
H$s{OE :

(b)

(i)

A_moZrH$aU

(ii)

H$pbJ (`w_Z) A{^{H$`m

(iii)

Eo_rZm| H$m Eogr{Q>brH$aU

mW{_H$ (mB_ar), {Vr`H$ (goH$S>ar) Ama VVr`H$ (Q>{e`ar) E_rZm| H$s nhMmZ
H$aZo Ho$ {bE {hg~J {d{Y H$m dUZ H$s{OE & g~ A{^{H$`mAm| Ho$ amgm`{ZH$
g_rH$aUm| H$mo ^r {b{IE &

AWdm
(a)

(b)

O~ ~oOrZ S>mBEOmo{Z`_ bmoamBS> (C6 H5 N2Cl ) {ZZ{b{IV A{^H$maH$m| go


A{^{H$`m H$aVm h, V~ mV _w` CnmXm| H$s gaMZmE {b{IE :
(i)

HBF4 /

(ii)

Cu / HBr

{ZZ{b{IV A{^{H$`mAm| _|

A, B Ama C H$s

gaMZmE {b{IE :

5
56/2/3/F

11

P.T.O.

(a)

(b)

Illustrate the following reactions giving suitable example in each


case :
(i)

Ammonolysis

(ii)

Coupling reaction

(iii)

Acetylation of amines

Describe Hinsberg method for the identification of primary,


secondary and tertiary amines. Also write the chemical equations
of the reactions involved.
OR

(a)

Write the structures of main products when benzene diazonium


chloride (C6 H5 N 2 Cl ) reacts with the following reagents :

25.

(i)

HBF4 /

(ii)

Cu / HBr

(b)

Write the structures of A, B and C in the following reactions :

(a)

{ZZ{b{IV nXm| H$mo n[a^m{fV H$s{OE :


(i)
g{H$`U D$Om
(ii) Xa pWamH$

(b)

25%

{d`moOZ Ho$ {bE EH$ W_ H$mo{Q> H$s A{^{H$`m


A{^{H$`m Ho$ {bE t1/2 H$m n[aH$bZ H$s{OE >&
({X`m J`m

{_ZQ> boVr h &

: log 2 = 03010, log 3 = 04771, log 4 = 06021)

AWdm
56/2/3/F

10

12

(a)

EH$ amgm`{ZH$ A{^{H$`m


ZrMo bmQ> _| {X`m J`m h

R P Ho$
:

{bE gmU _| n[adVZ

ln [R] vs. g_` (s)

ln [R]
t (s)
(i)
(ii)
(iii)

(b)

(a)

(b)

A{^{H$`m H$s H$mo{Q> H$s mJw{$ H$s{OE &


dH$ H$m T>bmZ `m h ?
A{^{H$`m Ho$ {bE Xa pWamH$ H$s `y{ZQ> {b{IE &

XemBE {H$ 99% nyU hmoZo _| Omo g_` bJVm h dh Cg g_` H$m XwJwZm h Omo
A{^{H$`m Ho$ 90% nyU hmoZo _| bJVm h &

Define the following terms :


(i)

Activation energy

(ii)

Rate constant

A first order reaction takes 10 minutes for 25% decomposition.


Calculate t1/2 for the reaction.
(Given : log 2 = 03010, log 3 = 04771, log 4 = 06021)
OR

(a)

For a chemical reaction R P, the variation in the concentration,


ln [R] vs. time (s) plot is given as

ln [R]
t (s)
56/2/3/F

13

P.T.O.

26.

(i)

Predict the order of the reaction.

(ii)

What is the slope of the curve ?

(iii)

Write the unit of rate constant for this reaction.

(b)

Show that the time required for 99% completion is double of the
time required for the completion of 90% reaction.

(a)

{ZZ{b{IV Ho$ H$maU XoVo hE nQ> H$s{OE :

(b)

(i)

NH4

_| Omo Am~Y H$moU h dh NH3 Ho$ H$moU go CVa h &


(ii) H2O H$s Anojm H2S H$m dWZmH$ `yZVa h &
(iii) AnM`Z `dhma SO2 go TeO2 H$s Amoa KQ>Vm h &
{ZZ{b{IV H$s gaMZmE Amao{IV H$s{OE :
(i)
H4P2O7 (nm`amo\$m\$mo[aH$ Eo{gS>)
(ii)

XeF2

AWdm
(a)

(b)

(a)

56/2/3/F

{ZZ{b{IV H$s gaMZmE Amao{IV H$s{OE :


(i)

XeF4

(ii)

H2S2O7

{ZZ{b{IV Ho$ H$maU Xr{OE :


(i)
HCl go A{^{H$`m go Am`aZ FeCl2 ~ZmVm h Z H$s FeCl3.
(ii) HClO H$s Anojm HClO4 ~bVa Ab h &
(iii) dJ 15 Ho$ g^r hmBS>mBS>m| _| BiH3 ~bV_ AnMm`H$ h &
Account for the following :

(i)

Bond angle in NH4 is higher than NH3.

(ii)

H2S has lower boiling point than H2O.

(iii)

Reducing character decreases from SO2 to TeO2.

14

(b)

Draw the structures of the following :


(i)

H4P2O7 (Pyrophosphoric acid)

(ii)

XeF2
OR

(a)

(b)

56/2/3/F

Draw the structures of the following :


(i)

XeF4

(ii)

H2S2O7

Account for the following :


(i)

Iron on reaction with HCl forms FeCl2 and not FeCl3.

(ii)

HClO4 is a stronger acid than HClO.

(iii)

BiH3 is the strongest reducing agent amongst all the


hydrides of group 15.

15

P.T.O.

CHEMISTRY MARKING SCHEME 2015


SET -56/2/3 F
Qn

Marks

Value points

X2Y3

3-Methylbut-2-en-1-ol

Because of weak van der Waals forces in physisorption whereas there are strong chemical
forces in chemisorption.
CH3CH2I , because I is a better leaving group.

Rhombic sulphur

a) Cu2+ (aq) + 2 e

Cu(s) because of high E0 value/ more negative G

b) It states that limiting molar conductivity of an electrolyte is equal to the sum of the individual 1
contributions of cations and anions of the electrolyte.
It is used to calculate the m0 for weak electrolyte / It is used to calculate and Kc
(Any one application) 1
7

10

a) Due to presence of unpaired d-electrons/ comparable energies of 3d and 4s orbitals.


b) Mn , due to involvement of 4s and 3d electrons/ presence of maximum unpaired delectrons.
i) tris-(ethane-1,2-diamine)chromium(III) chloride

1
,

ii) K3[ Cr(C2O4)3]

(i) CH3MgBr/ H3O+

(ii) PCl5/ PCl3 / SOCl2

When solute- solvent interaction is stronger than pure solvent or solute interaction.

Eg: chloroform and acetone (or any other correct eg)

mixH= negative

OR
10

Azeotropes binary mixtures having same composition in liquid and vapour phase and boil at 1
constant temperature / is a liquid mixture which distills at constant temperature without
undergoing change in composition

Maximum boiling azeotropes

eg: HNO3 (68%) and H2O(32%) (or any other correct example)
11

12

a) Because they are unable to form H-bonds with water molecules.


b) Because of the presence of chiral carbon in butan-2-ol.
c) Due to dominating +R effect
i)
C6H5COOH PCl5 C6H5COCl H2/Pd
C6H5CHO

1
1
1
1

BaSO4
ii)

CHCH + H2O

iii)

CH3COOH

Hg2+ /H2SO4

NaOH

CH3COONa

CH3CHO
NaOH + CaO , heat

1
CH4

OR

i)

1
ii)
1
iii)
13

Tf = i. Kf m
= i Kf wB x 1000

MB x wA
2K= 2 x 1.86K kg/mol x wB x 1000

58.5 g/mol x 37.2 g


wB = 1.17g

14

1
i)

1
ii)
Phenol

and

formaldehyde

iii)
(Note: half mark for structure/s and half mark for name/s)
15

i)
ii)
iii)

Fructose
1
Acidic amino acid has more number of acidic carboxylic group than basic amino
1
group whereas basic amino acid has more number of basic amino group.
Vitamin C
1

16

i)
cis- isomer

17

trans-isomer

ii)

t2g4

iii)

dsp 2 , diamagnetic

a) Impure Ni reacts with CO to form volatile Ni(CO)4 which when heated at higher 1
temperature decomposes to give pure Ni.

18

b) NaCN acts as a leaching agent to form a soluble complex with gold.

c) It is a mixture of Cu2S and FeS

E cell = E0 cell

log

E cell = 0.76 V -

V log

E cell = 0.76 0.0295 V log 10


1

= 0.7305 V
19

i)

CH3CH2CH2OH

1
ii)

1
iii)
20

CH3CHO

d=
6.23 g cm-3 =

z=4

fcc

21

22

i)

Because oxygen stabilizes Mn more than F due to multiple bonding

ii)

Because of their ability to show variable oxidation state(or any other correct reason)

iii)

3MnO42- + 4H+

i)

Due to coagulation of colloidal clay particles.

ii)

Because NH3 is easily liquefiable than N2 due to its larger molecular size.

iii)

Because of more surface area.

2MnO4- + MnO2 + 2H2O

23

a) Concern for students health, Application of knowledge of chemistry to daily life, empathy ,
, caring or any other
b) Through posters, nukkad natak in community, social media, play in assembly (or any other
1
relevant answer)
1
c) Wrong choice and overdose may be harmful
+
d) Aspartame, saccharin (or any other correct example)

24

a) i)ammonolysis

ii)

(any one)

iii)

(or any other correct reaction)

b)reaction of primary amine


1

(soluble in alkali)
Reaction of secondary amine

(insoluble in alkali)
Tertiary amine doesnt react

OR
24

a) i)
1

ii)
,,

,,

b) i) Aii) A- CH3CN
25

B-

C-

B- CH3CH2NH2

C- CH3CH2OH

a)i) Activation energy- Extra energy required by reactants to form activated complex.
ii) Rate constant- rate of reaction when the concentration of reactant is unity.

1
1

b)

k= 2.303 log [ A0 ]
t
[A]
k = 2.303 log 100
10 min
75

k = 2.303 x 0.125
10 min
k = 0.02879 min-1
t1/2 =

t1/2

24.07min
1

OR
a) i)First order ii) -k

iii) s-1
1,1,1

b)
t=

log

t99% =
t=

log

x 2

t90% =

log

=
t99% = 2 x t90%
26

a) i)Because of lone pair in NH3 , lone pair- bond pair repulsion decreases the bond angle

ii)Because of absence of H-bonding in H2S

iii)Because stability of +4 oxidation state increases from SO2 to TeO2

1,1

b)
OR

a)

1,1

b)i)Because iron on reaction with HCl produces H2(g) which prevents the formation of FeCl2 to

FeCl3 / Because HCl is a weak oxidising agent.


ii) Because of higher oxidation state of chlorine in HClO4

iii) Because of lower dissociation enthalpy of Bi-H bond.

SET 1
.

Series : SSO/1

Code No.

56/1/1/D

- -

Roll No.

Candidates must write the Code on


the title page of the answer-book.

- 12
- - -
- 26
,
- 15 - 10.15
10.15 10.30 -
Please check that this question paper contains 12 printed pages.
Code number given on the right hand side of the question paper should be written on the
title page of the answer-book by the candidate.
Please check that this question paper contains 26 questions.
Please write down the Serial Number of the question before attempting it.
15 minutes time has been allotted to read this question paper. The question paper will be
distributed at 10.15 a.m. From 10.15 a.m. to 10.30 a.m., the students will only read the
question paper and will not write any answer on the answer-book during this period.

()
CHEMISTRY (Theory)
: 3 ]

[ : 70

Time allowed : 3 hours ]

[ Maximum Marks : 70

:
(i)

(ii)

- 1 5 - 1
- 6 10 - 2
- 11 22 - 3
- 23 4
- 24 26 - 5

(iii)
(iv)
(v)
(vi)
(vii)
56/1/1/D

[P.T.O.

General Instructions :
(i)

All questions are compulsory.

(ii)

Q. no. 1 to 5 are very short answer questions and carry 1 mark each.

(iii) Q. no. 6 to 10 are short answer questions and carry 2 marks each.
(iv)

Q. no. 11 to 22 are also short answer questions and carry 3 marks each.

(v)

Q. no. 23 is a value based question and carry 4 marks.

(vi)

Q. no. 24 to 26 are long answer questions and carry 5 marks each.

(vii) Use log tables if necessary, use of calculators is not allowed.

1.

H3PO4

What is the basicity of H3PO4 ?

2.

..... :

Write the IUPAC name of the given compound :

3.

SN2 ?
CH3 CH2 Br

CH3
|
CH3 C CH3
|
Br

Which would undergo SN2 reaction faster in the following pair and why ?
CH3
|
CH3 CH2 Br and CH3 C CH3
|
Br
56/1/1/D

4.

KCl
?

BaCl2

Out of BaCl2 and KCl, which one is more effective in causing coagulation of a
negatively charged colloidal Sol ? Give reason.
5.

Y ccp X 1/3
?

What is the formula of a compound in which the element Y forms ccp lattice and
atoms of X occupy 1/3rd of tetrahedral voids ?
6.

What are the transition elements ? Write two characteristics of the transition elements.
7.

(i)
(ii)

(i)

[Cr(NH3)2Cl2(en)]Cl

:
-o- (III).

(en = ethylenediamine)

.....

Write down the IUPAC name of the following complex :


[Cr(NH3)2Cl2(en)]Cl (en = ethylenediamine)

(ii)

Write the formula for the following complex :


Pentaamminenitrito-o-Cobalt (III).

8.

Name the reagents used in the following reactions :


?
(i) CH3 CO CH3 CH3 C H CH3
|
OH
?
(ii) C6H5 CH2 CH3 C6H5 COOK+
9.

?
mixH ?

What is meant by positive deviations from Raoults law ? Give an example. What is
the sign of mixH for positive deviation ?

/OR
56/1/1/D

[P.T.O.

9.

Define azeotropes. What type of azeotrope is formed by positive deviation from


Raoults law ? Give an example.

10.

(a)

:
Ag+(aq) + e Ag(s)

E = +0.80 V

1
H+(aq) + e H2(g)
2

E = 0.00 V


?
(b)

(a)

(E)

Following reactions occur at cathode during the electrolysis of aqueous silver


chloride solution :
Ag+(aq) + e Ag(s)

E = +0.80 V

1
H+(aq) + e H2(g)
2

E = 0.00 V

On the basis of their standard reduction electrode potential (E) values, which
reaction is feasible at the cathode and why ?
(b)

11.

Define limiting molar conductivity. Why conductivity of an electrolyte solution


decreases with the decrease in concentration ?

49 g 3.9 g 1.62 K
( )
( : = 122 g mol1, Kf = 4.9 K kg mol1)
3.9 g of benzoic acid dissolved in 49 g of benzene shows a depression in freezing point
of 1.62 K. Calculate the vant Hoff factor and predict the nature of solute (associated
or dissociated).
(Given : Molar mass of benzoic acid = 122 g mol1, Kf for benzene = 4.9 K kg mol1)

56/1/1/D

12.

(iii)


?
?

(i)

Indicate the principle behind the method used for the refining of zinc.

(ii)

What is the role of silica in the extraction of copper ?

(i)
(ii)

(iii) Which form of the iron is the purest form of commercial iron ?

13.

27 g mol1 4.05 108 cm


2.7 g cm3 , ?

An element with molar mass 27 g mol1 forms a cubic unit cell with edge length
4.05 108 cm. If its density is 2.7 g cm3, what is the nature of the cubic unit cell ?
14.

(a)

(b)

?
(i)
(ii)
:

2MnO4 + 6H+ + 5NO2


(a)

(b)

How would you account for the following :


(i)

Actinoid contraction is greater than lanthanoid contraction.

(ii)

Transition metals form coloured compounds.

Complete the following equation :

2MnO4 + 6H+ + 5NO2


15.

(i)

[Pt(NH3)2Cl2]

(ii)
(iii)

o < P d4
[Ni(CO)4] (.. Ni = 28)

(i)

Draw the geometrical isomers of complex [Pt(NH3)2Cl2].

(ii)

On the basis of crystal field theory, write the electronic configuration for d4 ion
if o < P.

(iii) Write the hybridization and magnetic behaviour of the complex [Ni(CO)4].
(At.no. of Ni = 28)
56/1/1/D

[P.T.O.

16.

25 C emf :
Fe | Fe2+(0.001 M) || H+(0.01 M) | H2(g) (1 bar) | Pt(s)
E(Fe2+ | Fe) = 0.44 V E(H+ | H2) = 0.00 V

Calculate emf of the following cell at 25 C :


Fe | Fe2+(0.001 M) || H+(0.01 M) | H2(g) (1 bar) | Pt(s)
E(Fe2+ | Fe) = 0.44 V E(H+ | H2) = 0.00 V

17.

:
(i)
(ii)
(iii) CO

Give reasons for the following observations :


(i)

Leather gets hardened after tanning.

(ii)

Lyophilic sol is more stable than lyophobic sol.

(iii) It is necessary to remove CO when ammonia is prepared by Habers process.

18.


(i) -6, 6
(ii)

PHBV

(iii)

Write the names and structures of the monomers of the following polymers :
(i)

Nylon-6, 6

(ii)

PHBV

(iii) Neoprene
19.

:
(i)

(ii)

CH3 C = O (i) H2N NH3


?
|
(ii) KOH/Glycol,
CH3
NaOH/I2
C6H5 CO CH3
?+?

NaOH / CaO
(iii) CH3 COONa ?

56/1/1/D

3
6

Predict the products of the following reactions :


(i)

(ii)

CH3 C = O (i) H2N NH3


?
|
(ii) KOH/Glycol,
CH3
NaOH/I2
C6H5 CO CH3
?+?

NaOH / CaO
(iii) CH3 COONa ?

20.

?
(i)
(ii) -2- 2--2-
(iii)

How do you convert the following :


(i)

Phenol to anisole

(ii)

Propan-2-ol to 2-methylpropan-2-ol

(iii) Aniline to phenol

/OR
20.

(a)

- :
H+
2CH3CH2OH CH3CH2 O CH2CH3

(b)

(a)

Write the mechanism of the following reaction :

H+
2CH3CH2OH CH3CH2 O CH2CH3

21.

(b)

Write the equation involved in the acetylation of Salicylic acid.

(i)

: , , , ?
?
?

(ii)
(iii)

(i)

Which one of the following is a disaccharide : Starch, Maltose, Fructose,


Glucose ?

(ii)

What is the difference between fibrous protein and globular protein ?

(iii) Write the name of vitamin whose deficiency causes bone deformities in children.
56/1/1/D

[P.T.O.

22.

:
(a) t- n-
(b)
(c) o/p
(NO2)

Give reasons :
(a)
(b)
(c)

23.

n-Butyl bromide has higher boiling point than t-butyl bromide.


Racemic mixture is optically inactive.
The presence of nitro group (NO2) at o/p positions increases the reactivity of
haloarenes towards nucleophilic substitution reactions.



-
, ,
:
: :

:
(i) ( ) ?
(ii) ?
(iii) ?
(iv) ?
Mr. Roy, the principal of one reputed school organized a seminar in which he invited
parents and principals to discuss the serious issue of diabetes and depression in
students. They all resolved this issue by strictly banning the junk food in schools and
to introduce healthy snacks and drinks like soup, lassi, milk etc. in school canteens.
They also decided to make compulsory half an hour physical activities for the students
in the morning assembly daily. After six months, Mr. Roy conducted the health survey
in most of the schools and discovered a tremendous improvement in the health of
students.
After reading the above passage, answer the following :
(i) What are the values (at least two) displayed by Mr. Roy ?
(ii) As a student, how can you spread awareness about this issue ?
(iii) What are tranquilizers ? Give an example.
(iv) Why is use of aspartame limited to cold foods and drinks ?

56/1/1/D

24.

(a)

(b)

(a)

:
(i)

HF HI

(ii)

(iii)

:
(i)

Cl F3

(ii)

XeF4

Account for the following :


(i)

Acidic character increases from HF to HI.

(ii)

There is large difference between the melting and boiling points of oxygen
and sulphur.

(iii) Nitrogen does not form pentahalide.


(b)

Draw the structures of the following :


(i)

Cl F3

(ii)

XeF4

/OR
24.

(i)

(ii)

?
Cl2 F2 ?

(iii)
(iv)

(v)

:
5

XeF2 + PF5
(i)

Which allotrope of phosphorus is more reactive and why ?

(ii)

How the supersonic jet aeroplanes are responsible for the depletion of ozone
layers ?

(iii) F2 has lower bond dissociation enthalpy than Cl2. Why ?


(iv) Which noble gas is used in filling balloons for meteorological observations ?
(v)

Complete the equation :


XeF2 + PF5

56/1/1/D

[P.T.O.

25.

C7H7ON

A, B, C, D E :

An aromatic compound A of molecular formula C7H7ON undergoes a series of


reactions as shown below. Write the structures of A, B, C, D and E in the following
reactions :

/OR
25.

(a)

:
(i)

Br2

(ii)

HCl

(iii) (CH3CO)2O / pyridine


(b)

:
C2H5NH2, C2H5OH, (CH3)3N

(c)

:
(CH3)2NH (CH3)3N

56/1/1/D

5
10

(a)

Write the structures of main products when aniline reacts with the following
reagents :
(i)

Br2 water

(ii)

HCl

(iii) (CH3CO)2O / pyridine


(b)

Arrange the following in the increasing order of their boiling point :


C2H5NH2, C2H5OH, (CH3)3N

(c)

Give a simple chemical test to distinguish between the following pair of


compounds :
(CH3)2NH and (CH3)3N

26.

- :
t/s
[CH3COOCH3]/mol L1

30

60

0.60

0.30

0.15

(i)

(ii)

30 60
(

log 2 = 0.3010, log 4 = 0.6021)

For the hydrolysis of methyl acetate in aqueous solution, the following results were
obtained :
t/s
[CH3COOCH3]/mol L1
(i)

30

60

0.60

0.30

0.15

Show that it follows pseudo first order reaction, as the concentration of water
remains constant.

(ii)

Calculate the average rate of reaction between the time interval 30 to 60 seconds.
(Given log 2 = 0.3010, log 4 = 0.6021)

/OR
56/1/1/D

11

[P.T.O.

26.

(a)

A + B P
= k[A] [B]2

(b)

(i)

B ?

(ii)

A ?

50% 30 90%

5

(log 2 = 0.3010)
(a)

For a reaction A + B P, the rate is given by


Rate = k[A] [B]2

(b)

(i)

How is the rate of reaction affected if the concentration of B is doubled ?

(ii)

What is the overall order of reaction if A is present in large excess ?

A first order reaction takes 30 minutes for 50% completion. Calculate the time
required for 90% completion of this reaction.
(log 2 = 0.3010)
_________

56/1/1/D

12

CHEMISTRY MARKING SCHEME


DELHI -2015
SET -56/1/3/D

Qu
es.

Answers

Marks

BaCl2 because it has greater charge / +2 charge

X2Y3

2, 5 - dinitrophenol

CH3-CH2-Br

Because it is a primary halide / (10) halide


6.

When vapour pressure of solution is higher than that predicted by Raoults law /

the intermolecular attractive forces between the solute-solvent/(A-B) molecules are weaker than
those between the solute-solute and solvent-solvent molecules/A-A or B-B molecules.
Eg. ethanol-acetone/ethanol-cyclohexane/CS2-acetone or any other correct example
mixH is positive

OR
(a)Azeotropes are binary mixtures having the same composition in the liquid and vapour phase 1
and boil at a constant temperature.

7.

8.

9.

(b) Minimum boiling azeotrope

eg - ethanol + water or any other example

(i)Ag+ (aq) + e- Ag (s)

0
0
Reaction with higher E value / G negative

(ii) Molar conductivity of a solution at infinite dilution or when concentration approaches


zero
Number of ions per unit volume decreases

Elements which have partially filled d-orbital in its ground states or any one of its oxidation
states.
1) Variable oxidation states
2) Form coloured ion
Or any other two correct characteristics
1) Diamminedichloridoethylenediaminechromium(III) chloride
2) [Co(NH3)5(ONO)]2+

1
+

1+ 1

10

11

12

13

(i)LiAlH4 / NaBH4 /H2, Pt

(ii)KMnO4 , KOH

(i)Hexamethylene diamine NH2 (CH2)6 NH2 and


adipic acid HOOC- (CH2)4- COOH
(ii)3 hydroxybutanoic acid CH3CH(OH)CH2COOH and
3 hydroxypentanoic acid CH3CH2CH(OH)CH2COOH
(iii)Chloroprene H2C=C(Cl)CH=CH2
IUPAC names are accepted
Note : mark for name /s and mark for structure / s
(i)CH3CH2CH3
(ii) C6H5COONa + CHI3
(iii)CH4
(i) C6H5OH + NaOH C6H5ONa
CH3X
C6H5OCH3
Or
C6H5OH + Na C6H5ONa
CH3X
C6H5OCH3

1
,
1

1
(ii)CH3CH(OH)CH3

CrO3 or Cu/573K CH3COCH3

(i)CH3MgX
(ii)H2O

(iii)C6H5NH2 NaNO2 + HCl C6H5N2Cl H2O warm


273K

C6H5OH

(CH3)2C(OH)CH3
1

OR
13

a)

1
b)

(Acetyl chloride instead of acetic anhydride may be used)


14

15

(i)Maltose

(ii) fibrous proteins: parallel polypeptide chain , insoluble in water


Globular proteins: spherical shape, soluble in water, (or any 1 suitable difference)
(iii) Vitamin D

(i)Larger surface area, higher van der Waals forces , higher the boiling point

(ii)Rotation due to one enantiomer is cancelled by another enantiomer

(iii) - NO2 acts as Electron withdrawing group or I effect

Tf = i Kf m
Tf = i Kf mb x1000
Mb x m a

16.

1.62 K = i x 4.9K kg mol-1 x

3.9 g
122 gmol-1

1000
49 kg

i = 0.506

Or by any other correct method

17

18

As i<1 , therefore solute gets associated.

(i) Zinc being low boiling will distil first leaving behind impurities/ or on electrolysis the pure
metal gets deposited on cathode from anode.
(ii)Silica acts as flux to remove iron oxide which is an impurity as slag or FeO + SiO2 FeSiO3
(iii)Wrought iron
d=zxM
a3 NA
z = d a3 NA
M
z = 2.7 g cm-3 x 6.022 x1023 mol-1 x ( 4.05 x 10-8cm)3
27 g mol-1

1
1
1

= 3.999 4
Face centered cubic cell/ fcc
19

(i) 5f orbital electrons have poor shielding effect than 4f


(ii)due to d-d transition / or the energy of excitation of an electron from lower d orbital to higher
d-orbital lies in the visible region /presence of unpaired electrons in the d-orbital.
(iii) 2 MnO4- + 6 H+ + 5 NO2- 2 Mn2+ + 3 H2O + 5 NO3-

1
1
1
1

20

(i)

1
1

(ii)t2 g3 e g1
(iii) sp3 , diamagnetic
21

The cell reaction : Fe(s) + 2H (aq) Fe2+ (aq) + H2(g)


+

Eocell = Eoc - Eoa


= [0-(-0.44)]V=0.44V
o

Ecell = Ecell - 0.059 log [ Fe2+]


+ 2
2
[H ]

Ecell = 0.44 V - 0.059 log ( 0.001 )


2
2
( 0.01 )
1
= 0.44 V - 0.059 log ( 10 )
2
= 0.44 V - 0.0295 V
= 0.410 V

22

23

(i) mutual coagulation


(ii)strong interaction between dispersed phase and dispersion medium or solvated layer
(iii)CO acts as a poison for catalyst or iron
(i)

Concern for students health, Application of knowledge of chemistry to daily life,


empathy , caring or any other
(ii)Through posters, nukkad natak in community, social media, play in assembly or any other
(iii)Tranquilizers are drugs used for treatment of stress or mild and severe mental disorders .. Eg:
equanil (or any other suitable example)
(iv) Aspartame is unstable at cooking temperature.

1
1
1
,
1
,
1

24
(a)
1

k = 2.303 log [ A0 ]
t
[A]
k = 2.303 log 0.60
30
0.30

k = 2.303 x
30

0.301 = 0.023 s-1

k = 2.303 log 0.60


60
0.15

k = 2.303 x 0.6021 = 0.023 s-1


60

As k is constant in both the readings, hence it is a pseudofirst order reaction.


ii)
Rate = - [R]/t
= -[0.15-0.30]
60-30

= 0.005 mol L-1s-1


OR
24.

a)
(i) Rate will increase 4 times of the actual rate of reaction.
(ii) Second order reaction

b)

t
1/2

= 0.693
k

30min =

0.693
k

1+1

k =

0.0231min-1

k = 2.303 log [ A0 ]
t
[A]
t = 2.303
0.0231

log 100
10

t = 2.303 min
0.0231
t = 99.7min
1

25

(a) (i) Due to decrease in bond dissociation enthalpy from HF to HI , there is an increase in acidic
character observed.
(ii)Oxygen exists as diatomic O2 molecule while sulphur as polyatomic S8
(iii)Due to non availability of d orbitals

1
1

(b)

25
OR
(i) White Phosphorus because it is less stable due to angular strain
(ii)Nitrogen oxides emitted by supersonic jet planes are responsible for depletion of ozone layer.
Or
NO+O3 NO2+ O2
(iii)due to small size of F, large inter electronic repulsion / electron- electron repulsion among the
lone pairs of fluorine

,
1
1

(iv)Helium
(v) XeF2 + PF5 [XeF]+ [PF6]-

1
1
1x5=

26.

A=

B=

C=

D=

E=

OR
O 26.
a. i)
1
1
1
ii)

iii)

b. ( CH3)3N < C2H5NH2 < C2H5OH


c. By Hinsberg test secondary amines ( CH3)2NH shows ppt formation which is insoluble
KOH
while
in tertiary amines ( CH3)3N do not react with benzene sulphonyl choride

1
1

SET-2

H$moS> Z.

Series SSO/2

Code No.

amob Z.

56/2/2/F

narjmWu H$moS >H$mo Cma-nwpVH$m Ho$ _wI-n


>na Ad` {bIo &

Roll No.

Candidates must write the Code on the


title page of the answer-book.

H$n`m OmM H$a b| {H$ Bg Z-n _o _w{V n> 15 h &


Z-n _| Xm{hZo hmW H$s Amoa {XE JE H$moS >Z~a H$mo N>m Cma-nwpVH$m Ho$ _wI-n> na
{bI| &
H$n`m OmM H$a b| {H$ Bg Z-n _| >26 Z h &
H$n`m Z H$m Cma {bIZm ew$ H$aZo go nhbo, Z H$m H$_mH$ Ad` {bI| &
Bg Z-n H$mo nT>Zo Ho$ {bE 15 {_ZQ >H$m g_` {X`m J`m h & Z-n H$m {dVaU nydm
_| 10.15 ~Oo {H$`m OmEJm & 10.15 ~Oo go 10.30 ~Oo VH$ N>m Ho$db Z-n H$mo nT>|Jo
Ama Bg Ad{Y Ho$ XmamZ do Cma-nwpVH$m na H$moB Cma Zht {bI|Jo &
Please check that this question paper contains 15 printed pages.
Code number given on the right hand side of the question paper should be
written on the title page of the answer-book by the candidate.
Please check that this question paper contains 26 questions.
Please write down the Serial Number of the question before
attempting it.
15 minute time has been allotted to read this question paper. The question
paper will be distributed at 10.15 a.m. From 10.15 a.m. to 10.30 a.m., the
students will read the question paper only and will not write any answer on
the answer-book during this period.

agm`Z {dkmZ (gmpVH$)


CHEMISTRY (Theory)

{ZYm[aV g_` : 3 KQ>o

A{YH$V_ AH$ : 70

Time allowed : 3 hours


56/2/2/F

Maximum Marks : 70
1

P.T.O.

gm_m` {ZX}e :
(i)

g^r Z A{Zdm` h &

(ii)

Z g`m 1 go 5 VH$ A{V bKw-Cmar` Z h Ama `oH$ Z Ho$ {bE 1 AH$ h &

(iii)

Z g`m 6 go 10 VH$ bKw-Cmar` Z h Ama `oH$ Z Ho$ {bE 2 AH$ h &

(iv)

Z g`m 11 go 22 VH$ ^r bKw-Cmar` Z h Ama `oH$ Z Ho$ {bE 3 AH$ h &

(v)

Z g`m 23 _y`mYm[aV Z h Ama BgHo$ {bE 4 AH$ h &

(vi)

Z g`m 24 go 26 VH$ XrK-Cmar` Z h Ama `oH$ Z Ho$ {bE 5 AH$ h &

(vii)

`{X Amd`H$Vm hmo, Vmo bmJ Q>o~bm| H$m `moJ H$a| & H$Hw$boQ>am| Ho$ Cn`moJ H$s AZw_{V
Zht h &

General Instructions :
(i)

All questions are compulsory.

(ii)

Questions number 1 to 5 are very short answer questions and carry


1 mark each.

(iii)

Questions number 6 to 10 are short answer questions and carry 2 marks


each.

(iv)

Questions number 11 to 22 are also short answer questions and carry


3 marks each.

(v)

Question number 23 is a value based question and carry 4 marks.

(vi)

Questions number 24 to 26 are long answer questions and carry 5 marks


each.

(vii)

Use log tables, if necessary. Use of calculators is not allowed.

1.

{XE JE `m{JH$ H$m AmB.`y.nr.E.gr. Zm_ {b{IE :


HO CH2 CH = C CH3
CH3

56/2/2/F

Write the IUPAC name of the given compound :


HO CH2 CH = C CH3
CH3
2.

^m{VH$emofU CH$_Ur` h O~{H$ amgm`{ZH$emofU AZwH$_Ur` hmoVm h & `m| ?

Physisorption is reversible while chemisorption is irreversible. Why ?


3.

{ZZ{b{IV `w_ _| H$mZ

A{^{H$`m A{YH$ VrdVm go H$aoJm Ama `m| ?


Ama CH3 CH2 I

SN2

CH3 CH2 Br

Which would undergo SN2 reaction faster in the following pair and why ?
CH3 CH2 Br and CH3 CH2 I
4.

gm_m` Vmn_mZ na g\$a H$m H$mZ-gm Ana$n (EbmoQ>mon) D$_r` $n go Wm`r h ?

Which allotrope of sulphur is thermally stable at room temperature ?


5.

Cg `m{JH$ H$m gy `m h {Og_| Vd Y


AQ>\$bH$s` [a>{$ H$m 2/3dm ^mJ KoaVo h ?

ccp

OmbH$ ~ZmVm h Ama

Ho$ na_mUw
1

What is the formula of a compound in which the element Y forms ccp


lattice and atoms of X occupy 2/3rd of octahedral voids ?
6.

{ZZ{b{IV A{^{H$`mAm| _| `moJ AmZo dmbo A{^H$maH$m| Ho$ Zm_ Xr{OE :


(i)

?
CH3 CHO CH3 CH CH3

|
OH
(ii)

?
CH3 COOH CH3 COCl

Name the reagents used in the following reactions :


(i)

(ii)
56/2/2/F

?
CH3 CHO CH3 CH CH3
|
OH
?
CH3 COOH CH3 COCl

P.T.O.

7.

(a)

Obr` H$mna(II) bmoamBS> {db`Z Ho$ {dwV-AnKQ>Z Ho$ XmamZ H$WmoS> na


{ZZ{b{IV A{^{H$`mE hmoVr h :
Cu2+ (aq) + 2e
Cu(s)
H+ (aq) + e

(b)

(a)

1
H (g)
2 2

E0 = + 034 V
E0 = 000 V

CZHo$ _mZH$ AnM`Z BboQ>moS> {d^d (E0) Ho$ _mZm| Ho$ AmYma na H$WmoS> na {H$g
A{^{H$`m H$s g^mdZm (gwgJVVm) h Ama `m| ?
Am`Zm| Ho$ dV A{^J_Z Ho$ H$mobamD$e {Z`_ H$m H$WZ H$s{OE & BgH$m EH$
AZw`moJ {b{IE &

Following reactions occur at cathode during the electrolysis of


aqueous copper(II) chloride solution :
Cu2+ (aq) + 2e
Cu(s)
H+ (aq) + e

1
H (g)
2 2

E0 = + 034 V
E0 = 000 V

On the basis of their standard reduction electrode potential (E0)


values, which reaction is feasible at the cathode and why ?
(b)

State Kohlrausch law of independent migration of ions. Write its


one application.

8.

amCQ> Ho$ {Z`_ go G$Um_H$ {dMbZ go `m Vmn` h ? EH$ CXmhaU Xr{OE & G$Um_H$
{dMbZ Ho$ {bE mixH H$m `m {M hmoVm h ?

2>

AWdm
E{OAmoQ>mon H$mo n[a^m{fV H$s{OE & amCQ> Ho$ {Z`_ go G$Um_H$ {dMbZ mam ~ZZo dmbm
E{OAmoQ>mon {H$g H$ma H$m hmoVm h ? EH$ CXmhaU Xr{OE &

56/2/2/F

What is meant by negative deviation from Raoults law ? Give an


example. What is the sign of mixH for negative deviation ?
OR
Define azeotropes. What type of azeotrope is formed by negative
deviation from Raoults law ? Give an example.
9.

gH$_U Vd `m| n[adVu CnM`Z AdWmAmo H$mo X{eV H$aVo h


H$mZ-gm Vd gdm{YH$ CnM`Z AdWmE XemVm h Ama `m| ?

? 3d loUr

_|

(Sc go Zn)
2

Why do transition elements show variable oxidation states ? In 3d series


(Sc to Zn), which element shows the maximum number of oxidation
states and why ?
10.

(i)

{ZZ{b{IV H$mbog H$m AmB.`y.nr.E.gr. Zm_ {b{IE

[Cr (en)3]Cl3
(ii)

(i)

{ZZ{b{IV H$mbog H$m gy {b{IE :


nmoQ>{e`_ Q>mB AmgbQ>mo H$mo_oQ>(III)

Write down the IUPAC name of the following complex :


[Cr (en)3]Cl3

(ii)

Write the formula for the following complex :


Potassium tri oxalato chromate(III)

11.

25C

na {ZZ gob H$m {dwV-dmhH$ ~b (B.E_.E\$.) n[aH${bV H$s{OE

Zn | Zn2+ (0.001 M) | | H+ (0.01 M) | H2(g) (1 bar) | Pt(s)

E0

(Zn 2 / Zn )

= 0.76 V,

E0

(H / H )
2

= 0.00 V

Calculate the emf of the following cell at 25C :


Zn | Zn2+ (0.001 M) | | H+ (0.01 M) | H2(g) (1 bar) | Pt(s)

E0

(Zn 2 / Zn )

56/2/2/F

= 0.76 V,

E0

(H / H )
2

= 0.00 V
P.T.O.

12.

{ZZ{b{IV AdbmoH$Zm| Ho$ {bE H$maUm| H$mo Xr{OE :


(i)
g_wr Ob Ama ZXr H$m Ob Ohm {_bVo h dhm EH$ S>oQ>m ~Z OmVm h &
(ii)
MmaH$mob H$s gVh na N2 Jg H$s Anojm NH3 Jg A{YH$ erKVm go A{Yemo{fV
hmoVr h &
(iii) MyU {H$E hE nXmW A{YH$ ^mdembr A{YemofH$ hmoVo h &

Give reasons for the following observations :

13.

(i)

A delta is formed at the meeting point of sea water and river


water.

(ii)

NH3 gas adsorbs more readily than N2 gas on the surface of


charcoal.

(iii)

Powdered substances are more effective adsorbents.

(i)

H$mbog [Pt(en)2Cl2]2+ Ho$ `m{_Vr` g_md`dm| H$mo Amao{IV H$s{OE &


{H$Q>b \$sS> {gmV Ho$ AmYma na `{X o > P h, Vmo d4 Am`Z H$m BboQ>m{ZH$
{d`mg {b{IE &
H$mbog [Ni(CN)4]2 H$m gH$aU H$ma Ama Mw~H$s` `dhma {b{IE &
(Ni H$m na_mUw H$_mH$ = 28)

(ii)
(iii)

14.

(i)

Draw the geometrical isomers of complex [Pt(en)2Cl2]2+.

(ii)

On the basis of crystal field theory, write the electronic


configuration for d4 ion, if o P.

(iii)

Write the hybridization type and magnetic behaviour of the


complex [Ni(CN)4]2. (Atomic number of Ni = 28)

(a)

{ZZ{b{IV H$mo Amn H$maU XoVo hE H$go g_PmEJo :


(i)
Mn H$m CV_ bwAmoamBS> MnF4 h O~{H$ CV_ AmgmBS>
h &
(ii) gH$_U YmVwE Ama CZHo$ `m{JH$ CoaH$ JwUY_ XemVo h &

(b)

{ZZ{b{IV g_rH$aU H$mo nyU H$s{OE


2

3 MnO 4
56/2/2/F

+ 4H+

Mn2O7

:
3

(a)

(b)

How would you account for the following :


(i)

Highest fluoride of Mn is MnF4 whereas the highest oxide is


Mn2O7.

(ii)

Transition metals and their compounds show catalytic


properties.

Complete the following equation :


2

3 MnO 4

15.

+ 4H+

{ZZ{b{IV A{^{H$`mAm| Ho$ CnmXm| H$s mJw{$ H$s{OE :


(i)

i) B H
CH3 CH = CH2 2 6 ?
ii) 3 H 2O 2 / OH

(ii)

Br2 (aq)
C6H5 OH
?

(iii)

Cu / 573 K
CH3CH2OH ?

Predict the products of the following reactions :

16.

(i)

i) B H
CH3 CH = CH2 2 6 ?
ii) 3 H 2O 2 / OH

(ii)

Br2 (aq)
C6H5 OH
?

(iii)

Cu / 573 K
CH3CH2OH ?

EH$ Vd X (_moba `_mZ = 60 g mol1) H$m KZd 623 g cm3 h & `{X `y{ZQ> gob
Ho$ H$moa H$s b~mB 4 108 cm h, Vmo `y{~H$ `y{ZQ> gob Ho$ H$ma H$s `m nhMmZ
hmoJr ?

An element X (molar mass = 60 g mol1) has a density of 6.23 g cm3.


Identify the type of cubic unit cell, if the edge length of the unit cell is
4 108 cm.
56/2/2/F

P.T.O.

17.

Ob _| NaCl (_mob `_mZ = 585 g mol1) H$s {H$VZr _mm KwbmB OmE {H$
{h_mH$ 2C KQ> OmE, `h _mZVo hE {H$ NaCl nyU $n go {dK{Q>V hmoVm h ?
(Kf Ob Ho$ {bE = 186 K kg mol1)
372 g

Calculate the mass of NaCl (molar mass = 585 g mol1) to be dissolved in


372 g of water to lower the freezing point by 2C, assuming that NaCl
undergoes complete dissociation. (Kf for water = 186 K kg mol1)

18.

{ZZ{b{IV ~hbH$m| Ho$ EH$bH$m| Ho$ Zm_ Ama CZH$s gaMZmE {b{IE :
(i)
Q>oarbrZ
(ii)
~Ho$bmBQ>
(iii) ~wZm-S

Write the names and structures of the monomers of the following


polymers :

19.

(i)

Terylene

(ii)

Bakelite

(iii)

Buna-S

(a)

EpH$b hbmBS>| Ob _| KwbZerb Zht h & `m| ?


`wQ>Z-1-Amb H$m{eH$s` {ZpH$` (YwdU AKyUH$) h naVw `wQ>Z-2-Amb
H$m{eH$s` g{H$` (YwdU KyUH$) h & `m| ?
`{n bmoarZ BboQ>mZ H$mo AmH${fV H$aZo dmbm Jwn h {\$a ^r `h BboQ>mZZohr
Eamo_{Q>H$ {VWmnZ A{^{H$`mAm| _| AmWm}- VWm nam- {ZX}eH$ h & `m| ?

(b)
(c)
(a)

Why are alkyl halides insoluble in water ?

(b)

Why is Butan-1-ol optically inactive but Butan-2-ol is optically


active ?

(c)

Although chlorine is an electron withdrawing group, yet it is


ortho-, para- directing in electrophilic aromatic substitution
reactions. Why ?

56/2/2/F

20.

{ZZ{b{IV H$m $nmVaU Amn H$go H$a|Jo :


(i)
~oOmoBH$ E{gS> H$mo ~oOpS>hmBS> _|
(ii)
EWmBZ H$mo EWZb _|
(iii) Eogr{Q>H$ E{gS> H$mo _rWoZ _|
AWdm

{ZZ{b{IV A{^{H$`mAm| go g~pYV g_rH$aUm| H$mo {b{IE :


(i)
Q>r\$Z A{^{H$`m
(ii)
dmo\$-{H$Za AnM`Z
(iii) EQ>mS> A{^{H$`m
How do you convert the following :
(i)

Benzoic acid to Benzaldehyde

(ii)

Ethyne to Ethanal

(iii)

Acetic acid to Methane


OR

Write the equations involved in the following reactions :

21.

(i)

Stephen reaction

(ii)

Wolff-Kishner reduction

(iii)

Etard reaction

(i)

{ZZ{b{IV _o H$mZ _moZmogHo$amBS> h :


Q>mM, _mQ>mog, \$Q>mog, gobwbmog
Abr` Eo{_Zmo Eo{gS>m| Ama jmar` Eo{_Zmo Eo{gS>m| Ho$ ~rM$ `m AVa hmoVm h ?
Cg {dQ>m{_Z H$m Zm_ {b{IE {OgH$s H$_r Ho$ H$maU _gyS>m| _| IyZ AmZo bJVm h &

(ii)
(iii)
(i)

Which one of the following is a monosaccharide :


starch, maltose, fructose, cellulose

(ii)

What is the difference between acidic amino acids and basic amino
acids ?

(iii)

Write the name of the vitamin whose deficiency causes bleeding of


gums.

56/2/2/F

P.T.O.

22.

23.

(i)
(ii)

{ZH$b Ho$ n[aH$aU _| H$m_ AmZo dmbr {d{Y Ho$ nrN>o Omo {gmV hmoVm h CgH$m
CoI H$s{OE &
gmoZo Ho$ {ZH$fU _| VZw NaCN H$s `m ^y{_H$m hmoVr h ?

(iii)

H$mna _Q>o `m hmoVm h

(i)

Indicate the principle behind the method used for the refining of
Nickel.

(ii)

What is the role of dilute NaCN in the extraction of gold ?

(iii)

What is copper matte ?

~m| _| _Yw_oh Ama CXmgr Ho$ ~T>Vo Ho$gm| H$mo XoIZo Ho$ ~mX EH$ {g Hy$b Ho$ q{gnb
lr MmonS>m Zo EH$ go{_Zma H$m Am`moOZ {H$`m {Og_| Chm|Zo ~m| Ho$ A{^^mdH$m| VWm A`
Hy$bm| Ho$ q{gnbm| H$mo Am_{V {H$`m & Chm|Zo Hy$bm| _| gS>o hE ^mo` nXmWm] (OH$ \y$S>)
na {V~Y bJmZo H$m {ZU` {b`m, gmW hr `h {ZU` {b`m {H$ Hy$bm| _| dm`dYH$
nXmW Ogo gyn, bgr, XY Am{X H$Q>rZm| _| CnbY H$amB OmE & Chm|Zo `h ^r {ZU`
{b`m {H$ mV:H$mbrZ Ego~br Ho$ g_` ~m| H$mo {V{XZ AmYo KQ>o H$s emar[aH$ H$gaV
^r H$amB OmE & N>: _mh nMmV lr MmonS>m Zo ~m| Ho$ dm` H$m A{YH$V_ {dmb`m| _|
nwZ: {ZarjU H$adm`m Ama ~m| Ho$ dm` _| AZwn_ gwYma nm`m J`m &
Cn`w$ H$aU H$mo nT>Zo Ho$ ~mX, {ZZ{b{IV Zm| Ho$ Cma Xr{OE :
(i)
lr MmonS>m mam {H$Z _y`m| (H$_-go-H$_ Xmo) H$mo Xem`m J`m h ?
(ii)
EH$ {dmWu Ho$ $n _|, Amn Bg {df` _| H$go OmJ$H$Vm \$bmEJo ?
(iii) AdZ_Z-{damoYr S>J {~Zm S>mQ>a H$s gbmh Ho$ `m| Zht boZo Mm{hE ?
(iv) H${_ _YwaH$ Ho$ Xmo CXmhaU Xr{OE &

Seeing the growing cases of diabetes and depression among children,


Mr. Chopra, the principal of one reputed school organized a seminar in
which he invited parents and principals. They all resolved this issue by
strictly banning the junk food in schools and by introducing healthy
snacks and drinks like soup, lassi, milk etc. in school canteens. They also
decided to make compulsory half an hour physical activities for the
students

in

the

morning

assembly

daily.

After

six

months,

Mr. Chopra conducted the health survey in most of the schools and
discovered a tremendous improvement in the health of students.
56/2/2/F

10

After reading the above passage, answer the following questions :


(i)

What are the values (at least two) displayed by Mr. Chopra ?

(ii)

As a student, how can you spread awareness about this issue ?

(iii)

Why should antidepressant drugs not be taken without consulting


a doctor ?

24.

(iv)

Give two examples of artificial sweeteners.

(a)

{ZZ{b{IV Ho$ H$maU XoVo hE nQ> H$s{OE :

(b)

(i)

NH4

_| Omo Am~Y H$moU h dh NH3 Ho$ H$moU go CVa h &


(ii) H2O H$s Anojm H2S H$m dWZmH$ `yZVa h &
(iii) AnM`Z `dhma SO2 go TeO2 H$s Amoa KQ>Vm h &
{ZZ{b{IV H$s gaMZmE Amao{IV H$s{OE :
(i)
H4P2O7 (nm`amo\$m\$mo[aH$ Eo{gS>)
(ii)

XeF2

AWdm
(a)

(b)

(a)

56/2/2/F

{ZZ{b{IV H$s gaMZmE Amao{IV H$s{OE :


(i)

XeF4

(ii)

H2S2O7

{ZZ{b{IV Ho$ H$maU Xr{OE :


(i)
HCl go A{^{H$`m go Am`aZ FeCl2 ~ZmVm h Z H$s FeCl3.
(ii) HClO H$s Anojm HClO4 ~bVa Ab h &
(iii) dJ 15 Ho$ g^r hmBS>mBS>m| _| BiH3 ~bV_ AnMm`H$ h &

Account for the following :

(i)

Bond angle in NH4 is higher than NH3.

(ii)

H2S has lower boiling point than H2O.

(iii)

Reducing character decreases from SO2 to TeO2.


11

P.T.O.

(b)

Draw the structures of the following :


(i)

H4P2O7 (Pyrophosphoric acid)

(ii)

XeF2
OR

(a)

(b)

25.

Draw the structures of the following :


(i)

XeF4

(ii)

H2S2O7

Account for the following :


(i)

Iron on reaction with HCl forms FeCl2 and not FeCl3.

(ii)

HClO4 is a stronger acid than HClO.

(iii)

BiH3 is the strongest reducing agent amongst all the


hydrides of group 15.

(a)

`oH$ Ho$ {bE Cn`w$ CXmhaU XoVo hE {ZZ{b{IV A{^{H$`mAm| H$mo X{eV
H$s{OE :
(i)
A_moZrH$aU
(ii) H$pbJ (`w_Z) A{^{H$`m
(iii) Eo_rZm| H$m Eogr{Q>brH$aU

(b)

mW{_H$ (mB_ar), {Vr`H$ (goH$S>ar) Ama VVr`H$ (Q>{e`ar) E_rZm| H$s nhMmZ
H$aZo Ho$ {bE {hg~J {d{Y H$m dUZ H$s{OE & g~ A{^{H$`mAm| Ho$ amgm`{ZH$
g_rH$aUm| H$mo ^r {b{IE &
AWdm

(a)

56/2/2/F

O~ ~oOrZ S>mBEOmo{Z`_ bmoamBS> (C6 H5 N2Cl ) {ZZ{b{IV A{^H$maH$m| go


A{^{H$`m H$aVm h, V~ mV _w` CnmXm| H$s gaMZmE {b{IE :
(i)

HBF4 /

(ii)

Cu / HBr
12

(b)

{ZZ{b{IV A{^{H$`mAm| _|

A, B Ama C H$s

gaMZmE {b{IE :

5
(a)

(b)

Illustrate the following reactions giving suitable example in each


case :
(i)

Ammonolysis

(ii)

Coupling reaction

(iii)

Acetylation of amines

Describe Hinsberg method for the identification of primary,


secondary and tertiary amines. Also write the chemical equations
of the reactions involved.
OR

(a)

Write the structures of main products when benzene diazonium


chloride (C6 H5 N 2 Cl ) reacts with the following reagents :

(b)

56/2/2/F

(i)

HBF4 /

(ii)

Cu / HBr

Write the structures of A, B and C in the following reactions :

13

P.T.O.

26.

(a)

{ZZ{b{IV nXm| H$mo n[a^m{fV H$s{OE :


(i)
g{H$`U D$Om
(ii) Xa pWamH$

(b)

25%

{d`moOZ Ho$ {bE EH$ W_ H$mo{Q> H$s A{^{H$`m


A{^{H$`m Ho$ {bE t1/2 H$m n[aH$bZ H$s{OE >&
({X`m J`m

(a)

10

{_ZQ> boVr h &


5

: log 2 = 03010, log 3 = 04771, log 4 = 06021)

AWdm
EH$ amgm`{ZH$ A{^{H$`m R P Ho$ {bE gmU _| n[adVZ
ZrMo bmQ> _| {X`m J`m h :

ln [R] vs. g_` (s)

ln [R]
t (s)
(i)
(ii)
(iii)
(b)

(a)

(b)

A{^{H$`m H$s H$mo{Q> H$s mJw{$ H$s{OE &


dH$ H$m T>bmZ `m h ?
A{^{H$`m Ho$ {bE Xa pWamH$ H$s `y{ZQ> {b{IE &

XemBE {H$ 99% nyU hmoZo _| Omo g_` bJVm h dh Cg g_` H$m XwJwZm h Omo
A{^{H$`m Ho$ 90% nyU hmoZo _| bJVm h &
Define the following terms :
(i)

Activation energy

(ii)

Rate constant

A first order reaction takes 10 minutes for 25% decomposition.


Calculate t1/2 for the reaction.
(Given : log 2 = 03010, log 3 = 04771, log 4 = 06021)
OR

56/2/2/F

14

(a)

For a chemical reaction R P, the variation in the concentration,


ln [R] vs. time (s) plot is given as

ln [R]
t (s)

(b)

56/2/2/F

(i)

Predict the order of the reaction.

(ii)

What is the slope of the curve ?

(iii)

Write the unit of rate constant for this reaction.

Show that the time required for 99% completion is double of the
time required for the completion of 90% reaction.

15

P.T.O.

CHEMISTRY MARKING SCHEME 2015


SET -56/2/2 F
Qn

Marks

Value points

3-Methylbut-2-en-1-ol

Because of weak van der Waals forces in physisorption whereas there are strong chemical
forces in chemisorption.
CH3CH2I , because I is a better leaving group.

Rhombic sulphur

X2Y3

(i) CH3MgBr/ H3O+

(ii) PCl5/ PCl3 / SOCl2

a) Cu2+ (aq) + 2 e

Cu(s) because of high E0 value/ more negative G

b) It states that limiting molar conductivity of an electrolyte is equal to the sum of the individual 1
contributions of cations and anions of the electrolyte.
It is used to calculate the m0 for weak electrolyte / It is used to calculate and Kc
(Any one application) 1
8

When solute- solvent interaction is stronger than pure solvent or solute interaction.

Eg: chloroform and acetone (or any other correct eg)

mixH= negative

OR

Azeotropes binary mixtures having same composition in liquid and vapour phase and boil at 1
constant temperature / is a liquid mixture which distills at constant temperature without
undergoing change in composition

Maximum boiling azeotropes

eg: HNO3 (68%) and H2O(32%) (or any other correct example)
9

a) Due to presence of unpaired d-electrons/ comparable energies of 3d and 4s orbitals.


b) Mn , due to involvement of 4s and 3d electrons/ presence of maximum unpaired delectrons.

1
,

10

11

i) tris-(ethane-1,2-diamine)chromium(III) chloride

ii) K3[ Cr(C2O4)3]

E cell = E0 cell

log

E cell = 0.76 V -

V log

E cell = 0.76 0.0295 V log 10


1

= 0.7305 V
12

i)

Due to coagulation of colloidal clay particles.

ii)

Because NH3 is easily liquefiable than N2 due to its larger molecular size.

iii)

Because of more surface area.

13

i)
cis- isomer

14

15

trans-isomer

ii)

t2g4

iii)

dsp 2 , diamagnetic

i)

Because oxygen stabilizes Mn more than F due to multiple bonding

ii)

Because of their ability to show variable oxidation state(or any other correct reason)

iii)

3MnO42- + 4H+

i)

CH3CH2CH2OH

2MnO4- + MnO2 + 2H2O

1
ii)
iii)

CH3CHO

16

d=
6.23 g cm-3 =

17

z=4

fcc

Tf = i. Kf m
= i Kf wB x 1000

MB x wA
2K= 2 x 1.86K kg/mol x wB x 1000

58.5 g/mol x 37.2 g


wB = 1.17g

18

1
i)

1
ii)
Phenol

and

formaldehyde

1
iii)
(Note: half mark for structure/s and half mark for name/s)
19

20

a) Because they are unable to form H-bonds with water molecules.


b) Because of the presence of chiral carbon in butan-2-ol.
c) Due to dominating +R effect
i)
C6H5COOH PCl5 C6H5COCl H2/Pd
C6H5CHO

1
1
1
1

BaSO4
ii)

CHCH + H2O

iii)

CH3COOH

NaOH

2+

Hg /H2SO4
CH3COONa

CH3CHO
NaOH + CaO , heat

OR

1
CH4

1
20

i)
1
ii)

iii)

21

i)
ii)
iii)

22

Fructose
1
Acidic amino acid has more number of acidic carboxylic group than basic amino
1
group whereas basic amino acid has more number of basic amino group.
Vitamin C
1

a) Impure Ni reacts with CO to form volatile Ni(CO)4 which when heated at higher 1
temperature decomposes to give pure Ni.
b) NaCN acts as a leaching agent to form a soluble complex with gold.

c) It is a mixture of Cu2S and FeS

23

a) Concern for students health, Application of knowledge of chemistry to daily life, empathy ,
, caring or any other
b) Through posters, nukkad natak in community, social media, play in assembly (or any other
1
relevant answer)
1
c) Wrong choice and overdose may be harmful
+
d) Aspartame, saccharin (or any other correct example)

24

a) i)Because of lone pair in NH3 , lone pair- bond pair repulsion decreases the bond angle

ii)Because of absence of H-bonding in H2S

iii)Because stability of +4 oxidation state increases from SO2 to TeO2

1,1

b)
4

OR

24

a)

1,1

b)i)Because iron on reaction with HCl produces H2(g) which prevents the formation of FeCl2 to
FeCl3 / Because HCl is a weak oxidising agent.
ii) Because of higher oxidation state of chlorine in HClO4
iii) Because of lower dissociation enthalpy of Bi-H bond.

1
1
25

a) i)ammonolysis

ii)

(any one)

iii)

(or any other correct reaction)

b)reaction of primary amine


1

(soluble in alkali)
Reaction of secondary amine
1

(insoluble in alkali)
Tertiary amine doesnt react

OR
1
25
1

a) i)

ii)

,,

,,

b) i) Aii) A- CH3CN
26

B-

C-

B- CH3CH2NH2

C- CH3CH2OH

a)i) Activation energy- Extra energy required by reactants to form activated complex.
ii) Rate constant- rate of reaction when the concentration of reactant is unity.

1
1

b)

k= 2.303 log [ A0 ]
t
[A]
k = 2.303 log 100
10 min
75

k = 2.303 x 0.125
10 min
k = 0.02879 min-1
t1/2 =

t1/2

24.07min
1

OR
a) i)First order ii) -k
26

iii) s-1
1,1,1

b)
t=

log

t99% =
t=

log

x 2

t90% =

log

=
t99% = 2 x t90%

SET 2
.

Series : SSO/1

Code No.

56/1/2/D

- -

Roll No.

Candidates must write the Code on


the title page of the answer-book.

- 12
- - -
- 26
,
- 15 - 10.15
10.15 10.30 -
Please check that this question paper contains 12 printed pages.
Code number given on the right hand side of the question paper should be written on the
title page of the answer-book by the candidate.
Please check that this question paper contains 26 questions.
Please write down the Serial Number of the question before attempting it.
15 minutes time has been allotted to read this question paper. The question paper will be
distributed at 10.15 a.m. From 10.15 a.m. to 10.30 a.m., the students will only read the
question paper and will not write any answer on the answer-book during this period.

()
CHEMISTRY (Theory)
: 3 ]

[ : 70

Time allowed : 3 hours ]

[ Maximum Marks : 70

:
(i)

(ii)

- 1 5 - 1
- 6 10 - 2
- 11 22 - 3
- 23 4
- 24 26 - 5

(iii)
(iv)
(v)
(vi)
(vii)
56/1/2/D

[P.T.O.

General Instructions :
(i)

All questions are compulsory.

(ii)

Q. no. 1 to 5 are very short answer questions and carry 1 mark each.

(iii) Q. no. 6 to 10 are short answer questions and carry 2 marks each.
(iv)

Q. no. 11 to 22 are also short answer questions and carry 3 marks each.

(v)

Q. no. 23 is a value based question and carry 4 marks.

(vi)

Q. no. 24 to 26 are long answer questions and carry 5 marks each.

(vii) Use log tables if necessary, use of calculators is not allowed.

1.

SN2 ?
CH3 CH2 Br

CH3
|
CH3 C CH3
|
Br

Which would undergo SN2 reaction faster in the following pair and why ?
CH3
|
CH3 CH2 Br and CH3 C CH3
|
Br

2.

KCl
?

BaCl2

Out of BaCl2 and KCl, which one is more effective in causing coagulation of a
negatively charged colloidal Sol ? Give reason.

3.

Y ccp X 1/3
?
What is the formula of a compound in which the element Y forms ccp lattice and
atoms of X occupy 1/3rd of tetrahedral voids ?

56/1/2/D

4.

H3PO4

What is the basicity of H3PO4 ?

5.

..... :

Write the IUPAC name of the given compound :

6.

Name the reagents used in the following reactions :


(i)

(ii)

7.

?
CH3 CO CH3 CH3 C H CH3
|
OH
?
C6H5 CH2 CH3 C6H5 COOK+

?
mixH ?

What is meant by positive deviations from Raoults law ? Give an example. What is
the sign of mixH for positive deviation ?

/OR
7.

Define azeotropes. What type of azeotrope is formed by positive deviation from


Raoults law ? Give an example.
56/1/2/D

[P.T.O.

8.

(a)

:
Ag+(aq) + e Ag(s)

E = +0.80 V

1
H+(aq) + e H2(g)
2

E = 0.00 V


?
(b)

(a)

(E)

Following reactions occur at cathode during the electrolysis of aqueous silver


chloride solution :
Ag+(aq) + e Ag(s)

E = +0.80 V

1
H+(aq) + e H2(g)
2

E = 0.00 V

On the basis of their standard reduction electrode potential (E) values, which
reaction is feasible at the cathode and why ?
(b)

9.

Define limiting molar conductivity. Why conductivity of an electrolyte solution


decreases with the decrease in concentration ?

What are the transition elements ? Write two characteristics of the transition elements.

10.

(i)

(ii)

[Cr(NH3)2Cl2(en)]Cl (en = ethylenediamine)

-o- (III).
(i)

Write down the IUPAC name of the following complex :


[Cr(NH3)2Cl2(en)]Cl (en = ethylenediamine)

(ii)

Write the formula for the following complex :


Pentaamminenitrito-o-Cobalt (III).

56/1/2/D

.....

11.

(iii)

[Pt(NH3)2Cl2]
o < P d4
[Ni(CO)4] (.. Ni = 28)

(i)

Draw the geometrical isomers of complex [Pt(NH3)2Cl2].

(ii)

On the basis of crystal field theory, write the electronic configuration for d4 ion
if o < P.

(i)
(ii)

(iii) Write the hybridization and magnetic behaviour of the complex [Ni(CO)4].
(At.no. of Ni = 28)
12.

25 C emf :
Fe | Fe2+(0.001 M) || H+(0.01 M) | H2(g) (1 bar) | Pt(s)
E(Fe2+ | Fe) = 0.44 V E(H+ | H2) = 0.00 V

Calculate emf of the following cell at 25 C :


Fe | Fe2+(0.001 M) || H+(0.01 M) | H2(g) (1 bar) | Pt(s)
E(Fe2+ | Fe) = 0.44 V E(H+ | H2) = 0.00 V
13.

:
(i)
(ii)
(iii) CO

Give reasons for the following observations :


(i) Leather gets hardened after tanning.
(ii) Lyophilic sol is more stable than lyophobic sol.
(iii) It is necessary to remove CO when ammonia is prepared by Habers process.
14.


(i) -6, 6

(ii) PHBV
(iii)

Write the names and structures of the monomers of the following polymers :
(i) Nylon-6, 6
(ii) PHBV
(iii) Neoprene
56/1/2/D

[P.T.O.

15.

:
(i)

(ii)

CH3 C = O (i) H2N NH3


?
|
(ii) KOH/Glycol,
CH3
NaOH/I2
C6H5 CO CH3
?+?

NaOH / CaO
(iii) CH3 COONa ?

Predict the products of the following reactions :


(i)

(ii)

CH3 C = O (i) H2N NH3


?
|
(ii) KOH/Glycol,
CH3
NaOH/I2
C6H5 CO CH3 ? + ?

NaOH / CaO
(iii) CH3 COONa ?

16.

?
(i)

(ii)

-2- 2--2-

(iii)

How do you convert the following :


(i)

Phenol to anisole

(ii)

Propan-2-ol to 2-methylpropan-2-ol

(iii) Aniline to phenol

/OR
16.

(a)

- :
H+
2CH3CH2OH CH3CH2 O CH2CH3

(b)

(a)

Write the mechanism of the following reaction :


H+
2CH3CH2OH CH3CH2 O CH2CH3

(b)
56/1/2/D

Write the equation involved in the acetylation of Salicylic acid.


6

17.

(i)
(ii)
(iii)

: , , , ?
?
?

(i)

Which one of the following is a disaccharide : Starch, Maltose, Fructose,


Glucose ?

(ii)

What is the difference between fibrous protein and globular protein ?

(iii) Write the name of vitamin whose deficiency causes bone deformities in children.
18.

:
(a) t- n-
(b)
(c) o/p
(NO2)

Give reasons :
(a)
(b)
(c)

19.

n-Butyl bromide has higher boiling point than t-butyl bromide.


Racemic mixture is optically inactive.
The presence of nitro group (NO2) at o/p positions increases the reactivity of
haloarenes towards nucleophilic substitution reactions.

49 g 3.9 g 1.62 K
( )
( : = 122 g mol1, Kf = 4.9 K kg mol1)

3.9 g of benzoic acid dissolved in 49 g of benzene shows a depression in freezing point


of 1.62 K. Calculate the vant Hoff factor and predict the nature of solute (associated
or dissociated).
(Given : Molar mass of benzoic acid = 122 g mol1, Kf for benzene = 4.9 K kg mol1)
20.

(iii)


?
?

(i)

Indicate the principle behind the method used for the refining of zinc.

(ii)

What is the role of silica in the extraction of copper ?

(i)
(ii)

(iii) Which form of the iron is the purest form of commercial iron ?
56/1/2/D

[P.T.O.

21.

27 g mol1 4.05 108 cm


2.7 g cm3 , ?

An element with molar mass 27 g mol1 forms a cubic unit cell with edge length
4.05 108 cm. If its density is 2.7 g cm3, what is the nature of the cubic unit cell ?

22.

(a)

(b)

?
(i)

(ii)

2MnO4 + 6H+ + 5NO2


(a)

(b)

How would you account for the following :


(i)

Actinoid contraction is greater than lanthanoid contraction.

(ii)

Transition metals form coloured compounds.

Complete the following equation :

2MnO4 + 6H+ + 5NO2

23.



-
, ,
:
: :

:
(i)

( ) ?

(ii)

(iii)

(iv)

56/1/2/D

Mr. Roy, the principal of one reputed school organized a seminar in which he invited
parents and principals to discuss the serious issue of diabetes and depression in
students. They all resolved this issue by strictly banning the junk food in schools and
to introduce healthy snacks and drinks like soup, lassi, milk etc. in school canteens.
They also decided to make compulsory half an hour physical activities for the students
in the morning assembly daily. After six months, Mr. Roy conducted the health survey
in most of the schools and discovered a tremendous improvement in the health of
students.
After reading the above passage, answer the following :
(i) What are the values (at least two) displayed by Mr. Roy ?
(ii) As a student, how can you spread awareness about this issue ?
(iii) What are tranquilizers ? Give an example.
(iv) Why is use of aspartame limited to cold foods and drinks ?
24.

C7H7ON

A, B, C, D E :

An aromatic compound A of molecular formula C7H7ON undergoes a series of


reactions as shown below. Write the structures of A, B, C, D and E in the following
reactions :

/OR
56/1/2/D

[P.T.O.

24.

(a)

:
(i)

Br2

(ii)

HCl

(iii) (CH3CO)2O / pyridine


(b)

:
C2H5NH2, C2H5OH, (CH3)3N

(c)

:
(CH3)2NH (CH3)3N

(a)

Write the structures of main products when aniline reacts with the following
reagents :
(i)

Br2 water

(ii)

HCl

(iii) (CH3CO)2O / pyridine


(b)

Arrange the following in the increasing order of their boiling point :


C2H5NH2, C2H5OH, (CH3)3N

(c)

Give a simple chemical test to distinguish between the following pair of


compounds :
(CH3)2NH and (CH3)3N

25.

- :
t/s
[CH3COOCH3]/mol L1

30

60

0.60

0.30

0.15

(i)

(ii)

30 60
(

56/1/2/D

log 2 = 0.3010, log 4 = 0.6021)


10

For the hydrolysis of methyl acetate in aqueous solution, the following results were
obtained :
t/s
[CH3COOCH3]/mol L1
(i)

30

60

0.60

0.30

0.15

Show that it follows pseudo first order reaction, as the concentration of water
remains constant.

(ii)

Calculate the average rate of reaction between the time interval 30 to 60 seconds.
(Given log 2 = 0.3010, log 4 = 0.6021)

25.

(a)

(b)

/OR
A + B P
= k[A] [B]2
(i) B ?
(ii) A ?
50% 30 90%

5

(log 2 = 0.3010)
(a)

For a reaction A + B P, the rate is given by


Rate = k[A] [B]2

(b)

(i)

How is the rate of reaction affected if the concentration of B is doubled ?

(ii)

What is the overall order of reaction if A is present in large excess ?

A first order reaction takes 30 minutes for 50% completion. Calculate the time
required for 90% completion of this reaction.
(log 2 = 0.3010)

26.

(a)

:
(i) HF HI
(ii)
(iii)

(b)

56/1/2/D

(i)

Cl F3

(ii)

XeF4

5
11

[P.T.O.

(a)

Account for the following :


(i)

Acidic character increases from HF to HI.

(ii)

There is large difference between the melting and boiling points of oxygen
and sulphur.

(iii) Nitrogen does not form pentahalide.


(b)

Draw the structures of the following :


(i)

Cl F3

(ii)

XeF4

/OR
26.

(i)

(ii)

?
Cl2 F2 ?

(iii)
(iv)

(v)

:
5

XeF2 + PF5
(i)

Which allotrope of phosphorus is more reactive and why ?

(ii)

How the supersonic jet aeroplanes are responsible for the depletion of ozone
layers ?

(iii) F2 has lower bond dissociation enthalpy than Cl2. Why ?


(iv) Which noble gas is used in filling balloons for meteorological observations ?
(v)

Complete the equation :


XeF2 + PF5
_________

56/1/2/D

12

CHEMISTRY MARKING SCHEME


DELHI -2015
SET -56/1/2/D

Qu
es.

Value points
CH3-CH2-Br

Marks

Because it is a primary halide / (10) halide


2

BaCl2 because it has greater charge / +2 charge

X2Y3

2, 5 - dinitrophenol

6.

(i)LiAlH4 / NaBH4 /H2, Pt


(ii)KMnO4 , KOH
When vapour pressure of solution is higher than that predicted by Raoults law /

1
1
1

the intermolecular attractive forces between the solute-solvent/(A-B) molecules are weaker than
those between the solute-solute and solvent-solvent molecules/A-A or B-B molecules.
Eg. ethanol-acetone/ethanol-cyclohexane/CS2-acetone or any other correct example
mixH is positive

7.

OR
7.

(a)Azeotropes are binary mixtures having the same composition in the liquid and vapour phase 1
and boil at a constant temperature.

8.

9.

10

(b) Minimum boiling azeotrope

eg - ethanol + water or any other example

(i)Ag+ (aq) + e- Ag (s)


Reaction with higher E0 value / G0 negative
(ii) Molar conductivity of a solution at infinite dilution or when concentration approaches
zero
Number of ions per unit volume decreases
Elements which have partially filled d-orbital in its ground states or any one of its oxidation
states.
1) Variable oxidation states
2) Form coloured ion
Or any other two correct characteristics
1) Diamminedichloridoethylenediaminechromium(III) chloride
2) [Co(NH3)5(ONO)]2+

1
+

1+ 1

11

(i)

1
1

(ii)t2 g3 e g1
(iii) sp3 , diamagnetic
12

The cell reaction : Fe(s) + 2H (aq) Fe2+ (aq) + H2(g)


+

Eocell = Eoc - Eoa


= [0-(-0.44)]V=0.44V
o

Ecell = Ecell - 0.059 log [ Fe2+]


+ 2
2
[H ]

Ecell = 0.44 V - 0.059 log ( 0.001 )


2
2
( 0.01 )
1
= 0.44 V - 0.059 log ( 10 )
2
= 0.44 V - 0.0295 V

13

14

= 0.410 V
(i) mutual coagulation
(ii)strong interaction between dispersed phase and dispersion medium or solvated layer
(iii)CO acts as a poison for catalyst.

1
1
1
1

(i)Hexamethylene diamine NH2 (CH2)6 NH2 and


adipic acid HOOC- (CH2)4- COOH
(ii)3 hydroxybutanoic acid CH3CH(OH)CH2COOH and
3 hydroxypentanoic acid CH3CH2CH(OH)CH2COOH
(iii)Chloroprene H2C=C(Cl)CH=CH2
IUPAC names are accepted

Note : mark for name /s and mark for structure / s


15

(i)CH3CH2CH3
(ii) C6H5COONa + CHI3
(iii)CH4

1
,
1

16.

(i) C6H5OH + NaOH C6H5ONa


CH3X
C6H5OCH3
Or
C6H5OH + Na C6H5ONa
CH3X
C6H5OCH3
1
(ii)CH3CH(OH)CH3

CrO3 or Cu/573K CH3COCH3

(i)CH3MgX
(ii)H2O

(iii)C6H5NH2 NaNO2 + HCl C6H5N2Cl H2O warm


273K

16.

(CH3)2C(OH)CH3

C6H5OH

OR
a)

1
b)

1
(Acetyl chloride instead of acetic anhydride may be used)
17

18

(i)Maltose

(ii) fibrous proteins: parallel polypeptide chain , insoluble in water


Globular proteins: spherical shape, soluble in water, (or any 1 suitable difference)
(iii) Vitamin D

(i)Larger surface area, higher van der Waals forces , higher the boiling point
(ii)Rotation due to one enantiomer is cancelled by another enantiomer
(iii) - NO2 acts as Electron withdrawing group or I effect

1
1
1

19
Tf = i Kf m
Tf = i Kf mb x1000
Mb x m a
1.62 K = i x 4.9K kg mol-1 x

3.9 g
122 gmol-1

1000
49 kg

i = 0.506

Or by any other correct method

20

21

As i<1 , therefore solute gets associated.

(i) Zinc being low boiling will distil first leaving behind impurities/ or on electrolysis the pure
metal gets deposited on cathode from anode.
(ii)Silica acts as flux to remove iron oxide which is an impurity as slag or FeO + SiO2 FeSiO3
(iii)Wrought iron
d=zxM
a3 NA
z = d a3 NA
M
z = 2.7 g cm-3 x 6.022 x1023 mol-1 x ( 4.05 x 10-8cm)3
27 g mol-1

1
1
1

= 3.999 4
Face centered cubic cell/ fcc
22

23

(i) 5f orbital electrons have poor shielding effect than 4f


(ii)due to d-d transition / or the energy of excitation of an electron from lower d orbital to higher
d-orbital lies in the visible region /presence of unpaired electrons in the d-orbital.
(iii) 2 MnO4- + 6 H+ + 5 NO2- 2 Mn2+ + 3 H2O + 5 NO3(i)

Concern for students health, Application of knowledge of chemistry to daily life,


empathy , caring or any other
(ii)Through posters, nukkad natak in community, social media, play in assembly or any other
(iii)Tranquilizers are drugs used for treatment of stress or mild and severe mental disorders .. Eg:
equanil (or any other suitable example)
(iv) Aspartame is unstable at cooking temperature.

1
1
1
1
,
1
,
1

24

1x5=
5

A=

B=

C=

D=

E=

OR
O 24.
a. i)
1
1
1
ii)

iii)

b. ( CH3)3N < C2H5NH2 < C2H5OH


c. By Hinsberg test secondary amines ( CH3)2NH shows ppt formation which is

1
1

tertiary insoluble in KOH , tertiary amines ( CH3)3N do not react with benzene sulphonyl choride

25
(a)

k = 2.303 log [ A0 ]
t
[A]

k = 2.303 log 0.60


30
0.30
k = 2.303 x
30

0.301 = 0.023 s-1

k = 2.303 log 0.60


60
0.15
k = 2.303 x 0.6021 = 0.023 s-1
60

As k is constant in both the readings, hence it is a pseudofirst order reaction.


ii)
Rate = - [R]/t
= -[0.15-0.30]
60-30

= 0.005 mol L-1s-1


OR
a)
25.

(i) Rate will increase 4 times of the actual rate of reaction.


(ii) Second order reaction

b)

t
1/2

= 0.693
k

30min =

1+1

0.693
k

k =

0.0231min-1

k = 2.303 log [ A0 ]
t
[A]
t = 2.303
0.0231

log 100
10

t = 2.303 min
0.0231
t = 99.7min

26. (a) (i) Due to decrease in bond dissociation enthalpy from HF to HI , there is an increase in acidic
character observed.
(ii)Oxygen exists as diatomic O2 molecule while sulphur as polyatomic S8
(iii)Due to non availability of d orbitals

1
1

(b)

26.
OR
(i) White Phosphorus because it is less stable due to angular strain
(ii)Nitrogen oxides emitted by supersonic jet planes are responsible for depletion of ozone layer.
Or
NO+O3 NO2+ O2
(iii)due to small size of F, large inter electronic repulsion / electron- electron repulsion among the
lone pairs of fluorine
(iv)Helium
(v) XeF2 + PF5 [XeF]+ [PF6]-

,
1

1
1
1

SET-1

H$moS> Z.

Series SSO/2

Code No.

amob Z.

56/2/1/F

narjmWu H$moS >H$mo Cma-nwpVH$m Ho$ _wI-n


>na Ad` {bIo &

Roll No.

Candidates must write the Code on the


title page of the answer-book.

H$n`m OmM H$a b| {H$ Bg Z-n _o _w{V n> 15 h &


Z-n _| Xm{hZo hmW H$s Amoa {XE JE H$moS >Z~a H$mo N>m Cma -nwpVH$m Ho$ _wI-n> na
{bI| &
H$n`m OmM H$a b| {H$ Bg Z-n _| >26 Z h &
H$n`m Z H$m Cma {bIZm ew$ H$aZo go nhbo, Z H$m H$_mH$ Ad` {bI| &
Bg Z-n H$mo nT>Zo Ho$ {bE 15 {_ZQ >H$m g_` {X`m J`m h & Z-n H$m {dVaU nydm
_| 10.15 ~Oo {H$`m OmEJm & 10.15 ~Oo go 10.30 ~Oo VH$ N>m Ho$db Z-n H$mo nT>|Jo
Ama Bg Ad{Y Ho$ XmamZ do Cma-nwpVH$m na H$moB Cma Zht {bI|Jo &
Please check that this question paper contains 15 printed pages.
Code number given on the right hand side of the question paper should be
written on the title page of the answer-book by the candidate.
Please check that this question paper contains 26 questions.
Please write down the Serial Number of the question before
attempting it.
15 minute time has been allotted to read this question paper. The question
paper will be distributed at 10.15 a.m. From 10.15 a.m. to 10.30 a.m., the
students will read the question paper only and will not write any answer on
the answer-book during this period.

agm`Z {dkmZ (gmpVH$)


CHEMISTRY (Theory)

{ZYm[aV g_` : 3 KQ>o

A{YH$V_ AH$ : 70

Time allowed : 3 hours


56/2/1/F

Maximum Marks : 70
1

P.T.O.

gm_m` {ZX}e :
(i)

g^r Z A{Zdm` h &

(ii)

Z g`m 1 go 5 VH$ A{V bKw-Cmar` Z h Ama `oH$ Z Ho$ {bE 1 AH$ h &

(iii)

Z g`m 6 go 10 VH$ bKw-Cmar` Z h Ama `oH$ Z Ho$ {bE 2 AH$ h &

(iv)

Z g`m 11 go 22 VH$ ^r bKw-Cmar` Z h Ama `oH$ Z Ho$ {bE 3 AH$ h &

(v)

Z g`m 23 _y`mYm[aV Z h Ama BgHo$ {bE 4 AH$ h &

(vi)

Z g`m 24 go 26 VH$ XrK-Cmar` Z h Ama `oH$ Z Ho$ {bE 5 AH$ h &

(vii)

`{X Amd`H$Vm hmo, Vmo bmJ Q>o~bm| H$m `moJ H$a| & H$Hw$boQ>am| Ho$ Cn`moJ H$s AZw_{V
Zht h &

General Instructions :
(i)

All questions are compulsory.

(ii)

Questions number 1 to 5 are very short answer questions and carry


1 mark each.

(iii)

Questions number 6 to 10 are short answer questions and carry 2 marks


each.

(iv)

Questions number 11 to 22 are also short answer questions and carry


3 marks each.

(v)

Question number 23 is a value based question and carry 4 marks.

(vi)

Questions number 24 to 26 are long answer questions and carry 5 marks


each.

(vii)

Use log tables, if necessary. Use of calculators is not allowed.

1.

{ZZ{b{IV `w_ _| H$mZ

A{^{H$`m A{YH$ VrdVm go H$aoJm Ama `m| ?


Ama CH3 CH2 I

SN2

CH3 CH2 Br

Which would undergo SN2 reaction faster in the following pair and why ?
CH3 CH2 Br and CH3 CH2 I
56/2/1/F

2.

gm_m` Vmn_mZ na g\$a H$m H$mZ-gm Ana$n (EbmoQ>mon) D$_r` $n go Wm`r h ?

Which allotrope of sulphur is thermally stable at room temperature ?

3.

{XE JE `m{JH$ H$m AmB.`y.nr.E.gr. Zm_ {b{IE :

HO CH2 CH = C CH3
CH3
Write the IUPAC name of the given compound :
HO CH2 CH = C CH3
CH3

4.

Cg `m{JH$ H$m gy `m h {Og_| Vd Y


AQ>\$bH$s` [a>{$ H$m 2/3dm ^mJ KoaVo h ?

ccp

OmbH$ ~ZmVm h Ama

Ho$ na_mUw
1

What is the formula of a compound in which the element Y forms ccp


lattice and atoms of X occupy 2/3rd of octahedral voids ?

5.

^m{VH$emofU CH$_Ur` h O~{H$ amgm`{ZH$emofU AZwH$_Ur` hmoVm h & `m| ?

Physisorption is reversible while chemisorption is irreversible. Why ?


6.

(i)

{ZZ{b{IV H$mbog H$m AmB.`y.nr.E.gr. Zm_ {b{IE

[Cr (en)3]Cl3
(ii)

(i)

{ZZ{b{IV H$mbog H$m gy {b{IE :


nmoQ>{e`_ Q>mB AmgbQ>mo H$mo_oQ>(III)

Write down the IUPAC name of the following complex :


[Cr (en)3]Cl3

(ii)

Write the formula for the following complex :


Potassium tri oxalato chromate(III)

56/2/1/F

P.T.O.

7.

amCQ> Ho$ {Z`_ go G$Um_H$ {dMbZ go `m Vmn` h ? EH$ CXmhaU Xr{OE & G$Um_H$
{dMbZ Ho$ {bE mixH H$m `m {M hmoVm h ?

2>

AWdm
E{OAmoQ>mon H$mo n[a^m{fV H$s{OE & amCQ> Ho$ {Z`_ go G$Um_H$ {dMbZ mam ~ZZo dmbm
E{OAmoQ>mon {H$g H$ma H$m hmoVm h ? EH$ CXmhaU Xr{OE &

What is meant by negative deviation from Raoults law ? Give an


example. What is the sign of mixH for negative deviation ?
OR
Define azeotropes. What type of azeotrope is formed by negative
deviation from Raoults law ? Give an example.
8.

{ZZ{b{IV A{^{H$`mAm| _| `moJ AmZo dmbo A{^H$maH$m| Ho$ Zm_ Xr{OE :


(i)

?
CH3 CHO CH3 CH CH3
|

OH
(ii)

?
CH3 COOH CH3 COCl

Name the reagents used in the following reactions :


(i)

(ii)
9.

(a)

?
CH3 CHO CH3 CH CH3
|
OH
?
CH3 COOH CH3 COCl

Obr` H$mna(II) bmoamBS> {db`Z Ho$ {dwV-AnKQ>Z Ho$ XmamZ H$WmoS> na


{ZZ{b{IV A{^{H$`mE hmoVr h :
Cu(s)
Cu2+ (aq) + 2e

H+ (aq) + e

1
H (g)
2 2

E0 = + 034 V
E0 = 000 V

CZHo$ _mZH$ AnM`Z BboQ>moS> {d^d (E0) Ho$ _mZm| Ho$ AmYma na H$WmoS> na {H$g
A{^{H$`m H$s g^mdZm (gwgJVVm) h Ama `m| ?
56/2/1/F

(b)

(a)

Am`Zm| Ho$ dV A{^J_Z Ho$ H$mobamD$e {Z`_ H$m H$WZ H$s{OE & BgH$m EH$
AZw`moJ {b{IE &

Following reactions occur at cathode during the electrolysis of


aqueous copper(II) chloride solution :
Cu2+ (aq) + 2e
Cu(s)
H+ (aq) + e

1
H (g)
2 2

E0 = + 034 V
0

E = 000 V

On the basis of their standard reduction electrode potential (E0)


values, which reaction is feasible at the cathode and why ?
(b)

State Kohlrausch law of independent migration of ions. Write its


one application.

10.

gH$_U Vd `m| n[adVu CnM`Z AdWmAmo H$mo X{eV H$aVo h


H$mZ-gm Vd gdm{YH$ CnM`Z AdWmE XemVm h Ama `m| ?

? 3d loUr

_|

(Sc go Zn)
2

Why do transition elements show variable oxidation states ? In 3d series


(Sc to Zn), which element shows the maximum number of oxidation
states and why ?
11.

Ob _| NaCl (_mob `_mZ = 585 g mol1) H$s {H$VZr _mm KwbmB OmE {H$
{h_mH$ 2C KQ> OmE, `h _mZVo hE {H$ NaCl nyU $n go {dK{Q>V hmoVm h ?
(Kf Ob Ho$ {bE = 186 K kg mol1)
372 g

Calculate the mass of NaCl (molar mass = 585 g mol1) to be dissolved in


372 g of water to lower the freezing point by 2C, assuming that NaCl
undergoes complete dissociation. (Kf for water = 186 K kg mol1)

56/2/1/F

P.T.O.

12.

{ZZ{b{IV ~hbH$m| Ho$ EH$bH$m| Ho$ Zm_ Ama CZH$s gaMZmE {b{IE :
(i)
Q>oarbrZ
(ii)
~Ho$bmBQ>
(iii) ~wZm-S

Write the names and structures of the monomers of the following


polymers :

13.

(i)

Terylene

(ii)

Bakelite

(iii)

Buna-S

(i)

{ZZ{b{IV _o H$mZ _moZmogHo$amBS> h :


Q>mM, _mQ>mog, \$Q>mog, gobwbmog
Abr` Eo{_Zmo Eo{gS>m| Ama jmar` Eo{_Zmo Eo{gS>m| Ho$ ~rM$ `m AVa hmoVm h ?
Cg {dQ>m{_Z H$m Zm_ {b{IE {OgH$s H$_r Ho$ H$maU _gyS>m| _| IyZ AmZo bJVm h &

(ii)
(iii)

14.

(i)

Which one of the following is a monosaccharide :


starch, maltose, fructose, cellulose

(ii)

What is the difference between acidic amino acids and basic amino
acids ?

(iii)

Write the name of the vitamin whose deficiency causes bleeding of


gums.

(i)
(ii)

{ZH$b Ho$ n[aH$aU _| H$m_ AmZo dmbr {d{Y Ho$ nrN>o Omo {gmV hmoVm h CgH$m
CoI H$s{OE &
gmoZo Ho$ {ZH$fU _| VZw NaCN H$s `m ^y{_H$m hmoVr h ?

(iii)

H$mna _Q>o `m hmoVm h

(i)

Indicate the principle behind the method used for the refining of
Nickel.

(ii)

What is the role of dilute NaCN in the extraction of gold ?

(iii)

What is copper matte ?

56/2/1/F

15.

25C

na {ZZ gob H$m {dwV-dmhH$ ~b (B.E_.E\$.) n[aH${bV H$s{OE

Zn | Zn2+ (0.001 M) | | H+ (0.01 M) | H2(g) (1 bar) | Pt(s)

E0

(Zn 2 / Zn )

= 0.76 V,

E0

(H / H )
2

= 0.00 V

Calculate the emf of the following cell at 25C :


Zn | Zn2+ (0.001 M) | | H+ (0.01 M) | H2(g) (1 bar) | Pt(s)

E0

(Zn 2 / Zn )

16.

= 0.76 V,

E0

(H / H )
2

= 0.00 V

{ZZ{b{IV AdbmoH$Zm| Ho$ {bE H$maUm| H$mo Xr{OE :


(i)
g_wr Ob Ama ZXr H$m Ob Ohm {_bVo h dhm EH$ S>oQ>m ~Z OmVm h &
(ii)
MmaH$mob H$s gVh na N2 Jg H$s Anojm NH3 Jg A{YH$ erKVm go A{Yemo{fV
hmoVr h &
(iii) MyU {H$E hE nXmW A{YH$ ^mdembr A{YemofH$ hmoVo h &

Give reasons for the following observations :

17.

(i)

A delta is formed at the meeting point of sea water and river


water.

(ii)

NH3 gas adsorbs more readily than N2 gas on the surface of


charcoal.

(iii)

Powdered substances are more effective adsorbents.

(i)

H$mbog [Pt(en)2Cl2]2+ Ho$ `m{_Vr` g_md`dm| H$mo Amao{IV H$s{OE &


{H$Q>b \$sS> {gmV Ho$ AmYma na `{X o > P h, Vmo d4 Am`Z H$m BboQ>m{ZH$
{d`mg {b{IE &
H$mbog [Ni(CN)4]2 H$m gH$aU H$ma Ama Mw~H$s` `dhma {b{IE &
(Ni H$m na_mUw H$_mH$ = 28)

(ii)
(iii)
(i)

Draw the geometrical isomers of complex [Pt(en)2Cl2]2+.

(ii)

On the basis of crystal field theory, write the electronic


configuration for d4 ion, if o P.

(iii)

Write the hybridization type and magnetic behaviour of the


complex [Ni(CN)4]2. (Atomic number of Ni = 28)

56/2/1/F

P.T.O.

18.

(a)
(b)
(c)

19.

EpH$b hbmBS>| Ob _| KwbZerb Zht h & `m| ?


`wQ>Z-1-Amb H$m{eH$s` {ZpH$` (YwdU AKyUH$) h naVw `wQ>Z-2-Amb
H$m{eH$s` g{H$` (YwdU KyUH$) h & `m| ?
`{n bmoarZ BboQ>mZ H$mo AmH${fV H$aZo dmbm Jwn h {\$a ^r `h BboQ>mZZohr
Eamo_{Q>H$ {VWmnZ A{^{H$`mAm| _| AmWm}- VWm nam- {ZX}eH$ h & `m| ?

(a)

Why are alkyl halides insoluble in water ?

(b)

Why is Butan-1-ol optically inactive but Butan-2-ol is optically


active ?

(c)

Although chlorine is an electron withdrawing group, yet it is


ortho-, para- directing in electrophilic aromatic substitution
reactions. Why ?

{ZZ{b{IV H$m $nmVaU Amn H$go H$a|Jo :


(i)
~oOmoBH$ E{gS> H$mo ~oOpS>hmBS> _|
(ii)
EWmBZ H$mo EWZb _|
(iii) Eogr{Q>H$ E{gS> H$mo _rWoZ _|
AWdm
{ZZ{b{IV A{^{H$`mAm| go g~pYV g_rH$aUm| H$mo {b{IE :
(i)
Q>r\$Z A{^{H$`m
(ii)
dmo\$-{H$Za AnM`Z
(iii) EQ>mS> A{^{H$`m
How do you convert the following :
(i)

Benzoic acid to Benzaldehyde

(ii)

Ethyne to Ethanal

(iii)

Acetic acid to Methane


OR

Write the equations involved in the following reactions :


(i)

Stephen reaction

(ii)

Wolff-Kishner reduction

(iii)

Etard reaction

56/2/1/F

20.

(a)

(b)

{ZZ{b{IV H$mo Amn H$maU XoVo hE H$go g_PmEJo :


(i)
Mn H$m CV_ bwAmoamBS> MnF4 h O~{H$ CV_ AmgmBS>
h &
(ii) gH$_U YmVwE Ama CZHo$ `m{JH$ CoaH$ JwUY_ XemVo h &
{ZZ{b{IV g_rH$aU H$mo nyU H$s{OE
2

3 MnO 4
(a)

(b)

+ 4H+

How would you account for the following :


(i)

Highest fluoride of Mn is MnF4 whereas the highest oxide is


Mn2O7.

(ii)

Transition metals and their compounds show catalytic


properties.

Complete the following equation :


2

3 MnO 4

21.

Mn2O7

+ 4H+

{ZZ{b{IV A{^{H$`mAm| Ho$ CnmXm| H$s mJw{$ H$s{OE :


(i)

i) B H
CH3 CH = CH2 2 6 ?
ii) 3 H 2O 2 / OH

(ii)

Br2 (aq)
C6H5 OH
?

(iii)

Cu / 573 K
CH3CH2OH ?

Predict the products of the following reactions :


(i)

i) B H
CH3 CH = CH2 2 6 ?
ii) 3 H 2O 2 / OH

(ii)

Br2 (aq)
C6H5 OH
?

(iii)

Cu / 573 K
CH3CH2OH ?

56/2/1/F

P.T.O.

22.

EH$ Vd X (_moba `_mZ = 60 g mol1) H$m KZd 623 g cm3 h & `{X `y{ZQ> gob
Ho$ H$moa H$s b~mB 4 108 cm h, Vmo `y{~H$ `y{ZQ> gob Ho$ H$ma H$s `m nhMmZ
hmoJr ?

An element X (molar mass = 60 g mol1) has a density of 6.23 g cm3.


Identify the type of cubic unit cell, if the edge length of the unit cell is
4 108 cm.

23.

~m| _| _Yw_oh Ama CXmgr Ho$ ~T>Vo Ho$gm| H$mo XoIZo Ho$ ~mX EH$ {g Hy$b Ho$ q{gnb
lr MmonS>m Zo EH$ go{_Zma H$m Am`moOZ {H$`m {Og_| Chm|Zo ~m| Ho$ A{^^mdH$m| VWm A`
Hy$bm| Ho$ q{gnbm| H$mo Am_{V {H$`m & Chm|Zo Hy$bm| _| gS>o hE ^mo` nXmWm] (OH$ \y$S>)
na {V~Y bJmZo H$m {ZU` {b`m, gmW hr `h {ZU` {b`m {H$ Hy$bm| _| dm`dYH$
nXmW Ogo gyn, bgr, XY Am{X H$Q>rZm| _| CnbY H$amB OmE & Chm|Zo `h ^r {ZU`
{b`m {H$ mV:H$mbrZ Ego~br Ho$ g_` ~m| H$mo {V{XZ AmYo KQ>o H$s emar[aH$ H$gaV
^r H$amB OmE & N>: _mh nMmV lr MmonS>m Zo ~m| Ho$ dm` H$m A{YH$V_ {dmb`m| _|
nwZ: {ZarjU H$adm`m Ama ~m| Ho$ dm` _| AZwn_ gwYma nm`m J`m &
Cn`w$ H$aU H$mo nT>Zo Ho$ ~mX, {ZZ{b{IV Zm| Ho$ Cma Xr{OE

(i)

lr MmonS>m mam {H$Z _y`m| (H$_-go-H$_ Xmo) H$mo Xem`m J`m h

(ii)

EH$ {dmWu Ho$ $n _|, Amn Bg {df` _| H$go OmJ$H$Vm \$bmEJo

(iii)

AdZ_Z-{damoYr S>J {~Zm S>mQ>a H$s gbmh Ho$ `m| Zht boZo Mm{hE

(iv)

H${_ _YwaH$ Ho$ Xmo CXmhaU Xr{OE &

?
?

Seeing the growing cases of diabetes and depression among children,


Mr. Chopra, the principal of one reputed school organized a seminar in
which he invited parents and principals. They all resolved this issue by
strictly banning the junk food in schools and by introducing healthy
snacks and drinks like soup, lassi, milk etc. in school canteens. They also
decided to make compulsory half an hour physical activities for the
students

in

the

morning

assembly

daily.

After

six

months,

Mr. Chopra conducted the health survey in most of the schools and
discovered a tremendous improvement in the health of students.
56/2/1/F

10

After reading the above passage, answer the following questions :


(i)

What are the values (at least two) displayed by Mr. Chopra ?

(ii)

As a student, how can you spread awareness about this issue ?

(iii)

Why should antidepressant drugs not be taken without consulting


a doctor ?

24.

(iv)

Give two examples of artificial sweeteners.

(a)

{ZZ{b{IV nXm| H$mo n[a^m{fV H$s{OE :


(i)
g{H$`U D$Om
(ii) Xa pWamH$

(b)

25%

{d`moOZ Ho$ {bE EH$ W_ H$mo{Q> H$s A{^{H$`m


A{^{H$`m Ho$ {bE t1/2 H$m n[aH$bZ H$s{OE >&
({X`m J`m

(a)

10

{_ZQ> boVr h &


5

: log 2 = 03010, log 3 = 04771, log 4 = 06021)

AWdm
EH$ amgm`{ZH$ A{^{H$`m R P Ho$ {bE gmU _| n[adVZ
ZrMo bmQ> _| {X`m J`m h :

ln [R] vs. g_` (s)

ln [R]
t (s)
(i)
(ii)
(iii)

(b)

56/2/1/F

A{^{H$`m H$s H$mo{Q> H$s mJw{$ H$s{OE &


dH$ H$m T>bmZ `m h ?
A{^{H$`m Ho$ {bE Xa pWamH$ H$s `y{ZQ> {b{IE &

XemBE {H$ 99% nyU hmoZo _| Omo g_` bJVm h dh Cg g_` H$m XwJwZm h Omo
A{^{H$`m Ho$ 90% nyU hmoZo _| bJVm h &
11

P.T.O.

(a)

(b)

Define the following terms :


(i)

Activation energy

(ii)

Rate constant

A first order reaction takes 10 minutes for 25% decomposition.


Calculate t1/2 for the reaction.
(Given : log 2 = 03010, log 3 = 04771, log 4 = 06021)
OR

(a)

For a chemical reaction R P, the variation in the concentration,


ln [R] vs. time (s) plot is given as

ln [R]
t (s)

25.

(i)

Predict the order of the reaction.

(ii)

What is the slope of the curve ?

(iii)

Write the unit of rate constant for this reaction.

(b)

Show that the time required for 99% completion is double of the
time required for the completion of 90% reaction.

(a)

{ZZ{b{IV Ho$ H$maU XoVo hE nQ> H$s{OE :

(b)

(i)

NH4

(ii)

XeF2

_| Omo Am~Y H$moU h dh NH3 Ho$ H$moU go CVa h &


(ii) H2O H$s Anojm H2S H$m dWZmH$ `yZVa h &
(iii) AnM`Z `dhma SO2 go TeO2 H$s Amoa KQ>Vm h &
{ZZ{b{IV H$s gaMZmE Amao{IV H$s{OE :
(i)
H4P2O7 (nm`amo\$m\$mo[aH$ Eo{gS>)
5

AWdm
56/2/1/F

12

(a)

(b)

(a)

(b)

{ZZ{b{IV H$s gaMZmE Amao{IV H$s{OE :


(i)

XeF4

(ii)

H2S2O7

{ZZ{b{IV Ho$ H$maU Xr{OE :


(i)
HCl go A{^{H$`m go Am`aZ FeCl2 ~ZmVm h Z H$s FeCl3.
(ii) HClO H$s Anojm HClO4 ~bVa Ab h &
(iii) dJ 15 Ho$ g^r hmBS>mBS>m| _| BiH3 ~bV_ AnMm`H$ h &

Account for the following :

(i)

Bond angle in NH4 is higher than NH3.

(ii)

H2S has lower boiling point than H2O.

(iii)

Reducing character decreases from SO2 to TeO2.

Draw the structures of the following :


(i)

H4P2O7 (Pyrophosphoric acid)

(ii)

XeF2
OR

(a)

(b)

56/2/1/F

Draw the structures of the following :


(i)

XeF4

(ii)

H2S2O7

Account for the following :


(i)

Iron on reaction with HCl forms FeCl2 and not FeCl3.

(ii)

HClO4 is a stronger acid than HClO.

(iii)

BiH3 is the strongest reducing agent amongst all the


hydrides of group 15.
13

P.T.O.

26.

(a)

(b)

`oH$ Ho$ {bE Cn`w$ CXmhaU XoVo hE {ZZ{b{IV A{^{H$`mAm| H$mo X{eV
H$s{OE :
(i)

A_moZrH$aU

(ii)

H$pbJ (`w_Z) A{^{H$`m

(iii)

Eo_rZm| H$m Eogr{Q>brH$aU

mW{_H$ (mB_ar), {Vr`H$ (goH$S>ar) Ama VVr`H$ (Q>{e`ar) E_rZm| H$s nhMmZ
H$aZo Ho$ {bE {hg~J {d{Y H$m dUZ H$s{OE & g~ A{^{H$`mAm| Ho$ amgm`{ZH$
g_rH$aUm| H$mo ^r {b{IE &

AWdm
(a)

(b)

O~ ~oOrZ S>mBEOmo{Z`_ bmoamBS> (C6 H5 N2Cl ) {ZZ{b{IV A{^H$maH$m| go


A{^{H$`m H$aVm h, V~ mV _w` CnmXm| H$s gaMZmE {b{IE :
(i)

HBF4 /

(ii)

Cu / HBr

{ZZ{b{IV A{^{H$`mAm| _|

A, B Ama C H$s

gaMZmE {b{IE :

5
(a)

(b)

56/2/1/F

Illustrate the following reactions giving suitable example in each


case :
(i)

Ammonolysis

(ii)

Coupling reaction

(iii)

Acetylation of amines

Describe Hinsberg method for the identification of primary,


secondary and tertiary amines. Also write the chemical equations
of the reactions involved.
14

OR
(a)

Write the structures of main products when benzene diazonium


chloride (C6 H5 N 2 Cl ) reacts with the following reagents :

(b)

56/2/1/F

(i)

HBF4 /

(ii)

Cu / HBr

Write the structures of A, B and C in the following reactions :

15

P.T.O.

CHEMISTRY MARKING SCHEME 2015


SET -56/2/1 F
Qn

Marks

Value points

CH3CH2I , because I is a better leaving group.

Rhombic sulphur

3-Methylbut-2-en-1-ol

X2Y3

Because of weak van der Waals forces in physisorption whereas there are strong chemical
forces in chemisorption.
i) tris-(ethane-1,2-diamine)chromium(III) chloride

ii) K3[ Cr(C2O4)3]

When solute- solvent interaction is stronger than pure solvent or solute interaction.

Eg: chloroform and acetone (or any other correct eg)

mixH= negative

6.

7.

OR
7.

Azeotropes binary mixtures having same composition in liquid and vapour phase and boil at 1
constant temperature / is a liquid mixture which distills at constant temperature without
undergoing change in composition

Maximum boiling azeotropes

eg: HNO3 (68%) and H2O(32%) (or any other correct example)
8.

9.

(i) CH3MgBr/ H3O+

(ii) PCl5/ PCl3 / SOCl2

a) Cu2+ (aq) + 2 e

Cu(s) because of high E0 value/ more negative G

b) It states that limiting molar conductivity of an electrolyte is equal to the sum of the individual 1
contributions of cations and anions of the electrolyte.
It is used to calculate the m0 for weak electrolyte / It is used to calculate and Kc
(Any one application) 1

10

11

a) Due to presence of unpaired d-electrons/ comparable energies of 3d and 4s orbitals.


b) Mn , due to involvement of 4s and 3d electrons/ presence of maximum unpaired delectrons.
Tf = i. Kf m
= i Kf wB x 1000

1
,

MB x wA
2K= 2 x 1.86K kg/mol x wB x 1000

58.5 g/mol x 37.2 g


wB = 1.17g

12

1
i)

1
ii)
Phenol

and

formaldehyde

1
iii)
(Note: half mark for structure/s and half mark for name/s)
13

i)
ii)
iii)

14

Fructose
1
Acidic amino acid has more number of acidic carboxylic group than basic amino
1
group whereas basic amino acid has more number of basic amino group.
Vitamin C
1

a) Impure Ni reacts with CO to form volatile Ni(CO)4 which when heated at higher 1
temperature decomposes to give pure Ni.
b) NaCN acts as a leaching agent to form a soluble complex with gold.

c) It is a mixture of Cu2S and FeS

15

E cell = E0 cell

log

E cell = 0.76 V -

V log

E cell = 0.76 0.0295 V log 10


1

= 0.7305 V
16

i)

Due to coagulation of colloidal clay particles.

ii)

Because NH3 is easily liquefiable than N2 due to its larger molecular size.

iii)

Because of more surface area.

17

i)
cis- isomer

18

19

trans-isomer

ii)

t2g4

iii)

dsp 2 , diamagnetic

1/2

a) Because they are unable to form H-bonds with water molecules.


b) Because of the presence of chiral carbon in butan-2-ol.
c) Due to dominating +R effect
i)
C6H5COOH PCl5 C6H5COCl H2/Pd
C6H5CHO

1
1
1
1

BaSO4
ii)

CHCH + H2O

iii)

CH3COOH

NaOH

Hg2+ /H2SO4
CH3COONa

CH3CHO
NaOH + CaO , heat

1
CH4

OR

19.

i)

1
ii)
1
iii)
3

20

21

i)

Because oxygen stabilizes Mn more than F due to multiple bonding

ii)

Because of their ability to show variable oxidation state(or any other correct reason)

2-

iii)

3MnO4 + 4H

i)

CH3CH2CH2OH

2MnO4 + MnO2 + 2H2O

1
1

1
ii)
iii)
22

d=
6.23 g cm-3 =

z=4

fcc

23

24

CH3CHO

a) Concern for students health, Application of knowledge of chemistry to daily life, empathy ,
, caring or any other
b) Through posters, nukkad natak in community, social media, play in assembly (or any other
1
relevant answer)
1
c) Wrong choice and overdose may be harmful
+
d) Aspartame, saccharin (or any other correct example)
a)i) Activation energy- Extra energy required by reactants to form activated complex.
ii) Rate constant- rate of reaction when the concentration of reactant is unity.

1
1

b)

k= 2.303 log [ A0 ]
t
[A]
k = 2.303 log 100
10 min
75

k = 2.303 x 0.125
10 min
4

k = 0.02879 min-1
t1/2 =

t1/2

24.07min
1

OR
24.

a) i)First order ii) -k

iii) s-1
1,1,1

b)
t=

log

t99% =
t=

log

x 2

t90% =

log

=
t99% = 2 x t90%
25

a) i)Because of lone pair in NH3 , lone pair- bond pair repulsion decreases the bond angle

ii)Because of absence of H-bonding in H2S

iii)Because stability of +4 oxidation state increases from SO2 to TeO2

1,1

b)
OR

25.

1,1

a)
b)i)Because iron on reaction with HCl produces H2(g) which prevents the formation of FeCl2 to

FeCl3 / Because HCl is a weak oxidising agent.


ii) Because of higher oxidation state of chlorine in HClO4

iii) Because of lower dissociation enthalpy of Bi-H bond.

26

a) i)ammonolysis

ii)

(any one)

iii)

(or any other correct reaction)


6

b)reaction of primary amine

(soluble in alkali)
Reaction of secondary amine

(insoluble in alkali)
Tertiary amine doesnt react

OR
26.
1
a) i)
1

ii)
,,

b) i) Aii) A- CH3CN

Sr.

B-

C-

,,

B- CH3CH2NH2

C- CH3CH2OH

Name

Sr.

Name

No.

No.

SET 2
.

Series : SSO/1

Code No.

56/1/2/D

- -

Roll No.

Candidates must write the Code on


the title page of the answer-book.

- 12
- - -
- 26
,
- 15 - 10.15
10.15 10.30 -
Please check that this question paper contains 12 printed pages.
Code number given on the right hand side of the question paper should be written on the
title page of the answer-book by the candidate.
Please check that this question paper contains 26 questions.
Please write down the Serial Number of the question before attempting it.
15 minutes time has been allotted to read this question paper. The question paper will be
distributed at 10.15 a.m. From 10.15 a.m. to 10.30 a.m., the students will only read the
question paper and will not write any answer on the answer-book during this period.

()
CHEMISTRY (Theory)
: 3 ]

[ : 70

Time allowed : 3 hours ]

[ Maximum Marks : 70

:
(i)

(ii)

- 1 5 - 1
- 6 10 - 2
- 11 22 - 3
- 23 4
- 24 26 - 5

(iii)
(iv)
(v)
(vi)
(vii)
56/1/2/D

[P.T.O.

General Instructions :
(i)

All questions are compulsory.

(ii)

Q. no. 1 to 5 are very short answer questions and carry 1 mark each.

(iii) Q. no. 6 to 10 are short answer questions and carry 2 marks each.
(iv)

Q. no. 11 to 22 are also short answer questions and carry 3 marks each.

(v)

Q. no. 23 is a value based question and carry 4 marks.

(vi)

Q. no. 24 to 26 are long answer questions and carry 5 marks each.

(vii) Use log tables if necessary, use of calculators is not allowed.

1.

SN2 ?
CH3 CH2 Br

CH3
|
CH3 C CH3
|
Br

Which would undergo SN2 reaction faster in the following pair and why ?
CH3
|
CH3 CH2 Br and CH3 C CH3
|
Br

2.

KCl
?

BaCl2

Out of BaCl2 and KCl, which one is more effective in causing coagulation of a
negatively charged colloidal Sol ? Give reason.

3.

Y ccp X 1/3
?
What is the formula of a compound in which the element Y forms ccp lattice and
atoms of X occupy 1/3rd of tetrahedral voids ?

56/1/2/D

4.

H3PO4

What is the basicity of H3PO4 ?

5.

..... :

Write the IUPAC name of the given compound :

6.

Name the reagents used in the following reactions :


(i)

(ii)

7.

?
CH3 CO CH3 CH3 C H CH3
|
OH
?
C6H5 CH2 CH3 C6H5 COOK+

?
mixH ?

What is meant by positive deviations from Raoults law ? Give an example. What is
the sign of mixH for positive deviation ?

/OR
7.

Define azeotropes. What type of azeotrope is formed by positive deviation from


Raoults law ? Give an example.
56/1/2/D

[P.T.O.

8.

(a)

:
Ag+(aq) + e Ag(s)

E = +0.80 V

1
H+(aq) + e H2(g)
2

E = 0.00 V


?
(b)

(a)

(E)

Following reactions occur at cathode during the electrolysis of aqueous silver


chloride solution :
Ag+(aq) + e Ag(s)

E = +0.80 V

1
H+(aq) + e H2(g)
2

E = 0.00 V

On the basis of their standard reduction electrode potential (E) values, which
reaction is feasible at the cathode and why ?
(b)

9.

Define limiting molar conductivity. Why conductivity of an electrolyte solution


decreases with the decrease in concentration ?

What are the transition elements ? Write two characteristics of the transition elements.

10.

(i)

(ii)

[Cr(NH3)2Cl2(en)]Cl (en = ethylenediamine)

-o- (III).
(i)

Write down the IUPAC name of the following complex :


[Cr(NH3)2Cl2(en)]Cl (en = ethylenediamine)

(ii)

Write the formula for the following complex :


Pentaamminenitrito-o-Cobalt (III).

56/1/2/D

.....

11.

(iii)

[Pt(NH3)2Cl2]
o < P d4
[Ni(CO)4] (.. Ni = 28)

(i)

Draw the geometrical isomers of complex [Pt(NH3)2Cl2].

(ii)

On the basis of crystal field theory, write the electronic configuration for d4 ion
if o < P.

(i)
(ii)

(iii) Write the hybridization and magnetic behaviour of the complex [Ni(CO)4].
(At.no. of Ni = 28)
12.

25 C emf :
Fe | Fe2+(0.001 M) || H+(0.01 M) | H2(g) (1 bar) | Pt(s)
E(Fe2+ | Fe) = 0.44 V E(H+ | H2) = 0.00 V

Calculate emf of the following cell at 25 C :


Fe | Fe2+(0.001 M) || H+(0.01 M) | H2(g) (1 bar) | Pt(s)
E(Fe2+ | Fe) = 0.44 V E(H+ | H2) = 0.00 V
13.

:
(i)
(ii)
(iii) CO

Give reasons for the following observations :


(i) Leather gets hardened after tanning.
(ii) Lyophilic sol is more stable than lyophobic sol.
(iii) It is necessary to remove CO when ammonia is prepared by Habers process.
14.


(i) -6, 6

(ii) PHBV
(iii)

Write the names and structures of the monomers of the following polymers :
(i) Nylon-6, 6
(ii) PHBV
(iii) Neoprene
56/1/2/D

[P.T.O.

15.

:
(i)

(ii)

CH3 C = O (i) H2N NH3


?
|
(ii) KOH/Glycol,
CH3
NaOH/I2
C6H5 CO CH3
?+?

NaOH / CaO
(iii) CH3 COONa ?

Predict the products of the following reactions :


(i)

(ii)

CH3 C = O (i) H2N NH3


?
|
(ii) KOH/Glycol,
CH3
NaOH/I2
C6H5 CO CH3 ? + ?

NaOH / CaO
(iii) CH3 COONa ?

16.

?
(i)

(ii)

-2- 2--2-

(iii)

How do you convert the following :


(i)

Phenol to anisole

(ii)

Propan-2-ol to 2-methylpropan-2-ol

(iii) Aniline to phenol

/OR
16.

(a)

- :
H+
2CH3CH2OH CH3CH2 O CH2CH3

(b)

(a)

Write the mechanism of the following reaction :


H+
2CH3CH2OH CH3CH2 O CH2CH3

(b)
56/1/2/D

Write the equation involved in the acetylation of Salicylic acid.


6

17.

(i)
(ii)
(iii)

: , , , ?
?
?

(i)

Which one of the following is a disaccharide : Starch, Maltose, Fructose,


Glucose ?

(ii)

What is the difference between fibrous protein and globular protein ?

(iii) Write the name of vitamin whose deficiency causes bone deformities in children.
18.

:
(a) t- n-
(b)
(c) o/p
(NO2)

Give reasons :
(a)
(b)
(c)

19.

n-Butyl bromide has higher boiling point than t-butyl bromide.


Racemic mixture is optically inactive.
The presence of nitro group (NO2) at o/p positions increases the reactivity of
haloarenes towards nucleophilic substitution reactions.

49 g 3.9 g 1.62 K
( )
( : = 122 g mol1, Kf = 4.9 K kg mol1)

3.9 g of benzoic acid dissolved in 49 g of benzene shows a depression in freezing point


of 1.62 K. Calculate the vant Hoff factor and predict the nature of solute (associated
or dissociated).
(Given : Molar mass of benzoic acid = 122 g mol1, Kf for benzene = 4.9 K kg mol1)
20.

(iii)


?
?

(i)

Indicate the principle behind the method used for the refining of zinc.

(ii)

What is the role of silica in the extraction of copper ?

(i)
(ii)

(iii) Which form of the iron is the purest form of commercial iron ?
56/1/2/D

[P.T.O.

21.

27 g mol1 4.05 108 cm


2.7 g cm3 , ?

An element with molar mass 27 g mol1 forms a cubic unit cell with edge length
4.05 108 cm. If its density is 2.7 g cm3, what is the nature of the cubic unit cell ?

22.

(a)

(b)

?
(i)

(ii)

2MnO4 + 6H+ + 5NO2


(a)

(b)

How would you account for the following :


(i)

Actinoid contraction is greater than lanthanoid contraction.

(ii)

Transition metals form coloured compounds.

Complete the following equation :

2MnO4 + 6H+ + 5NO2

23.



-
, ,
:
: :

:
(i)

( ) ?

(ii)

(iii)

(iv)

56/1/2/D

Mr. Roy, the principal of one reputed school organized a seminar in which he invited
parents and principals to discuss the serious issue of diabetes and depression in
students. They all resolved this issue by strictly banning the junk food in schools and
to introduce healthy snacks and drinks like soup, lassi, milk etc. in school canteens.
They also decided to make compulsory half an hour physical activities for the students
in the morning assembly daily. After six months, Mr. Roy conducted the health survey
in most of the schools and discovered a tremendous improvement in the health of
students.
After reading the above passage, answer the following :
(i) What are the values (at least two) displayed by Mr. Roy ?
(ii) As a student, how can you spread awareness about this issue ?
(iii) What are tranquilizers ? Give an example.
(iv) Why is use of aspartame limited to cold foods and drinks ?
24.

C7H7ON

A, B, C, D E :

An aromatic compound A of molecular formula C7H7ON undergoes a series of


reactions as shown below. Write the structures of A, B, C, D and E in the following
reactions :

/OR
56/1/2/D

[P.T.O.

24.

(a)

:
(i)

Br2

(ii)

HCl

(iii) (CH3CO)2O / pyridine


(b)

:
C2H5NH2, C2H5OH, (CH3)3N

(c)

:
(CH3)2NH (CH3)3N

(a)

Write the structures of main products when aniline reacts with the following
reagents :
(i)

Br2 water

(ii)

HCl

(iii) (CH3CO)2O / pyridine


(b)

Arrange the following in the increasing order of their boiling point :


C2H5NH2, C2H5OH, (CH3)3N

(c)

Give a simple chemical test to distinguish between the following pair of


compounds :
(CH3)2NH and (CH3)3N

25.

- :
t/s
[CH3COOCH3]/mol L1

30

60

0.60

0.30

0.15

(i)

(ii)

30 60
(

56/1/2/D

log 2 = 0.3010, log 4 = 0.6021)


10

For the hydrolysis of methyl acetate in aqueous solution, the following results were
obtained :
t/s
[CH3COOCH3]/mol L1
(i)

30

60

0.60

0.30

0.15

Show that it follows pseudo first order reaction, as the concentration of water
remains constant.

(ii)

Calculate the average rate of reaction between the time interval 30 to 60 seconds.
(Given log 2 = 0.3010, log 4 = 0.6021)

25.

(a)

(b)

/OR
A + B P
= k[A] [B]2
(i) B ?
(ii) A ?
50% 30 90%

5

(log 2 = 0.3010)
(a)

For a reaction A + B P, the rate is given by


Rate = k[A] [B]2

(b)

(i)

How is the rate of reaction affected if the concentration of B is doubled ?

(ii)

What is the overall order of reaction if A is present in large excess ?

A first order reaction takes 30 minutes for 50% completion. Calculate the time
required for 90% completion of this reaction.
(log 2 = 0.3010)

26.

(a)

:
(i) HF HI
(ii)
(iii)

(b)

56/1/2/D

(i)

Cl F3

(ii)

XeF4

5
11

[P.T.O.

(a)

Account for the following :


(i)

Acidic character increases from HF to HI.

(ii)

There is large difference between the melting and boiling points of oxygen
and sulphur.

(iii) Nitrogen does not form pentahalide.


(b)

Draw the structures of the following :


(i)

Cl F3

(ii)

XeF4

/OR
26.

(i)

(ii)

?
Cl2 F2 ?

(iii)
(iv)

(v)

:
5

XeF2 + PF5
(i)

Which allotrope of phosphorus is more reactive and why ?

(ii)

How the supersonic jet aeroplanes are responsible for the depletion of ozone
layers ?

(iii) F2 has lower bond dissociation enthalpy than Cl2. Why ?


(iv) Which noble gas is used in filling balloons for meteorological observations ?
(v)

Complete the equation :


XeF2 + PF5
_________

56/1/2/D

12

CHEMISTRY MARKING SCHEME


DELHI -2015
SET -56/1/1/D
Qu
es.

Value points

Marks

2, 5 - dinitrophenol

CH3-CH2-Br

Because it is a primary halide / (10) halide


4

BaCl2 because it has greater charge / +2 charge

X2Y3

6.

Elements which have partially filled d-orbital in its ground states or any one of its oxidation
states.
1) Variable oxidation states
2) Form coloured ion
Or any other two correct characteristics
1) Diamminedichloridoethylenediaminechromium(III) chloride

7.

1+ 1

2) [Co(NH3)5(ONO)]2+
8.

9.

(i)LiAlH4 / NaBH4 /H2, Pt

(ii)KMnO4 , KOH

When vapour pressure of solution is higher than that predicted by Raoults law /

the intermolecular attractive forces between the solute-solvent/(A-B) molecules are weaker than
those between the solute-solute and solvent-solvent molecules/A-A or B-B molecules.
Eg. ethanol-acetone/ethanol-cyclohexane/CS2-acetone or any other correct example
mixH is positive
OR

(a)Azeotropes are binary mixtures having the same composition in the liquid and vapour phase 1
and boil at a constant temperature.

10

(b) Minimum boiling azeotrope

eg - ethanol + water or any other example

(i)Ag (aq) + e Ag (s)

Reaction with higher E0 value / G0 negative

(ii) Molar conductivity of a solution at infinite dilution or when concentration approaches


zero
Number of ions per unit volume decreases

11
Tf = i Kf m
Tf = i Kf wb x1000
Mb x wa
1.62 K = i x 4.9K kg mol-1 x

3.9 g
122 gmol-1

1000
49 kg

i = 0.506
Or by any other correct method

12

13

As i<1 , therefore solute gets associated.

(i) Zinc being low boiling will distil first leaving behind impurities/ or on electrolysis the pure
metal gets deposited on cathode from anode.
(ii)Silica acts as flux to remove iron oxide which is an impurity as slag or FeO + SiO2 FeSiO3
(iii)Wrought iron
d=zxM
a3 NA
z = d a3 NA
M
z = 2.7 g cm-3 x 6.022 x1023 mol-1 x ( 4.05 x 10-8cm)3
27 g mol-1

1
1
1

= 3.999 4
Face centered cubic cell/ fcc
14

15

(i) 5f orbital electrons have poor shielding effect than 4f


(ii)due to d-d transition / or the energy of excitation of an electron from lower d orbital to higher
d-orbital lies in the visible region /presence of unpaired electrons in the d-orbital.
(iii) 2 MnO4- + 6 H+ + 5 NO2- 2 Mn2+ + 3 H2O + 5 NO3-

1
1
1

(i)

1
1

(ii)t2 g3 e g1
(iii) sp3 , diamagnetic

16

The cell reaction : Fe(s) + 2H (aq) Fe2+ (aq) + H2(g)


+

Eocell = Eoc - Eoa


= [0-(-0.44)]V=0.44V
o

Ecell = Ecell - 0.059 log [ Fe2+]


+ 2
2
[H ]

Ecell = 0.44 V - 0.059 log ( 0.001 )


2
2
( 0.01 )
1
= 0.44 V - 0.059 log ( 10 )
2
= 0.44 V - 0.0295 V

17

18

= 0.410 V
(i) mutual coagulation
(ii)strong interaction between dispersed phase and dispersion medium or solvated layer
(iii)CO acts as a poison for catalyst

1
1
1
1

(i)Hexamethylene diamine NH2 (CH2)6 NH2 and


adipic acid HOOC- (CH2)4- COOH
(ii)3 hydroxybutanoic acid CH3CH(OH)CH2COOH and
3 hydroxypentanoic acid CH3CH2CH(OH)CH2COOH
(iii)Chloroprene H2C=C(Cl)CH=CH2
IUPAC names are accepted

Note : mark for name /s and mark for structure / s


19

(i)CH3CH2CH3
(ii) C6H5COONa + CHI3
(iii)CH4

1
,
1

20

(i) C6H5OH + NaOH C6H5ONa


CH3X
C6H5OCH3
Or
C6H5OH + Na C6H5ONa
CH3X
C6H5OCH3
1
(ii)CH3CH(OH)CH3

CrO3 or Cu/573K CH3COCH3

(i)CH3MgX
(ii)H2O

(iii)C6H5NH2 NaNO2 + HCl C6H5N2Cl H2O warm


273K

C6H5OH

(CH3)2C(OH)CH3
1

20

OR
a)

1
b)

21

22

23

24

(Acetyl chloride instead of acetic anhydride may be used)

(i)Maltose

(ii) fibrous proteins: parallel polypeptide chain , insoluble in water


Globular proteins: spherical shape, soluble in water, (or any 1 suitable difference)
(iii) Vitamin D

(i)Larger surface area, higher van der Waals forces , higher the boiling point

(ii)Rotation due to one enantiomer is cancelled by another enantiomer

(iii) - NO2 acts as Electron withdrawing group or I effect

(i)

Concern for students health, Application of knowledge of chemistry to daily life,


empathy , caring or any other
(ii)Through posters, nukkad natak in community, social media, play in assembly or any other
(iii)Tranquilizers are drugs used for treatment of stress or mild and severe mental disorders .. Eg:
equanil (or any other suitable example)
(iv) Aspartame is unstable at cooking temperature.
(a) (i) Due to decrease in bond dissociation enthalpy from HF to HI , there is an increase in acidic
character observed.
(ii)Oxygen exists as diatomic O2 molecule while sulphur as polyatomic S8
(iii)Due to non availability of d orbitals

,
1
,
1
1
1
1

(b)
1

24
OR
(i) White Phosphorus because it is less stable due to angular strain
(ii)Nitrogen oxides emitted by supersonic jet planes are responsible for depletion of ozone layer.
Or
NO+O3 NO2+ O2
(iii)due to small size of F, large inter electronic repulsion / electron- electron repulsion among the
lone pairs of fluorine
(iv)Helium
(v) XeF2 + PF5 [XeF]+ [PF6]25

,
1
1
1
1
1x5
=5

A=

B=

C=

D=

E=

OR
O
25

a. i)
1
1
1
ii)

iii)

b. ( CH3)3N < C2H5NH2 < C2H5OH


c. By Hinsberg test secondary amines ( CH3)2NH shows ppt formation which is insoluble in1
KOH
while

1
tertiary amines ( CH3)3N do not react with benzene sulphonyl choride
5

26
(a)
1

k = 2.303 log [ A0 ]
t
[A]
k = 2.303 log 0.60
30
0.30
k = 2.303 x
30

0.301 = 0.023 s-1

k = 2.303 log 0.60


60
0.15

k = 2.303 x 0.6021 = 0.023 s


60

-1

As k is constant in both the readings, hence it is a pseudofirst order reaction.


ii)
Rate = - [R]/t
= -[0.15-0.30]
60-30

= 0.005 mol L-1s-1

OR
26

a)
(i) Rate will increase 4 times of the actual rate of reaction.
(ii) Second order reaction

b)

t
1/2

= 0.693
k

30min =

k =

1+1

0.693
k
0.0231min-1

k = 2.303 log [ A0 ]
t
[A]
t = 2.303
0.0231

log 100
10

t = 2.303 min
0.0231
t = 99.7min

H$moS> Z.

Series OSR/2

Code No.

amob Z.

56/2/1

narjmWu H$moS >H$mo Cma-nwpVH$m Ho$ _wI-n


>na Ad` {bIo &

Roll No.

Candidates must write the Code on the


title page of the answer-book.

H$n`m OmM H$a b| {H$ Bg Z-n _o _w{V n> 15 h &


Z-n _| Xm{hZo hmW H$s Amoa {XE JE H$moS >Z~a H$mo N>m Cma -nwpVH$m Ho$ _wI-n> na
{bI| &
H$n`m OmM H$a b| {H$ Bg Z-n _| >30 Z h &
H$n`m Z H$m Cma {bIZm ew$ H$aZo go nhbo, Z H$m H$_mH$ Ad` {bI| &
Bg Z-n H$mo nT>Zo Ho$ {bE 15 {_ZQ >H$m g_` {X`m J`m h & Z-n H$m {dVaU nydm
_| 10.15 ~Oo {H$`m OmEJm & 10.15 ~Oo go 10.30 ~Oo VH$ N>m Ho$db Z-n H$mo nT>|Jo
Ama Bg Ad{Y Ho$ XmamZ do Cma-nwpVH$m na H$moB Cma Zht {bI|Jo &
Please check that this question paper contains 15 printed pages.
Code number given on the right hand side of the question paper should be
written on the title page of the answer-book by the candidate.
Please check that this question paper contains 30 questions.
Please write down the Serial Number of the question before
attempting it.
15 minutes time has been allotted to read this question paper. The question
paper will be distributed at 10.15 a.m. From 10.15 a.m. to 10.30 a.m., the
students will read the question paper only and will not write any answer on
the answer-book during this period.

agm`Z {dkmZ (gmpVH$)


CHEMISTRY (Theory)

{ZYm[aV g_` : 3 KQ>o

A{YH$V_ AH$ : 70

Time allowed : 3 hours


56/2/1

Maximum Marks : 70
1

P.T.O.

gm_m` {ZX}e :
(i)

g^r Z A{Zdm` h &

(ii)

Z-g`m 1 go 8 VH$ A{V bKw-Cmar` Z h & `oH$ Z Ho$ {bE 1 AH$ h &

(iii)

Z-g`m 9 go 18 VH$ bKw-Cmar` Z h & `oH$ Z Ho$ {bE 2 AH$ h &

(iv)

Z-g`m 19 go 27 VH$ ^r bKw-Cmar` Z h & `oH$ Z Ho$ {bE 3 AH$ h &

(v)

Z-g`m 28 go 30 VH$ XrK-Cmar` Z h & `oH$ Z Ho$ {bE 5 AH$ h &

(vi)

Amd`H$VmZwgma bmJ Q>o~bm| H$m `moJ H$a| & H$Hw$boQ>am| Ho$ Cn`moJ H$s AZw_{V Zht h &

General Instructions :
(i)

All questions are compulsory.

(ii)

Questions number 1 to 8 are very short-answer questions and carry


1 mark each.

(iii)

Questions number 9 to 18 are short-answer questions and carry 2 marks


each.

(iv)

Questions number 19 to 27 are also short-answer questions and carry


3 marks each.

(v)

Questions number 28 to 30 are long-answer questions and carry 5 marks


each.

(vi)

Use Log Tables, if necessary. Use of calculators is not allowed.

1.

\o$Z bdZ {d{Y _| A`H$m| H$mo gm{V H$aZo _| gJmhH$m|


h ?

(collectors) H$m

`m H$m` hmoVm
1

What is the function of collectors in the froth floatation process for the
concentration of ores ?
2.

^m{VH$ A{YemofU hmoZo _| {H$g H$ma Ho$ ~b CmaXm`r hmoVo h

What type of forces are responsible for the occurrence of physisorption ?


56/2/1

3.

EH$b

NN

Am~Y EH$b

PP

Am~Y go X~b `m| hmoVm h

Why is the single N N bond weaker than the single P P bond ?


4.

{ZZ{b{IV gH$a {H$g H$ma H$s g_md`dVm

(isomerism) X{eV

H$aVm h

[Co(NH3)6][Cr(CN)6]
What type of isomerism is shown by the following complex :
[Co(NH3)6][Cr(CN)6]
5.

b.c.c.

EoH$H$ gob _| na_mUw {`m


g~Y H$mo `$ H$s{OE &

(r)

Ama gob Ho$ {H$Zmao H$s b~mB

(a)

Ho$ Amngr
1

Express the relationship between atomic radius (r) and the edge
length (a) in the b.c.c. unit cell.
6.

{ZZ{b{IV `m{JH$ H$m AmB.`y.nr.E.gr.

(IUPAC) Zm_

{b{IE

Write the IUPAC name of the following compound :

7.

BZ XmoZm| _| go H$mZ-gm A{YH$ jmar` h Ama `m|

Which of the two is more basic and why ?

8.

Q>mM ~ZmZo dmbo

-byH$moO

Ho$ Xmo KQ>H$m| Ho$ Zm_ {b{IE &

Name the two components of -glucose which constitute starch.

56/2/1

P.T.O.

9.

Ho$ EH$ {db`Z H$m b{Q>Z_ Ho$ BboQ>moS>m| Ho$ ~rM 5.0 Eonr`a {dwV Ymam go
20 {_ZQ> VH$ dwV AnKQ>Z {H$`m J`m & H$WmoS> na {ZH$b H$m {H$VZm `_mZ {Zjo{nV
hmoJm ?
Ni(NO3)2

({X`m J`m h : {ZH$b H$m na_mUw `_mZ = 58.7 g mol1, 1 F = 96500 C mol1)
A solution of Ni(NO3)2 is electrolysed between platinum electrodes using
a current of 5.0 ampere for 20 minutes. What mass of nickel will be
deposited at the cathode ?
(Given : At. Mass of Ni = 58.7 g mol1, 1 F = 96500 C mol1)
10.

A{^{H$`m H$s AY Am`w H$s n[a^mfm {b{IE & {ZZ Ho$ AY Am`w Ho$ {b`o `OH$
{b{IE :
(i)

ey` H$mo{Q> H$s A{^{H$`m

(ii)

W_ H$mo{Q> H$s A{^{H$`m

Define half-life of a reaction. Write the expression of half-life for

11.

(i)

zero order reaction and

(ii)

first order reaction.

{gda A`H$ go {gda Ho$ {ZH$fU go g~ amgm`{ZH$ A{^{H$`mE {b{IE &

Write the chemical reactions involved in the extraction of silver from


silver ore.

12.

g\$a Ho$ Xmo A{V _hdnyU Ana$nm| Ho$ Zm_ {b{IE & BZ XmoZm| _| go H$mZ-gm H$j Vmn
na Wm`r hmoVm h ? `m hmoVm h O~ Wm`r $n H$mo 370 K go D$na Ja_ {H$`m OmVm h ?

AWdm

56/2/1

(i)

gnH$ {d{Y go H2SO4 H$s m{ H$mo A{YH$V_ ~ZmZo Ho$ {V~Y {b{IE &

(ii)

Ob _o

H2SO4 Ho$

{bE Ka2 << Ka1 `m| h


4

Name the two most important allotropes of sulphur. Which one of the two
is stable at room temperature ? What happens when the stable form is
heated above 370 K ?
OR

13.

(i)

Write the conditions to maximize the yield of H2SO4 by contact


process.

(ii)

Why is Ka << Ka for H2SO4 in water ?


2

{ZZ g_rH$aUm| H$mo nyam H$s{OE

MnO 4

(i)

(ii)

Cr2O7

+ 5 S2 + 16 H+
+ 2 OH

Complete the following equations :

14.

(i)

2 MnO 4 + 5 S2 + 16 H+

(ii)

Cr2O7

+ 2 OH

gH$a [CoF6]3 Ho$ {b`o gH$aU AdWm, AmH${V Ama


(Co H$m na_mUw H$_mH$ = 27)

IUPAC Zm_

{b{IE &

Write the state of hybridization, shape and IUPAC name of the complex
[CoF6]3 . (Atomic no. of Co = 27)
15.

{ZZ Ho$ {bE amgm`{ZH$ g_rH$aU {b{IE :


(i)

O~ E{Wb bmoamBS> H$s Obr`

(ii)

O~ {ZOb
OmVr h &

AlCl3

KOH go

A{^{H$`m H$s OmVr h &

H$s CnpW{V _| bmoamo~oOrZ H$s

CH3COCl

go A{^{H$`m H$s

Write chemical equations when

56/2/1

(i)

ethyl chloride is treated with aqueous KOH.

(ii)

chlorobenzene is treated with CH3COCl in presence of anhydrous


AlCl3.
5

P.T.O.

16.

(a)

{ZZ `w_m| go {H$g EopH$b hobmBS> H$s Amn SN2 {H$`m{d{Y mam A{YH$ VrdVm
go A{^{H$`m H$aZo H$s Amem H$a|Jo Ama `m| ?
CH3 CH2 CH CH3

Br

(b)

SN1 A{^{H$`mAm|

(a)

Which alkyl halide from the following pairs would you expect to
react more rapidly by an SN2 mechanism and why ?

(b)

17.

CH3 CH2 CH2 CH2 Br

_| ao{g_rH$aU hmo OmVm h & `m|

CH3 CH2 CH CH3


CH3 CH2 CH2 CH2 Br

Br
Racemisation occurs in SN1 reactions. Why ?

{ZZ A{^{H$`m H$s {H$`m{d{Y {b{IE :

HBr
CH3CH2OH
CH3CH2Br H2O
Write the mechanism of the following reaction :

HBr
CH3CH2OH
CH3CH2Br H2O
18.

{ZZ A{^{H$`mAm| _| `w$ A{^H$maH$m| Ho$ Zm_ {b{IE :


(i)
\$sZmb Ho$ ~mo_rZoeZ go 2,4,6-Q>mB~mo_mo\$sZmb ~ZmZm
(ii)
`yQ>oZ-2-AmoZ go `yQ>oZ-2-Amob ~ZmZm
(iii) Eo{Zgmob H$m \$sS>ob H$mQ>g EopH$brH$aU
(iv) mW{_H$ EoH$mohmb Ho$ AmgrH$aU mam H$m~mopg{bH$ Ab ~ZmZm
Name the reagents used in the following reactions :

56/2/1

(i)

Bromination of phenol to 2,4,6-tribromophenol

(ii)

Butan-2-one to Butan-2-ol

(iii)

Friedel Crafts alkylation of anisole

(iv)

Oxidation of primary alcohol to carboxylic acid


6

19.

(i)

KCl {H$g

(ii)

{g{bH$mZ H$mo

(iii)

{ZZ _| go H$mZ-gm AmpdH$ R>mog H$m CXmhaU h


CO2 AWdm SiO2

(iv)

BZ_| go H$mZ-gm A{YH$ AN>o Mw~H$ ~ZmEJm, \o$amoMw~H$s` nXmW AWdm


\o$arMw~H$s` nXmW ?

H$ma H$m agg_rH$aU{_Vr` Xmof {XImVm h Ama `m|


As go

S>mo{nV H$aZo na {H$g H$ma H$m AYMmbH$ ~ZVm h

(i)

What type of stoichiometric defect is shown by KCl and why ?

(ii)

What type of semiconductor is formed when silicon is doped with


As ?

(iii)

Which one of the following is an example of molecular solid :

CO2 or SiO2

20.

(iv)

What type of substances would


ferromagnetic or ferrimagnetic ?

(i)

gmYmaU gob H$s VwbZm _|

(ii)

ZrMo Xr JB gob A{^{H$`m Ho$ {b`o gm` pWamH$

H2 O2

make

better

magnets,

BYZ gob Ho$ Xmo bm^ {b{IE &


(Kc) 10

h & BgHo$ {b`o

n[aH${bV H$s{OE &

gob
3

(i)

Write two advantages of H2 O2 fuel cell over ordinary cell.

(ii)

Equilibrium constant (Kc) for the given cell reaction is 10.


o

Calculate Ecell .

56/2/1

P.T.O.

21.

pWa Am`VZ AdWm _|


hE Wo :

SO2Cl2

Ho$ W_ H$mo{Q> Ho$ D$_r` AnKQ>Z _| {ZZ AmH$S>o m

SO2Cl2 (g) SO2 (g) Cl2(g)

g_`/s1 gH$b Xm~/atm

`moJ
1

0 .4

100

0 .7

doJ pWamH$ n[aH${bV H$s{OE &


({X`m J`m h : log 4 = 0.6021,

log 2 = 0.3010)

The following data were obtained during the first order thermal
decomposition of SO2Cl2 at a constant volume :

SO2Cl2 (g) SO2 (g) Cl2(g)


Experiment

Time/s1

Total pressure/atm

0 .4

100

0 .7

Calculate the rate constant.


(Given : log 4 = 0.6021, log 2 = 0.3010)
22.

(a)
(b)
(c)

56/2/1

R>mogm| na Jgm| Ho$ A{YemofU Ho$ {bE \$m`S>{bH$ A{YemofU g_Vmnr


(isotherm) Ho$ {bE EH$ g_rH$aU Ho$ $n _| `OH$ {b{IE &
_IZ Ho$ n[a{j mdWm Ama n[ajon _m`_ `m h ?
g_w Ama ZXr Ho$ {_bZo Ho$ WmZ na S>oQ>m ~ZVm h & `m| ?

(a)

Write the expression for the Freundlich adsorption isotherm for


the adsorption of gases on solids, in the form of an equation.

(b)

What are the dispersed phase and dispersion medium of butter ?

(c)

A delta is formed at the meeting place of sea and river water.


Why ?
8

23.

(a)

bWoZm`S> H$mZ-H$mZ gr {d{^ CnMm`r AdWmE {XImVo h

(b)

gH$_U Vdm| H$s Xmo {deofVmE {b{IE &

(c)

3d-bmH$
`m| ?

Ho$ Vdm| _| go {H$Z-{H$Z H$mo gH$_U Vd Zht _mZm Om gH$Vm h Ama


3

AWdm
{ZZ Ho$ {bE Cn`w$ H$maU {b{IE

+3

AmgrH$aU AdWm H$mo m H$aZo Ho$ {bE


2+
Mn `m{JH$ A{YH$ Wm`r hmoVo h &

Fe2+

(a)

AnZr

(b)

Sc (Z = 21)

(c)

Obr` {db`Z _|o

(a)

What are the different oxidation states exhibited by the


lanthanoids ?

(b)

Write two characteristics of the transition elements.

(c)

Which of the 3d-block elements may not be regarded as the


transition elements and why ?

go Zn (Z = 30) VH$ Ho$


na_mUwH$aU H$s EoWnr g~go H$_ hmoVr h &
Sc3+ aJhrZ

hmoVm h O~{H$

3d

`m{JH$mo H$s VwbZm _|

grarO Ho$ Vdm| _| go

Ti3+ aJrZ

Zn

H$s

hmoVm h &

OR
Assign suitable reasons for the following :
(a)

The Mn2+ compounds are more stable than Fe2+ towards oxidation
to their + 3 state.

(b)

In the 3d series from Sc (Z = 21) to Zn (Z = 30), the enthalpy of


atomization of Zn is the lowest.

(c)

56/2/1

Sc3+ is colourless in aqueous solution whereas Ti3+ is coloured.

P.T.O.

24.

{ZZ A{^{H$`mAm| _|

A, B Ama C H$s

gaMZmE ~VmBE :

(i)

HNO2
LiAlH4
KCN
CH3Br A
B
C
273 K

(ii)

NH3
CHCl NaOH
Br2 KOH
CH3COOH
A

B 3
C

Give the structures of A, B and C in the following reactions :

25.

(i)

HNO2
LiAlH4
KCN
CH3Br A
B
C
273 K

(ii)

NH3
CHCl NaOH
Br2 KOH
CH3COOH
A

B 3
C

{ZZ nXm| H$s n[a^mfmE {b{IE :


(a)
EoZmo_a
(b)
moQ>rZm| H$m {dH$VrH$aU
(c)
Amd`H$ Eo_rZmoo Ab

Define the following terms :

26.

(a)

Anomers

(b)

Denaturation of proteins

(c)

Essential amino acids

(i)

(iii)

EoQ>r{hQ>m_rZ H$s EH$ CXmhaU g{hV n[a^mfm {b{IE &


{ZZ Amf{Y`m| _| go H$mZ-gr {VO{dH$ h :
_m\$sZ, Bdm{Zb, bmoaEo\ ${ZH$mob, Eopn[aZ &
EonmQ>o_ H$m Cn`moJ R>S>o ^moOZ Ama no` nXmWmo VH$ gr{_V `m| hmoVm h

(i)

Define Antihistamine with an example.

(ii)

Which one of the following drugs is an antibiotic :


Morphine, Equanil, Chloramphenicol, Aspirin.

(iii)

Why is use of aspartame limited to cold food and drink ?

(ii)

56/2/1

10

27.

bmpQ>>H$ Ho$ Wbm| na {V~Y bJ OmZo Ho$ CnamV, EH$ Hy$b Ho$ N>mm| Zo {ZU` {b`m {H$
dh bmoJm| H$mo dmVmdaU Ama `_wZm ZXr na bmpQ>>H$ Ho$ Wbm| Ho$ hm{ZH$maH$ ^mdm| go
gy{MV H$a|Jo & ~mV H$mo A{YH$ ^mdr ~ZmZo Ho$ {bE, Chm|Zo Xgao Hy$bm| Ho$ gmW {_bH$a
EH$ abr aMr Ama gpO`m ~oMZo dmbm|, XH$mZXmam| Ama {S>nmQ>_oQ>b Q>moam| _| H$mJO Ho$
Wbo ~mQ>o & g^r N>mm| Zo U {H$`m {H$ do `_wZm ZXr H$mo ~MmZo Ho$ {bE ^{d` _| nmbrWrZ
Ho$ Wbm| H$m `moJ Zht H$a|Jo &
Cn`w$ boIme H$mo nT>H$a {ZZ Zm| Ho$ Cma Xr{OE

(i)

N>mm|o Zo {H$Z _y`m| H$mo Xem`m h

(ii)

Od-{ZZrH$aUr` ~hbH$ `m hmoVo h ? EH$ CXmhaU Xr{OE &

(iii)

`m nmbrWrZ EH$ g_ (hmo_mo) ~hbH$ h AWdm gh (co-) ~hbH$ h

After the ban on plastic bags, students of one school decided to create
awareness among the people about the harmful effects of plastic bags on
the environment and the Yamuna river. To make it more impactful, they
organized a rally by joining hands with other schools and distributed
paper bags to vegetable vendors, shopkeepers and departmental stores.
All students pledged not to use polythene bags in future to save the
Yamuna river.
After reading the above passage, answer the following questions :

56/2/1

(i)

What values are shown by the students ?

(ii)

What are biodegradable polymers ? Give one example.

(iii)

Is polythene a homopolymer or copolymer ?

11

P.T.O.

28.

(a)
(b)

(a)

(b)

(a)

dmnerb Ad`dm| dmbo {db`Z Ho$ {bE amCQ> {Z`_ {b{IE & g^r gmUm|o Ama
VmnH$_m| na amCQ> {Z`_ AZwgma ahZo dmbo {db`Z H$m Zm_ {b{IE &
Ob _| 10 g CaCl2 KmobZo go m hE {db`Z Ho$ {bE dWZmH$ C`Z
H$mo n[aH${bV H$s{OE & (Ob Ho$ {bE Kb = 0.512 K kg mol1,
CaCl2 H$m _moba `_mZ = 111 g mol1)
AWdm

200 g

{ZZ nXm| H$s n[a^mfmE {b{IE :


(i)

pWadmWr (Eo{O`moQ>mon)

(ii)

namgaUr (Amog_m{Q>H$) Xm~

(iii)

AUwg` (H$mo{bJo{Q>d) JwUY_

9.8% (w/w) H2SO4 Ho$ {db`Z H$s _mobaVm n[aH${bV H$s{OE `{X Bg {db`Z
H$m KZd 1.02 g ml1 hmo & (H2SO4 H$m _moba `_mZ = 98 g mol1)

State Raoults law for a solution containing volatile components.


and temperatures.
Calculate the boiling point elevation for a solution prepared by
adding 10 g of CaCl2 to 200 g of water. (Kb for water =
0.512 K kg mol1, Molar mass of CaCl2 = 111 g mol1)
OR

(a)

(b)

56/2/1

Define the following terms :


(i)

Azeotrope

(ii)

Osmotic pressure

(iii)

Colligative properties

Name the solution which follows Raoults law at all concentrations

(b)

Calculate the molarity of 9.8% (w/w) solution of H2SO4 if the density


of the solution is 1.02 g ml1. (Molar mass of H2SO4 = 98 g mol1)
12

29.

(a)

{ZZ Ho$ H$maU ~VmBE :


(i)
+ 5 AdWm _| Bi ~b CnMm`H$ hmoVm h &
(ii) PCl5 Vmo OmZm OmVm h naVw NCl5 Zht &
(iii) bmh HCl _| KwbH$a FeCl2 ~ZmVm h, FeCl3 Zht &

(b)

{ZZ H$s gaMZmE ~ZmBE :

(i)

XeOF4

(ii)

HClO4

AWdm
(a)

(b)

(a)

(b)

{ZZ H$s gaMZmE ~ZmBE :


(i)

H2S2O8

(ii)

bmb

P4

{ZZ Ho$ H$maU {b{IE

(i)

dmn AdWm _| JYH$ (g\$a) AZwMw~H$d X{eV H$aVm h &

(ii)

OrZmZ go {^, hr{b`_ H$m H$moB nQ amgm`{ZH$> `m{JH$ kmV Zht


h &

(iii)

H3PO3 go H3PO2 EH$

A{YH$ ~b AnMm`H$ h &

Account for the following :


(i)

Bi is a strong oxidizing agent in the + 5 state.

(ii)

PCl5 is known but NCl5 is not known.

(iii)

Iron dissolves in HCl to form FeCl2 and not FeCl3.

Draw the structures of the following :


(i)

XeOF4

(ii)

HClO4
OR

(a)

56/2/1

Draw the structures of the following :


(i)

H2S2O8

(ii)

Red P4
13

P.T.O.

30.

(b)

Account for the following :


(i)
Sulphur in vapour state exhibits paramagnetism.
(ii) Unlike xenon, no distinct chemical compound of helium is
known.
(iii) H3PO2 is a stronger reducing agent than H3PO3.

(a)

EoWoZb Ho$ {ZZ A{^H$maH$m| Ho$ gmW A{^{H$`m H$aZo na ~Zo CnmXm| H$mo {b{IE
(i)
CH3MgBr go Ama {\$a H3O+ go
(ii) Zn-Hg/gm HCl go

(iii)
(b)

56/2/1

VZw NaOH H$s CnpW{V _| C6H5CHO go


{ZZ `m{JH$ `w_mo _| nana ^oX H$aZo Ho$ {bE gab amgm`{ZH$ narjU Xr{OE
(i)
~oOmoBH$ Ab Ama E{Wb ~oOmoEQ>
(ii) monoZb Ama `yQ>oZ-2-AmoZ
AWdm

(a)

{ZZ Ho$ H$maU {b{IE :


(i)
HCN Ho$ gmW A{^{H$`m H$aZo _| CH3COCH3 go CH3CHO A{YH$
A{^{H$`merb hmoVm h &
(ii) go_rH$m~oOmBS> (H2NNHCONH2) _| Xmo NH2 Jwn hmoVo h & {\$a ^r
go_rH$m~mOmoZ ~ZmZo _| Ho$db EH$ NH2 Jwn {H$`mH$mar hmoVm h &

(b)

{ZZ Zm_Ymar A{^{H$`mAmo Ho$ {bE amgm`{ZH$ g_rH$aU {b{IE


(i)
amoOoZ_wS A{^{H$`m
(ii) hob-dmobmS>-Oo{bH$s A{^{H$`m
(iii) H$ZrOmamo A{^{H$`m

(a)

Write the products formed when ethanal reacts with the following
reagents :
(i)

CH3MgBr and then H3O+

(ii)

Zn-Hg/conc. HCl

(iii)

C6H5CHO in the presence of dilute NaOH


14

(b)

Give simple chemical tests to distinguish between the following


pairs of compounds :
(i)

Benzoic acid and Ethyl benzoate

(ii)

Propanal and Butan-2-one


OR

(a)

(b)

56/2/1

Account for the following :


(i)

CH3CHO is more reactive than CH3COCH3 towards reaction


with HCN.

(ii)

There
are
two
NH2 groups
in
semicarbazide
(H2NNHCONH2). However, only one is involved in the
formation of semicarbazone.

Write the chemical equation to illustrate each of the following


name reactions :
(i)

Rosenmund reduction

(ii)

Hell-Volhard-Zelinsky reaction

(iii)

Cannizzaro reaction

15

2,800
P.T.O.

MARKING SCHEME
Chemistry 2014
FOREIGN SET (56/2/1)
1
2
3
4
5
6
7
8
9

Collectors enhance non-wettability of the mineral/ore particles


van der Waals forces
Because of high inter-electronic repulsion of non bonding electrons owing to the small
bond length / atomic size
Coordination isomerism
3

r= a
or 4r =3 a
4
2 hydroxybenzaldehyde
CH3 NH2 ,because of the electron releasing (+I effect) tendency of methyl group
Amylose and amylopectin
m= z I t
I=5 A t= 20 x 60s = 1200s
atomic mass
m= n x F
xIxt
.

10

1
1
1
1
1
1
+
1

m=2 x 96500 C
x 5 A x 1200 s
m= 1.825 g
(or any other suitable method)
Half-life of a reaction is the time in which the concentration of a reactant is reduced
to half of its initial concentration.
(i)
(ii)

1
1

+
11

12

12

13

14

15

4Ag + 8 CN- + 2H2O + O2 4 [Ag(CN)2] - + 4 OH2[Ag(CN)2] - + Zn [Zn(CN)4] - 2 + 2Ag


Or
Ag2S + 4NaCN 2 Na[Ag(CN)2] + Na2S
2Na[Ag(CN)2] + Zn Na2 [Zn(CN)4] + 2Ag
(balancing of equation is not necessary)
Rhombic and Monoclinic
Rhombic Sulphur
Rhombic sulphur changes to monoclinic sulphur
OR
a) High pressure and low temperature
b) Because ionization of HSO-4 is difficult / removal of proton from negatively
charged HSO-4 is difficult.

1
1
1
1

(i)
(ii)
Hydridization : sp3d2
shape octahedral
IUPAC hexafluoridocobaltate(III)

(i)
(ii)

1
1

CH3 CH2- Cl + KOH (aq) CH3 CH2 OH +KCl

+
1

1
16

17

a) 1-Bromobutane / CH3 CH2 CH2 CH2Br


Because it is a primary alkyl halide
b) Because carbocation formed in SN1 reaction is sp2 hybridized and planar.

+
1

HBr H+ + Br-

Or

( where R = -CH3)

18

19

20

(i)
Br2 / H2O or aq. Br2
(ii)
LiAlH4 or NaBH4 or H2 / Ni (or any other)
(iii)
R Cl and anhyd . Al Cl3
(iv)
Acidic or alkaline KMnO4, K2Cr2 O7 (acidic)
(i) Schottky defect, due to similar size of K+ and Cl- ion
(ii) n-type
(iii) CO2
(iv) Ferromagnetic
a)
(i)
The fuel cell runs continuously as long as the reactants are supplied
(ii)
Highly efficient
(iii)
Pollution free
(any two)
b)

log Kc =

nE 0 cell
0.059

log Kc =

2xE 0 cell
0.059

2xE 0 cell
0.059
0.059
= 0.0295
2

+
1

log 10 =
E0cell =

x4=2

[log 10 = 1]
V
2

1
21

SO2

SO2 Cl2
At t = 0s

0.4 atm

At t = 100s

(0.4 x) atm

0 atm
x atm

Cl2
0 atm
x atm

Pt = 0.4 x + x + x
Pt = 0.4 + x
0.7 = 0.4 + x
x = 0.3
k=
k=
k=
k=
22

23

2.303
t
2.303

2.303

0.4

100

%.,-,
.--

a)
b)
c)
a)
b)

(i)
(ii)
(i)

(ii)

(iii)

0.4

log 0.8-0.7

23

25

#$
%#$ &#'

c)

24

log

log 0.1
1

x 0.6021 = 1.39 x 10-2 s-1


/

= k p1/n or log (x/m)= log k + 1/n log p


Dispersed phase = liquid
Dispersion medium = Solid
Because of coagulation of colloidal particles
+3 +2 +4 oxidation states
Transition elements
(i)
Form coloured compounds
(ii)
Form complexes
(iii)
Act as catalysts
(iv)
Paramagnetic
(v)
Form alloys
(vi)
Form interstitial compounds
(any two)
Or any other
Zn, because of fully filled d orbitals
OR
a) Because of stable half filled orbitals (3d5)
b) Because Zn has no unpaired electrons in d orbitals.
c) Because of the presence of one unpaired electron in Ti3+ whereas there is
no unpaired electron in Sc+3
A = CH3CN
B = CH3CH2NH2 C = CH3CH2OH
A = CH3 CONH2
B = CH3NH2
C = CH3NC
Anomers are the isomers which differ only in the configuration of
hydroxyl group at C-1 of glucose
Or
and 1 forms of glucose are called anomers
Denaturation of proteins when native protein is subjected to physical
or chemical change, it loses its biological activity and is called
denaturation.
Essential amino acids are the amino acids required in our diet for the
growth of the body / which are not synthesized by our body and
3

1
1
1
1

+
+
1
1
1
++
++
1

1
1

26

(i)

(ii)
(iii)
27

obtained through diet.


The drugs which are used to prevent the interaction of histamine with
the receptors present in the stomach wall. Eg. Cimetidine / Ranitidine /
Dimetapp (or any other)
Chloramphenicol
Because it is unstable at cooking temperature

(i) Concern towards environment / caring / socially aware / team work.

1
1
1

(atleast two values)


(ii) Polymers which can be degraded by the action of microorganisms. Eg.

PHBV , Nylon -2-nylon- 6/ any natural polymer


1

(iii) Homo polymer


28

(i)

(ii)

Raoults law : state that for a solution containing volatile components,


the partial vapour pressure of each component is directly proportional
to its mole fraction.
Ideal solution.
Wcacl

.---
78

1
1

Tb = i Kb xMcacl 2 x56
2

.-

1000

= 3x0.512 K kg mol-1x ...

x 200 kg

= 0.69K or 0.690C

1
1

OR
28

a)
(i)
(ii)

(iii)

b)

Azeotrope is a liquid mixture which boils at constant temperature with


constant composition.
Osmotic pressure : is the pressure applied on the solution side to stop
the flow of solvent across the semi permeable membrane from lower
concentration of the solution to higher concentration.
Colligative properties : are the properties of solution which depend upon
the no of moles of solute or concentration of solute and not on the
nature of solute.
;< 5<
1000
M =V(L)
=
x V( A)
(B Solute)
M=B

29

B.

<

.--x
.--

1.02

M = 1.02M
a) (i) Because Bi is more stable in +3 oxidation state.
(ii) Because of the availability to d orbital in P which is not in N/ nitrogen
cannot extend its covalency beyond 4
(iii) Because of the formation of H2(g) which prevents the oxidation of Fe+2 to
+3
Fe / HCl is only a mild oxidising agent

a) (i)

1
1

1x3=3

(ii)
1+1

OR
29

a) (i)

(ii)

Polymeric
b)
(i)
(ii)
(iii)

30

1
Because of the presence of two unpaired electrons .
Because of high ionization enthalpy of He.
Because of the presence of two P-H bonds in H3PO2 whereas in
H3PO3 one P-H bond is present.

a)
(i)
CH3-CHO CH3MgBr CH3CH(CH3)- OMgBr H3O+

CH3CH(OH)- CH3

(ii) CH3CHO

30

1
1

Zn-Hg
CH3-CH3
Conc HCl
(iii) C6H5CHO + CH3-CHO dil NaOH C6H5CH(OH) CH2CHO
(Award full marks even if only products are given)

b) (i) Add NaHCO3, benzoic acid will give brisk effervescence whereas ethyl benzoate
will not give this test. (or any other test)
(ii) Add tollens reagent , propanal will give silver mirror whereas Butan-2-one will
not give this test. (or any other test)
OR

a) (i) Because the positve charge on carbonyl carbon of CH3 CHO decreases to a lesser
extent due to one electron releasing (+I effect) CH3 group as compared to CH3 COCH3
(two electron releasing CH3 groups) and hence more reactive.
(ii) because one of the NH2 is involved in resonance with carbonyl group and hence
acquires positive charge.
(b) (i)

1
(ii)

(iii)

(or any other suitable reaction)

H$moS> Z.

Series OSR

Code No.

amob Z.

56/1

narjmWu H$moS >H$mo Cma-nwpVH$m Ho$ _wI-n


>na Ad` {bIo &

Roll No.

Candidates must write the Code on the


title page of the answer-book.

H$n`m OmM H$a b| {H$ Bg Z-n _o _w{V n> 15 h &


Z-n _| Xm{hZo hmW H$s Amoa {XE JE H$moS >Z~a H$mo N>m Cma -nwpVH$m Ho$ _wI-n> na
{bI| &
H$n`m OmM H$a b| {H$ Bg Z-n _| >30 Z h &
H$n`m Z H$m Cma {bIZm ew$ H$aZo go nhbo, Z H$m H$_mH$ Ad` {bI| &
Bg Z-n H$mo nT>Zo Ho$ {bE 15 {_ZQ >H$m g_` {X`m J`m h & Z-n H$m {dVaU nydm
_| 10.15 ~Oo {H$`m OmEJm & 10.15 ~Oo go 10.30 ~Oo VH$ N>m Ho$db Z-n H$mo nT>|Jo
Ama Bg Ad{Y Ho$ XmamZ do Cma-nwpVH$m na H$moB Cma Zht {bI|Jo &
Please check that this question paper contains 15 printed pages.
Code number given on the right hand side of the question paper should be
written on the title page of the answer-book by the candidate.
Please check that this question paper contains 30 questions.
Please write down the Serial Number of the question before
attempting it.
15 minutes time has been allotted to read this question paper. The question
paper will be distributed at 10.15 a.m. From 10.15 a.m. to 10.30 a.m., the
students will read the question paper only and will not write any answer on
the answer-book during this period.

agm`Z {dkmZ (gmpVH$)


CHEMISTRY (Theory)

{ZYm[aV g_` : 3 KQ>o

A{YH$V_ AH$ : 70

Time allowed : 3 hours


56/1

Maximum Marks : 70
1

P.T.O.

gm_m` {ZX}e :
(i)

g^r Z A{Zdm` h &

(ii)

Z-g`m 1 go 8 VH$ A{V bKw-Cmar` Z h & `oH$ Z Ho$ {bE 1 AH$ h &

(iii)

Z-g`m 9 go 18 VH$ bKw-Cmar` Z h & `oH$ Z Ho$ {bE 2 AH$ h &

(iv)

Z-g`m 19 go 27 VH$ ^r bKw-Cmar` Z h & `oH$ Z Ho$ {bE 3 AH$ h &

(v)

Z-g`m 28 go 30 VH$ XrK-Cmar` Z h & `oH$ Z Ho$ {bE 5 AH$ h &

(vi)

Amd`H$VmZwgma bmJ Q>o~bm| H$m `moJ H$a| & H$Hw$boQ>am| Ho$ Cn`moJ H$s AZw_{V Zht h &

General Instructions :
(i)

All questions are compulsory.

(ii)

Questions number 1 to 8 are very short-answer questions and carry


1 mark each.

(iii)

Questions number 9 to 18 are short-answer questions and carry 2 marks


each.

(iv)

Questions number 19 to 27 are also short-answer questions and carry


3 marks each.

(v)

Questions number 28 to 30 are long-answer questions and carry 5 marks


each.

(vi)

Use Log Tables, if necessary. Use of calculators is not allowed.

1.

amgm`{ZH$ emofU

(chemisorption) na

VmnH$_ H$m `m ^md hmoVm h

What is the effect of temperature on chemisorption ?


2.

{gda Ho$ {ZH$fU _| {OH$ YmVw H$m `m H$m` hmoVm h

What is the role of zinc metal in the extraction of silver ?


3.

H3PO3 H$s

jmaH$Vm (~o{gH$Vm) {H$VZr hmoVr h

What is the basicity of H3PO3 ?


56/1

4.

{ZZ OmoS>o _| {H$aob AUw H$mo nhMm{ZE :

Identify the chiral molecule in the following pair :

5.

{ZZ _| go H$mZ-gm mH${VH$ ~hbH$ h


~yZm-S, moQ>rZ|, PVC

Which of the following is a natural polymer ?


Buna-S, Proteins, PVC
6.

mW{_H$ Eoamo_{Q>H$ Eo_rZm| Ho$ S>mBEOmo{Z`_ bdUm| _| n[adVZ H$mo {H$g Zm_ go OmZm OmVm
h ?

The conversion of primary aromatic amines into diazonium salts is


known as ___________ .
7.

`yH$mog Ho$ Ob-AnKQ>Z (hydrolysis) Ho$ CnmX `m h

What are the products of hydrolysis of sucrose ?


8.

p-_o{Wb~OpS>hmBS>

H$s gaMZm {b{IE &

Write the structure of p-methylbenzaldehyde.


9.

KZd 2.8 g cm3 H$m EH$ Vd \$bH$ Ho$pV KZmH$ma (f.c.c.) H$ma H$m _mH$ gob
~ZmVm h {OgHo$ {H$Zmao H$s b~mB 4 108 cm h & Bg Vd H$m _moba `_mZ
n[aH${bV H$s{OE &
({X`m J`m h : NA = 6.022 1023 _mob 1)

An element with density 2.8 g cm3 forms a f.c.c. unit cell with edge
length 4 108 cm. Calculate the molar mass of the element.
(Given : NA = 6.022 1023 mol 1)
56/1

P.T.O.

10.

(i)

LiCl Ho$ Jwbm~r aJ Ho$ {bE BgH$m {H$g


(non-stoichiometric) Xmof CmaXm`r hmoVm h ?

(ii)

NaCl

H$ma H$m A-agg_rH$aU{_Vr`

{H$g H$ma H$m agg_rH$aU{_Vr` Xmof {XImVm h

AWdm
{ZZ{b{IV nXm| Ho$ OmoS>m| Ho$ ~rM Amn {d^oXZ H$go H$a|Jo
(i)
Q>oQ>mhoS>b VWm AmQ>mhoS>b [a{$`m
(ii)
{H$Q>b OmbH$ VWm _mH$ gob

(i)

What type of non-stoichiometric point defect is responsible for the


pink colour of LiCl ?

(ii)

What type of stoichiometric defect is shown by NaCl ?


OR

How will you distinguish between the following pairs of terms :

11.

(i)

Tetrahedral and octahedral voids

(ii)

Crystal lattice and unit cell

Am`Zm| Ho$ dV nbm`Z g~Yr H$mobamD$e (Kohlrausch) {Z`_ {b{IE & VZwH$aU
na {db`Z H$s MmbH$Vm H$_ `m| hmo OmVr h ?

State Kohlrausch law of independent migration of ions. Why does the


conductivity of a solution decrease with dilution ?
12.

56/1

EH$ amgm`{ZH$ A{^{H$`m, R P Ho$ {bE, g_`


Bg Jm\$ _| {XIm`m J`m h &

(order) gwPmBE

(i)

Bg A{^{H$`m H$s H$mo{Q>

(ii)

dH$ H$s dUVm (T>bmZ) `m hmoJr

&

(t)

Ho$ {V gmVm

(R)

_| n[adVZ H$mo
2

For a chemical reaction R P, the variation in the concentration (R) vs.


time (t) plot is given as

(i)
(ii)
13.

Predict the order of the reaction.


What is the slope of the curve ?

YmVwAm| Ho$ {dwV-AnKQ>Zr n[aH$aU H$m AmYma_yb {gmV g_PmBE & BgH$m EH$
CXmhaU Xr{OE &

Explain the principle of the method of electrolytic refining of metals. Give


one example.
14.

{ZZ g_rH$aUm| H$mo nyam H$s{OE


(i)

P4 + H2O

(ii)

XeF4 + O2F2

Complete the following equations :

15.

(i)

P4 + H2O

(ii)

XeF4 + O2F2

{ZZ H$s gaMZmE ~ZmBE


(i)

XeF2

(ii)

BrF3

Draw the structures of the following :

16.

(i)

XeF2

(ii)

BrF3

{ZZ A{^{H$`mAm| go g~pYV g_rH$aU {b{IE


(i)
amB_a Q>r_Z A{^{H$`m
(ii)
{d{b`_gZ gbofU (synthesis)

Write the equations involved in the following reactions :


(i)
Reimer Tiemann reaction
(ii)
Williamson synthesis
56/1

P.T.O.

17.

{ZZ A{^{H$`m H$s {H$`m{d{Y {b{IE

HBr

CH3CH2OH CH3CH2Br + H2O


Write the mechanism of the following reaction :
HBr

CH3CH2OH CH3CH2Br + H2O


18.

{ZZ ~hbH$m| H$mo m H$aZo Ho$ {bE `w$ EH$bH$m| Ho$ Zm_ {b{IE
(i)

~oHo$bmBQ>

(ii)

{ZAmorZ

Write the name of monomers used for getting the following polymers :

19.

(i)

Bakelite

(ii)

Neoprene

(a)

A{^{H$`m
Mg (s) + Cu2+ (Obr`) Mg2+ (Obr`) + Cu (s)

Ho$ {bE

rG

(a)

n[aH${bV H$s{OE &

o
: E gob = + 2.71 V, 1 F = 96500 C _mob 1

{X`m J`m h
(b)

Anmobmo (Apollo) AV[aj moJm_ Ho$ {bE {dwV e{$ CnbY H$amZo Ho$ {bE
`w$ gob Ho$ H$ma H$m Zm_ {b{IE &
o

Calculate rG for the reaction


Mg (s) + Cu2+ (aq) Mg2+ (aq) + Cu (s)
o

Given : E
(b)

56/1

cell

= + 2.71 V, 1 F = 96500 C mol1

Name the type of cell which was used in Apollo space programme
for providing electrical power.
6

20.

pWa Am`VZ AdWm _| SO2Cl2 Ho$ W_ H$mo{Q> Ho$ Vmnr` {dKQ>Z Ho$ XmamZ {ZZ{b{IV
AmH$S>o m hE :
SO2Cl2 (Jg) SO2 (Jg) + Cl2 (Jg)
`moJ

g_`/s1

gH$b Xm~/dm`w_S>b

0 .4

100

0 .7

doJ {Z`VmH$ n[aH${bV H$s{OE &


({X`m J`m h : log 4 = 0.6021,

3
log 2 = 0.3010)

The following data were obtained during the first order thermal
decomposition of SO2Cl2 at a constant volume :
SO2Cl2 (g) SO2 (g) + Cl2 (g)
Experiment

Time/s1

Total pressure/atm

0 .4

100

0 .7

Calculate the rate constant.


(Given : log 4 = 0.6021, log 2 = 0.3010)
21.

B_eg `m hmoVo h
Xr{OE &

BZHo$ {d{^ H$ma `m h

`oH$ H$ma H$m EH$$ CXmhaU


3

What are emulsions ? What are their different types ? Give one example
of each type.
22.

{ZZ{b{IV Ho$ H$maU Xr{OE


(i)

(CH3)3 P = O Vmo

(ii)

BboQ>mZ m H$aZo H$s G$Um_H$ {M dmbr EWnr H$m _mZ g\$a H$s Anojm
AmgrOZ Ho$ {bE H$_ hmoVm h &
H3PO3 H$s Anojm H3PO2 A{YH$ ~b AnMm`H$ h &

(iii)
56/1

nm`m OmVm h naVw

(CH3)3 N = O Zht

{_bVm &

P.T.O.

Give reasons for the following :

23.

(i)

(CH3)3 P = O exists but (CH3)3 N = O does not.

(ii)

Oxygen has less electron gain enthalpy with negative sign than
sulphur.

(iii)

H3PO2 is a stronger reducing agent than H3PO3.

(i)

gH$a

(ii)

gH$a [Co(en)3]3+ {H$g H$ma H$s g_md`dVm {XImVm h


(en = BWoZ-1,2-S>mBEo_rZ)

(iii)

[NiCl4]2

[Cr(NH3)4 Cl2]Cl H$m IUPAC Zm_

{b{IE &
?

`m| AZwMw~H$s` hmoVm h O~{H$ [Ni(CO)4] {VMw~H$s` hmoVm h


(na_mUw H$_mH$$ : Cr = 24, Co = 27, Ni = 28)

(i)

Write the IUPAC name of the complex [Cr(NH3)4 Cl2]Cl.

(ii)

What type of isomerism is exhibited by the complex [Co(en)3]3+ ?

?
3

(en = ethane-1,2-diamine)

24.

(iii)

Why is [NiCl4]2 paramagnetic but [Ni(CO)4] is diamagnetic ?


(At. nos. : Cr = 24, Co = 27, Ni = 28)

(a)

{ZZ _| go `oH$ A{^{H$`m Ho$ _wI EH$hbmoOZr CnmXm| H$s gaMZmE ~ZmBE :

(b)

{ZZ `w_m| _| go H$mZ-gm hbmoOZr `m{JH$


A{^{H$`m H$aoJm :
(i)
CH3Br AWdm CH3I
(ii)

56/1

(CH3)3 C Cl

AWdm

CH3 Cl
8

SN2

A{^{H$`m _| A{YH$ VrdVm go

25.

(a)

Draw the structures of major monohalo products in each of the


following reactions :

(b)

Which halogen compound in each of the following pairs will react


faster in SN2 reaction :
(i)

CH3Br or CH3I

(ii)

(CH3)3 C Cl or CH3 Cl

{ZZ{b{IV Ho$ H$maU {b{IE

(i)

VVr`H$ Eo_rZm|
hmoVo h &

(R3N) H$s

(ii)

Eo{ZbrZ \$sS>ob

(iii)

Obr` {db`Z _|

VwbZm _| mW{_H$ Eo_rZm|

(R-NH2) Ho$

dWZmH$ CVa

H$mQ>g A{^{H$`m Zht XoVr &


(CH3)3N

H$s VwbZm _|

(CH3)2NH

A{YH$ jmar` hmoVr h &

AWdm
{ZZ A{^{H$`mAm| _|

A, B

Ama

H$s gaMZmE Xr{OE

(i)

Sn HCl
H2O
NaNO2 HCl
C6H5NO2
A B
C
273 K

(ii)

H 2O / H
Br2 KOH
NH3
C
A
CH3CN
B

Account for the following :


(i)

Primary amines (R-NH2) have higher boiling point than tertiary


amines (R3N).

(ii)

Aniline does not undergo Friedel Crafts reaction.

(iii)

(CH3)2NH is more basic than (CH3)3N in an aqueous solution.


OR

56/1

P.T.O.

Give the structures of A, B and C in the following reactions :


(i)

Sn HCl
H2O
NaNO2 HCl
C6H5NO2
C
A B
273 K

(ii)

H 2O / H
Br2 KOH
NH3
CH3CN
C
A
B

26.

moQ>rZm| go g~pYV {ZZ nXm| H$s n[a^mfmE Xr{OE


(i)
noQ>mBS> Am~Y
(ii)
mW{_H$ gaMZm
(iii) {dH$VrH$aU

Define the following terms as related to proteins :

27.

56/1

(i)

Peptide linkage

(ii)

Primary structure

(iii)

Denaturation

{dd dm` {Xdg Ho$ Adga na, S>m. gVnmb Zo nmg Ho$ Jmd _| ahZo dmbo YZhrZ
{H$gmZm| Ho$ {bE EH$ dm` H$n bJm`m & OmM Ho$ ~mX, Cgo `h XoI H$a YH$m bJm
{H$ ~ma-~ma H$sQ>ZmeH$m| Ho$ gnH$ _| AmZo Ho$ H$maU {H$gmZm| _| go A{YH$m| H$mo H$ga H$m
amoJ hmo J`m Wm & CZ_| go ~hVm| H$mo _Yw_oh ^r Wm & Chm|Zo CZ_| YZ_w$ Amf{Y`m ~mQ>t &
S>m. gVnmb Zo Bg ~mV H$s gyMZm VH$mb ZoeZb y_Z amBQ>g H${_eZ (NHRC) H$mo
Xr & NHRC Ho$ gwPmdm| na gaH$ma Zo {ZU` {b`m H$s S>mQ>ar ghm`Vm Ama {dmr`
ghm`Vm bmoJm| H$mo Xr OmE Ama ^maV Ho$ g^r Jmdm| _| KmVH$ amoJm| Ho$ ^md H$mo amoH$Zo Ho$
{bE A`{YH$ gw{dYm dmbo AnVmb Imobo OmE &
(i)

(a)

(ii)

ApV_ H$ga _| nrS>m go ~MmZo Ho$ {bE _w`V`m H$mZ-gr nrS>mZmeH$ Amf{Y`m
`w$ H$s OmVr h ?

(iii)

_Yw_oh Ho$ amo{J`m| Ho$ {bE gwPmE JE H${_ _YwH$mam| _| go {H$gr EH$ H$m CXmhaU
Xr{OE &

S>m. gVnmb Ama

(b) NHRC mam

10

XemB JB _m` ~mV| {b{IE &

On the occasion of World Health Day, Dr. Satpal organized a health camp
for the poor farmers living in a nearby village. After check-up, he was
shocked to see that most of the farmers suffered from cancer due to regular
exposure to pesticides and many were diabetic. They distributed free
medicines to them. Dr. Satpal immediately reported the matter to the
National Human Rights Commission (NHRC). On the suggestions of
NHRC, the government decided to provide medical care, financial
assistance, setting up of super-speciality hospitals for treatment and
prevention of the deadly disease in the affected villages all over India.
(i)

28.

Write the values shown by


(a)

Dr. Satpal

(b)

NHRC.

(ii)

What type of analgesics are chiefly used for the relief of pains of
terminal cancer ?

(iii)

Give an example of artificial sweetener that could have been


recommended to diabetic patients.

(a)

{ZZ nXm| H$s n[a^mfm Xr{OE

(b)

(i)

_mobaVm

(ii)

_mobb C`Z pWamH$

(Kb)

EH$ Obr` {db`Z _| {V {bQ>a {db`Z _| 15 g `y[a`m (_moba `_mZ =


60 g _mob 1) Kw{bV h & Bg {db`Z H$m namgaU Xm~ Ob _| byH$moO (_moba
`_mZ = 180 g _mob 1) Ho$ EH$ {db`Z Ho$ g_mZ (g_namgar) h & EH$ {bQ>a
{db`Z _| CnpWV byH$moO H$m `_mZ n[aH${bV H$s{OE &

2, 3

AWdm
56/1

11

P.T.O.

(a)

EWoZmb Ama EogrQ>moZ H$m {_lU {H$g H$ma H$m {dMbZ {XImVm h
Xr{OE &

(b)

Ob _| byH$moO (_moba `_mZ = 180 g _mob 1) Ho$ EH$ {db`Z na bo~b bJm
h, 10% (`_mZ AZwgma) & Bg {db`Z H$s _mobbVm Ama _mobaVm `m hm|Jo ?
({db`Z H$m KZd = 1.2 g mL1)

(a)

(b)

H$maU

Define the following terms :


(i)

Molarity

(ii)

Molal elevation constant (Kb)

A solution containing 15 g urea (molar mass = 60 g mol1) per litre


of solution in water has the same osmotic pressure (isotonic) as a
solution of glucose (molar mass = 180 g mol1) in water. Calculate
the mass of glucose present in one litre of its solution.
OR

(a)

What type of deviation is shown by a mixture of ethanol and


acetone ? Give reason.

(b)

A solution of glucose (molar mass = 180 g mol1) in water is


labelled as 10% (by mass). What would be the molality and
molarity of the solution ?
(Density of solution = 1.2 g mL1)

29.

56/1

(a)

{ZZ g_rH$aUm| H$mo nyam H$s{OE

(i)

Cr2O72 + 2OH

(ii)

MnO4 + 4H+ + 3e

12

2, 3

(b)

{ZZ Ho$ H$maU {b{IE

(i)

Zn

(ii)

gH$_U YmVw ~hV go gH$a ~ZmVo h &

(iii)

Mn3+/Mn2+ `w_, Cr3+/Cr2+ `w_

H$mo gH$_U Vd Zht _mZm OmVm &


go H$ht A{YH$ Eo _mZ aIVm h &

2, 3

AWdm
(i)

gaMZm n[adVZerbVm Ama amgm`{ZH$ A{^{H$`merbVm Ho$ gX^ _| bWoZmBS>m|


Ama EopQ>Zm`S>m| Ho$ ~rM ^oX {b{IE &

(ii)

bWoZmBS> mIbm Ho$ Cg gX` H$m Zm_ {b{IE, Omo


{XImZo Ho$ {bE {g h &

(iii)

{ZZ g_rH$aU H$mo nyam H$s{OE

+4

AmgrH$aU AdWm

MnO4 + 8H+ + 5e
(iv)

Mn3+ Ama Cr3+

(na_mUw H$_mH$
(a)

(b)

_| go H$mZ A{YH$ AZwMw~H$s` h Ama `m|

: Mn = 25, Cr = 24)

Complete the following equations :

(i)

Cr2O72 + 2OH

(ii)

MnO4 + 4H+ + 3e

Account for the following :


(i)

Zn is not considered as a transition element.

(ii)

Transition metals form a large number of complexes.

(iii)

The E value for the Mn3+/Mn2+ couple is much more positive

than that for Cr3+/Cr2+ couple.


OR

56/1

13

P.T.O.

(i)

With reference to structural variability and chemical reactivity,


write the differences between lanthanoids and actinoids.

(ii)

Name a member of the lanthanoid series which is well known to


exhibit +4 oxidation state.

(iii)

Complete the following equation :


MnO4 + 8H+ + 5e

(iv)

Out of Mn3+ and Cr3+, which is more paramagnetic and why ?


(Atomic nos. : Mn = 25, Cr = 24)

30.

(a)

(b)

{ZZ A{^H$maH$m| go
(i)

HCN

(ii)

H2N OH

(iii)

VZw

NaOH

CH3CHO H$s

A{^{H$`m H$aZo na ~Zo CnmXm| H$mo {b{IE :

CH3CHO

H$s CnpW{V _|

{ZZ `m{JH$ `w_m| _| AVa {XImZo Ho$ {bE gab amgm`{ZH$ narjU {b{IE :
(i)

~OmoBH$ Ab Ama \$sZmb

(ii)

monoZb Ama monoZmoZ

3, 2

AWdm
(a)

(b)

(c)

{ZZ Ho$ H$maU {b{IE

(i)

CH3COOH H$s

VwbZm _| Cl CH2COOH A{YH$ ~b Ab h &

(ii)

H$m~m}pg{bH$ Ab H$m~m}{Zb g_yh H$s A{^{H$`mE Zht XoVo &

{ZZ Zm_ Ym[aV A{^{H$`mAm| Ho$ {bE amgm`{ZH$ g_rH$aU {b{IE


(i)

amoOoZ_wS> AnM`Z

(ii)

H${ZOmamo A{^{H$`m

CH3CH2 CO CH2 CH3

H$mZ Am`moS>mo\$m_ narjU XoVm h


56/1

Ama

CH3CH2 CH2 CO CH3

_| go
2, 2, 1

14

(a)

(b)

Write the products formed when CH3CHO reacts with the


following reagents :
(i)

HCN

(ii)

H2N OH

(iii)

CH3CHO in the presence of dilute NaOH

Give simple chemical tests to distinguish between the following


pairs of compounds :
(i)

Benzoic acid and Phenol

(ii)

Propanal and Propanone


OR

(a)

(b)

(c)

Account for the following :


(i)

Cl CH2COOH is a stronger acid than CH3COOH.

(ii)

Carboxylic acids do not give reactions of carbonyl group.

Write the chemical equations to illustrate the following name


reactions :
(i)

Rosenmund reduction

(ii)

Cannizzaros reaction

Out of CH3CH2 CO CH2 CH3 and CH3CH2 CH2 CO CH3,


which gives iodoform test ?

56/1

15

P.T.O.

Marking Scheme
Chemistry - 2014
Outside Delhi- SET (56 /1)
1

It first increases then decreases or graphical representation.

Zn acts as reducing agent.

1
1

4
2Chlorobutane

or first molecule of the pair.

or

Proteins

6.

Diazotization

7.

Glucose & Fructose

8.

9.

Given; d = 2.8g/cm3 ;

d=

M=

Z=4

a = 4 x 108 cm

or

M=

NA = 6.022 x 1023 per mol


2.8 g cm-3 4 x 10-8 cm x 6.022x1023


4

M = 2.8 x 16 x 101 x 6.022 = 26.97 g/mol


10

(i) Metal excess defect / Metal excess defect due to anionic vacancies filled by free electrons 1
/ Due to F centers.
(ii) Schottky defect.

1
Or

10

(i) Tetrahedral void is surrounded by 4 constituent particles (atoms / molecules / ions).


Octahedral void is surrounded by 6 constituent particles (atoms / molecules / ions).

OR
radius ratio (r

/r -) for Tetrahedral void is 0.225 & radius ratio for octahedral voids is

0.414
(ii) A regular three dimensional arrangement of points in space is called a crystal lattice.

Unit cell is the smallest portion of a crystal lattice which, when repeated in three
directions, generates an entire lattice. / unit cell is the miniature of crystal lattice /
microscopic edition of the crystal lattice.
11

Kohlrausch law of independent migration of ions. The law states that limiting molar 1
conductivity of an electrolyte can be stated as the sum of the individual contributions of the
anion and cation of the electrolyte.
On dilution,the conductivity () of the electrolyte decreases as the number of ions per unit 1
volume of solution decreases.

12

13

(i) Zero order reaction

(ii) slope = -k

In this method, the impure metal is made to act as anode. A strip of the same metal in pure 1
form is used as cathode. They are put in a suitable electrolytic bath containing soluble salt of
the same metal. Pure metal is deposited at the cathode and impurities remain in the solution.

14

For example: electro refining of Cu, Ag, Au (any one)

(i) P4 + H2O

(ii) XeF4 + O2F2

no reaction or if attempted in any form, award one mark

XeF6 + O2.

15

16

1+1

Reimer-Tiemann reaction
1

Williamson synthesis
1
17

HBr H+ + Br

Or

( where R = -CH3)
18

(i) Phenol & Formaldehyde

(ii) 2Chloro1,3butadiene (or Chloroprene)


19

(a) Given E Cell = +2.71V &

F = 96500C mol

-1

n = 2 (from the given reaction)

rGO = n x F x EoCell

rGO = 2 x 96500 C mol-1 x 2.71V

= - 523030 J / mol or - 523.030 kJ / mol

(b) Hydrogen oxygen fuel Cell / Fuel cell.


20

SO2 Cl2
At t = 0s

0.4 atm

At t = 100s

(0.4 x) atm

SO2

0 atm
x atm

Cl2
0 atm
x atm

Pt = 0.4 x + x + x
Pt = 0.4 + x
0.7 = 0.4 + x
x = 0.3
k=

2.303
t

log

"#

$"# %"&

k=
k=
k=
21

2.303
t
2.303
100s
$.)*)
+**,

log
log

0.4

0.8-0.7
0.4
0.1

x 0.6021 = 1.39 x 10-2 s-1

These are liquid-liquid colloidal systems or the dispersion of one liquid in another liquid.

Types: (i) Oil dispersed in water (O/W type) Example; milk and vanishing cream

(ii) Water dispersed in oil (W/O type) Example; butter and cream.

(Any one example of each type)


22

(i) As N cant form 5 covalent bonds / its maximum covalency is four.

(ii) This is due to very small size of Oxygen atom / repulsion between electrons is large in 1
relatively small 2p sub-shell.

23

(iii) In H3PO2 there are 2 PH bonds, whereas in H3PO3 there is 1 PH bond

(i) Tetraamminedichloridochromium (III) chloride.

(ii) Optical isomerism

(iii) In [NiCl4]2 ; Cl acts as weak ligand therefore does not cause forced pairing, thus
electrons will remain unpaired hence paramagnetic.

In [Ni(CO)4] ; CO acts as strong ligand therefore causes forced pairing, thus electrons will
become paired hence diamagnetic.
24

(a)

(b) (i) CH3I


(ii) CH3Cl
25

(i) As primary amines form inter molecular H bonds, but tertiary amines dont form H 1
bonds.

(ii) Aniline forms salt with Lewis acid AlCl3.

(iii) This is because of the combined effect of hydration and inductive effect (+I effect).

Or
25

(i) C6H5NO2

-./012

C6H5NH2

34 /012 ; $6)7

A
(ii) CH3CN

04 3 / 09

C6H5N2+Cl

04 3

CH3COOH

CH3CONH2

C6H5OH

C
:;4 / 730

+
+

CH3NH2

+
+

26

(i) Peptide linkage is an amide formed between COOH group and NH2 group ( -CO-NH- )

(ii) Specific sequence of amino acids in a polypeptide chain is said to be the primary 1
structure of the protein.

(iii) When a protein in its native form, is subjected to change in temperature or change in pH, 1
protein loses its biological activity. This is called denaturation of protein
27

(i) (a) dedicated towards work/ kind/ compassionate (any two).


(b) Dutiful / caring / humane in the large interest of public health in rural area.

(any other suitable value)

28

(ii) Narcotic analgesics

(iii) Aspartame / Saccharin / Alitame / Sucrolose.(any one)

(a)
(i) Molarity is defined as number of moles of solute dissolved in one litre of solution.

(ii) It is equal to elevation in boiling point of 1 molal solution.

(b)

For isotonic solutions:

urea = glucose
<=>?@

=>?@ AB

<=>?@
=>?@

<CD=EFB?

<CD=EFB?

W Glucose =

(As volume of solution is same)

CD=EFB? AB

or

CD=EFB?

I*J KL2MN

+GJ +O*J KL2MN


I*J KL2MN

+GH

<

CD=EFB?
= +O*J KL2
MN

= 45g

OR
28

(a) It shows positive deviation.

It is due to weaker interaction between acetone and ethanol than ethanol-ethanol interactions.

(b) Given: WB = 10g WS = 100g, WA = 90g MB = 180g/mol


<Q % S.TUQV +*

M=

L2.WQ.

+* +.$ +*

M=

+O*

= 0.66 M

or

0.66 mol/L

< (U. J)

+* +***
+O* [*

= 0.61m

29

<X +***

m=

m=

& d = 1.2g/m L

or 0.61mol/kg

(a) (i) Cr2O72 + 2 OH

(or any other suitable method)

2CrO42 + H2O

(ii) MnO4 + 4H+ + 3e

1
1

MnO2 + 2H2O

(b) (i) Zn / Zn2+ has fully filled d orbitals.

(ii) This is due to smaller ionic sizes / higher ionic charge and availability of d orbitals.

(iii) because Mn +2 is more stable(3d5) than Mn3+ (3d4). Cr+3 is more stable due to t2g3 / d3 1
configuration.
Or
29

(i)
Lanthanoids

Actinoids

Atomic / ionic radii does not show much Atomic / ionic radii show much variation /
variation / +3 is the most common oxidation Besides +3 oxidation state they exibit
state, in few cases +2 & +4

+4,+5,+6,+7 also.

They are quite reactive

Highly reactive in finely divided state

(Any two Points)


(ii) Cerium (Ce4+)

(iii) MnO4 + 8H+ + 5e

Mn2+ + 4H2O

(iv) Mn3+ is more paramgnetic

Because Mn3+ has 4 unpaired electrons (3d4) therefore more paramagnetic whereas Cr3+ has 3

unpaired electrons (3d3).

30

(a) (i)
1

1
(ii) CH3CH=NOH
(iii)
1

(b) (i) Add neutral FeCl3 in both the solutions, phenol forms violet colour but benzoic acid 1
does not.
1
(ii) Tollens reagent test: Add ammoniacal solution of silver nitrate (Tollens reagent) in
both the solutions propanal gives silver mirror whereas propanone does not.
(or any other correct test)
OR
30

(a) (i) As Cl acts as electron withdrawing group ( I effect) ,CH3 shows +I effect.

(ii) The carbonyl carbon atom in carboxylic acid is resonance stabilised.

(b) (i) Rosenmund reduction:

Or

RCOCl

04 / \ %: -3]

RCHO +HCl.

(ii) Cannizzaros Reaction:


1

Or With bezaldehyde
(c) CH3CH2CH2COCH3.

fi

Series : OSR/I

*]*,. s6nn,

iirTi.

q{qEwirffi

Roll No.

ffir-9fiffir

*{e,

Candidates must write the Code on


the title page of the answer-book.

o
o
o
r
o

![ttqt

dq ffi d fu'eq

lrFr-rrir

grrfi qiq

ErF+ rrq

*t

o
o
o

$qfr gsa 11

+t eilt fqqrrq*ts;rrqr *l ur* rtrcgfir+t *{e-gua.n

d f+'Vq

nFT-rEr

{ SO lrrq t

ffi

StrrfilrFtmvrrfrsqrvS66T++qtrd, svtclmqia^ wflqftrd I


gt wT-rH st q6+ + fdq 15 tne EFI TFrq flqqrrqr
er fuarur Wlq{ ii to.ts qi
flq,,qr !ilr+'n r 10.15 Tq t 10.30 ql a*"or* *m qsT-wr qtn et'{ qs erqftr + qkH t s+r-

yfiffirwettrirqaTfndi

o
r

s{q-rl:t

Please check that this question paper contains 11 printed pages.

Code number given on the right hand side of the question paper should be written on the
title page of the answer-book by the candidate.
Please check that this question paper contains 30 questions.
Please write down the Serial Number of the question before attempting it.
15 minutes time has been allotted to read this question paper. The question paper will be
disfibuted at 10.15 a.m. From 10.15 a.m. to 10.30 a.m,, the students will only read the
question paper and will not write any answer on the answer-book during this period.

Tqrq;Tfr{m@O
CHEMISTRY (Theory)
fuiffuerrt : 3

qv*j

Timeallowed:3hours

yafuw etq :70

t MaximumMarks:7O

snrr<r+qYr.

(i)

wtlsw efqqzii

(ii)

vw-d@rl da aaerfr ag-rafuvwt tv-dqsw #ftryr ;ra&

(iii) wld@rg *ra aoag-sadqsw* rsa*frw*fu2

erqr*

(iv) sw-iqrlg tzr aqq1ag-mt+r'w t rv-dqvw #frrq's eiqt r

(v)
(vi)

qlt-rmfrqvwt rs'dqs?a#ftzq's tiqt r


qrqwffiqstrdfuTfuitwyqfu *1 7 ffeyda<i *vqlrql aryqfu aaf*
sw-(i@rza

*ro

lP.T.O.

General Instructions

(i) All questions are compulsory.


(ii) Question numbers I to I are very short-answer questions and carry I mark each.
(iii) Question numbers 9 to 18 are short-answer questions and carry 2 marks each.
(iv) Question numbers 19 to 27 are also short-answer questions and carry i marks
each.

(v)

Question numbers 28 to 30 are long-answer questions and carry 5 mnrks each.

(vi) Use Log Tables, if necessary. Use of calculators is not allowed.

1.'qfl

i[

iH' eik'tm i[ srfr' ]FFR *' qqer+t s'r gs-ttsgqrflur

{H

Give one example each of 'oil in water' and 'water in oil' emulsion.

2.

foqfaaWns*qiw*sirg.t*ir{stHfu+Hqiq+ qqq'{*,,srnrq}rfuqrqmrt?
Which reducing agent is employed to get copper from the leached low grade copper
ore ?

3.

frTq

ffi { t s}t {T ifr{q^eTr+ 6tdr t eilr wif r


[Co(NH3)6]3*

oi{

[co(en)r]3+

Which of the following is more stable complex and why


[Co(NH)6]3+ and [Co(en)r]3+

4.

rq dFIfi.Hr IUPAC

{-IT

ImK+

cHr-cH-cHr-COOH
I

OH
Write the IUPAC name of the compound.

cHr-cH-cHr-COOH
I

OH

5.

Eq oTrqtqti 6*rn-emuHiy +

st{ rn qftrm'qru{vflE t r

s-qr{dtr{'ffi efu p-qr{irftrilH


Which of the following isomers is more volatile
o-nitrophenol or p-nitrophenol

6.

vrrwrur$ (HrEqi-dtfir) kmr{


What are isotonic solutions

56tUt

xr di t' r

7.

fiffi dffi

q+

qr+{ Tr*

C6H5NH2,

*'mc d q-cftqa 61m


(c2Hr2NH eftr crHrNtt,
goqvfr-dnr

Arrange the following compounds in increasing order of solubility in water

c6HsNH2, (C2H,2NH, C2H5NH2

8.

d +qi

st'il ii + frt{ qI

EreT

ii

ffi

t}or

tt

Which of the two components of starch is water soluble

9.

qT etr + x 10{ cm ffi #t


qrqrqruJu'qqrrmqfimitrdqfrH t

ll.2

gcm-3

(Na = 6'022

1023

ff

or qir.Htf f.c.c. afl?rcr rqrf,r

{ HEr
2

fr6-t;

An element with density tl.2 gcm-3 forms


Calculate the atomic mass of the element.

f.c.c. lattice with edge length of 4 x 10-8 cm.

(Given : No = 6.022x 1023 mol-l)

10. qfi M

rr{ q+*qr Fapi

A+ B- A+ BB-OB-A+BA+B-A+0A+
B- A+ B- A+

(i)
(ii1

q,r flTfrq{q H'{ sTri

{& q+ eq+f *ffi{

ffi :

A+

B_

qH B{rer*. GmaArft*o dq tqqqT qrer


*'qnor P*,w *'qqo tr{ fu Fnr wfltt qgdl t ?

Es fum Enr

(ii) Es iq

+{

tx-w)

HT

fus r+n*'silqk*.qEr{ tfir qiq Pqqr+ t

tr

Examine the given defective crystal

A+ B- A+ BB-OB-A+B_
A+B-A+0A+
B- A+ B- A+

A+

B-

Answer the following questions

(i)
(ii)
(iii)

$fltfi

What type of stoichiometric defect is shown by the crystal


How is the density of the crystal affected by this defect
What type of ionic substances show such defect

IP.T.O.

n.

qMmfr ftiH fu' oir"re rqqn =

dtrr

+'q\Fffi ft1 fuir+


qifir En+ fu'Es*-f6qi-qrrf o.+s K *t EF+ E+ qrA' r (r! = s.tzKkg
256 g

rTHr

6t 75 g ffi{

futr.

Calculate the mass of compound (molar mass = 256 gmol-l) to be dissolved in 75


benzene to lower its freezing point by 0.48 K (& = 5.tZ K kg mol-l).

t2.

qra ernlt fl{dq{ q+ qftqr$

sil{{s*l*tttrofuqtsdr fttrd

gof

Define an ideal solution and write one of its characteristics.


I

3.

aflItrqr *'

*q

(order)

ett E{r*t

srTurErfi

(molecularity) + s}'t q} Qr.ffi

ftfui t

Write two differences between 'order of reaction' and 'molecularity of reaction'.

14.

urEeif

(i)

*.yilqr *t
=f*

(*)

frfriqi *. e+nrcfc frqq flfifis+


qftFhn frfrT
FTrr

'

(i0 altffifrfti
Outline the principles behind the refining of metals by the following methods
Zone refining method

(i)
(ii)
15.

ga

Chromatographicmethod

q-ssfipn{r+dr{"if e} Trr *t
CarPr+ HrO ->

(i)
(ii) Cu + HrSOo{utO ->
g[?rdlT

fiTE{

(i)
(ii)

q\trs q*it qi y+*- qrq ffi

'1w

eripn anafuro E1k+

drHI - EEfr g* err**r M-s{ tdet w{sr


H2O, H2S, HrSe dturTe -v6* g$ enm HqHr srEqr
HF, HC/, Hsr

Complete the following chemical equations

(i)
(i0

C\Pz+ HrO -+
Cu + HrSOo(conc.) -+

OR
Arrange the following in the order of property indicated against each set :
(i) HF, HCr, HBr, HI - increasing bond dissociation enthalpy.
(ii) H2O, H2S, HrSe, HrTe - increasing acidic character.

16.

ri+"r 1cr(xg 3)4ct2i+ sT rupAC qrq

tt

fud

16 flsq mnr +t rmtErqErff (isomerism) ftrcrdr

Write the IUPAC name of the complex [Cr(NH)aCl21+. What type of isomerism does
it exhibit ?

L7

. (i)
(ii)

fua

gr t qir qr tka

nnBr
(a)

Frrr ftxftrqt

(i)

t Gil{ erRr*.+{ s*z orF+fuur tar t z

i-ifl{ststiT

a;
{ s*t *r s*z { t

qt{ rft Brfth.qr at fr t

(a) kqrg Et wrerlT (inversion)


(b) iftr+fiwr(Racemisation)
Which alkyl halide from the following pair is chiral and undergoes faster S*2

nA \n
reaction ?

Brl

(ii)

(a)

(i)
(ii)

(b)

Out of S*1 and S*2, which reaction occurs with

(a)
(b)
18. frq {

Br

Inversionofconfiguration
Racemisation

ur+o srfi{hqr if xgq

o_ou

qtritd

so%,

31q1q6t

O-cHz-cH:cH'+1*'

i{{fir

ffi'

Draw the structure of major monohalo product in each of the following reactions

(i)
(ii)

O-ou
GcHz

s9c4

'

- CH: CH2 + HBr

19. (a) steisfu{r erftNilqur rdrfr *. ffi


fed

Peroxide'

ulq qruT q{

Nt +

qftNilsrur

+ fu+ qqio.{ur

(b) ffitrk+.ria +t r'* te{iqffi kri


(c) r+1Fr-d srcrsTr + m."it *^ snqn q{ {dkd (associated) qEt-ifl?rs ek W+rrTq +fl?rg
inq6-qs'Bqrflsr{ffi r
(a) In reference to Freundlich adsorption isotherm write the expression for
r

adsorption of gases on solids in the form of an equation.

O)
(c)
S6lUt

Write an important characteristic of lyophilic sols.


Based on type of particles of dispersed phase, give one example each of
associated colloid and multimolecular colloid.

tp.T.O.

20. (a) frq

qq.Sf st d'r*rdt tfisn

(i)
(i0

qfrH

XeOFo

H2so4

(b) Y+fr srcst{s ak mm srstq + {'{ffnffi ,tE s'r rqa nfrH


(a) Draw the structures of the following molecules
(i) XeOFo
(ii) H2so4
(b) Write the structural difference between white phosphorus and red phosphorus.
r

2t.

flqr{+'ERor

(0

PC/3

(ii)

HC/

frfiri

st er*n eCl, qftm-stiiqW

(covalent)

t erF{hqT *i .R elt t Feclrq{f,r t Fec/, roT


(iii) e*ilq wl it i o-o *sarqrEqr qq1a fi et-fr t

Account for the following

(i)
(ii)

PC/, is more covalent than PC/r.


Iron on reaction with HC/ forms FeClrand not FeC/r.

(iii)
22.

fury

The two O-O bond lengths in the ozone molecule are equal.

qs6 srq{qr ii sorcl, *'qqq mq *'ar*q fue{

sOrC/r(e)---+
itFrrr gfi5} ,{Iqt 6g :

SO2(g) + C/2(B)

rqTTT

IITrt/s-1

Ftraqrqrdqrilrgfr

r00

0.4
0.7

+rflFniq,,qfi$ftrd+tH
(1og 4 =

0.6021, log 2 = 0.3010)

The following data were obtained during the


SO2C/2 at a constant volume :
SOrC/r(S)

---+

SO2( 91

frst

+ Cl2G)

Experiment

Time/s-1

Total pressure/atm

0.4

100

0.7

Calculate the rate constant.

(Given : log 4 = 0.6021, log 2 = 0.3010)


6

order thermal decomposition of

23.

(i)

edqq Hqq

+H

gi

qrd Tfi-qdi eB q"frib

En+

(ii) fftRhfrwrdlt rqs tstrcrq{H


(iii)

24.

Wq

(asparrame) mr

*'qi

{H

sq+q Fil Arrd dGsFii

ortr$Efui

tro'rftflqm

(i)
(ii)

What are antiseptics ? Give an example.

(iii)

Why is use of aspartame limited to cold foods and soft drinks

(i)

F*q frErirr

(iii)

(iii)

tr

Give two examples of macromolecules that are chosen as drug targets.

HI

*t

+.+

rm qT sienq{ (night-blindness) ei qrar

s{fi{Prqr ERr <T+tq

qintrt

(i)
(ii)

(ii) s{T qTr isr qrq {frrt} qi +{f, ptta };gMcrs

25.

sqrcrur

n-t*ilq

q?n

I q6 flsqr

tt fre-or

tt
r

r6ts *t {rfir }

qry*1

il wr
3

Deficiency of which vitamin causes night-blindness

Name the base that is found in nucleotide of RNA only.

Glucose on reaction with


structure of glucose ?

HI gives

n-hexane. What does

it

suggest about the

+ &ii w +s'Hrr qr+ *'sqrfr, \rs'q'd + BTrt + T6 fiTUiq fuqr t*,'q6 ialril si
qkflERuT eil{ q${r tfr qr
+ ftif + gqtTlq t qt{d qtt r Eu rSqqr q+ erftrs. ulTI*
"FTrfisr
ffir+ + fu+ sdl'+ ESt q.if *'qrq flqffir{ +d H oilr Edli qM H qdt, eqq
qA dt r qct urit + yur tsqr fu q5r r* fti
5-sr-rqTtf etk Miza {fit { $rrs +
gtkotrs++Hq6teTrfir*.+q?if q,rfult-rfi r
raTrftar

Btrttfi i{Hrs,, *1 qr+-r fiTq wqi

(0

orii rro wr rt

(ii) ffi

Erdrq

rri

{g?rqr NFftqg

(iii) wt dd*q {wr

* srr t

Hr d+ S r q+*rtroe+etur

(condensation)

erqqr {im-ir{ (addition)

frfrq{ t

After the ban on plastic bags, students of one school decided to make the people aware
of the harmful effects of plastic bags on environment and Yamuna River. To make the
awareness more impactful, they organized rally by joining hands with other schools
and distributed paper bags to vegetable vendors, shopkeepers and departmental stores.
All students pledged not to use polythene bags in future to save Yamuna River.

'
(i)
(ii)

(iii)

56lut

After reading the above passage, answer the following questions


What values are shown by the students

What are biodegradable polymers ? Give one example.


Is polythene a condensation or an addition polymer ?

!P.T.O.

26. (a) frq

srftfuqr +t ffdEfu

ffi

cH3cH2oH IrBr , cHrcHrBr + Hro

(b) ltq{-&m srfuf*qr+ffisfrq.{urfod


(a) Write the mechanism of the following reaction
r

cH3cH2oH HBr , cHrCHrBr + Hro

(b)

Write the equation involved in Reimer-Tiemann reaction.

27. frrr srflTlmqrcTi*'A, B silcc*tffiqdrt+.

(i) cHrnrI$eLiA&,nffic
(ii) cH3cooH+L o Brr+KoHr, CHC/r+NaoH
A
ETSr{T

fql{qfig+{er$Hqr(,t t

(i)

rrtnffimrtFr*rtt,

(ii) q+iEq,,oIRttF'I*ffi{,
(iii)

tFffihmr

N-ffi{'qqqr{sii,

6uq TrsRF{s' {rmqrror

ffi

r;

Give the structures of A, B and C in the following reactions

(i)

cHrBr KCN>R

LilHor"ffi"

--r --^; B CHC/.+NaOH


-^^--3'^'--^^
(ii) cH3cooH-NH. A Br^+KOH
^+
OR

How will you convert the following

(i)
(ii)
(iii)

Nitrobenzene into aniline


Ethanoic acid into methanamine

AnilineintoN-phenylethanamide
(Write the chemical equations involved.)

56lllt .,)

28. (a)

fiTqqqistqfr{rqrtffi:
(i) dtrd fu{{qrdffidr (Limiting molar conductivity)

2,3

(ii) tq{t--f, (Fuel cell)


(b) q6'{qtrfi.ti{ { 0.1 dFT t;r qr KC, irr fudffi{ qa t r $rfir sftriq 100 o t r qft
{S ti{ if 0.02 +m rr qr<ur irr KC/ .wr *i .R smr.s2o o *fir t d o.oz *m

rt + KC/ + fudrqr 6t {qtrfiflr sk dffi {qirffir qffii{il dftr+ r 0.1 dil rl


rcl kerq;r +t dqre+ar 1.29 x 10-2 Q-r cm-r d*

SNTET

(a) N
ii

ur

iqd [qtra (electrolysis) sT e6eII lmq ffi

s{qErfrrfr

q,{+

+ftr+

ffi

Wq?st

qs'dm
et onqsqqmr &fr t

(b) 298 K q{ fTq to q;r .*1qfimfufr s1H

gu2+

Effi qt Cu

Mg(s) | ug2+10.t M) ll cu2+ (0.01) | cu(s)


tf{qr

(a)

Ei"u = +2.71V,

Define the following terms

(i)
(ii)
O)

F = 96500 C mol-l1

Limiting molar conductivity


Fuel cell

Resistance of a conductivity cell filled with 0.1 mol L-r KC/ solution is 100 O.
KC, solution is
If the resistance of the same cell when filled with 0.02 mol
520 d2, calculate the conductivity and molar conductivity of 0.02 mol I;1 KCI
solution. The conductivity of 0.1 mol L-l KC/ solution is 1.29 x 10-2 (l-1 cm-I.

I;l

OR

(a)

State Faraday's

frst law of electrolysis. How much

charge in terms of Faraday is

required for the reduction of 1 mol of Cu2+ to Cu.

O)

Calculate emf of the following cell

at298K

Me(s) | Ms2+(0.1 M) ll Cu2+ (0.01) | Cu(s)


[Given Ei"u = +2.71V,

F = 96500 C mol

1]

2s. (a) eilq+tffiret

2,rr

(i) Mnort IlMnoo ?


(ii) Nqcro+ * NqCrrO, ?

O) qrorleri:

(i) Fe2* *t 6on tt Mn2* +3 eqqsr st ofrtr-fr d+ { olfrro*ntr t


(ii) 3dq'i' *'ten r"r qqi$f { zn* ffi ffi+.{ur fri tffi rrq$ oqd*t
(iii) M{Tsilf,c}.n5gii ffi erqTr(rg.dorit
I

qq{t
56tU1

[P.T.O.

(i)

3d q,l' *'nq
*il q{rfdr r

irtr sl 'nq fu{i qt sTft{$nq

nEsES erqrqr( uqd

sT

flr

.rr

tsr

(ii) 3d q{ or q*q qr {iffi'qur srq Eo(M2+[vI) s'r ffinrrs-qFr rEl-fri t etk eii z
(iii)

cp+ silr Mn3* i[

t qil{ erftrosqo oTrqstsn*.t ekwif z

(iv) d++rggqri*'ssntrsrilqkr**

+2

effi3qm

rrytErffi*'Hyfirct

(v) w{qtfi{q*}Trrft1H:

MnOa+8H++5e----+

(a)

How do you prepare

(,
(ii)
(b)

IlMnOo from MnO,

NarCrrO, from NarCrOo

Account for the following

(0
(i0
(iii)

?
?

Mn2+ is more stable than Fe2+ towards oxidation to +3 state.

The enthalpy of atomization is lowest for Zn in 3d series of the transition


elemenis.

Actinoid elements show wide range of oxidation states.


OR

(i)
(i0
(iii)
(iv)

(v)

Name the element of 3d transition series which shows maximum number of


oxidation states. Why does it show so ?

which transition metal of 3d series has positive E"(M2+714) value and why ?
Out of Cr3+ and Mn3*, which is a sffonger oxidizing agent and why

Name a member of the lanthanoid series which is well known to exhibit +2


oxidation state.
Complete the following equarion

MnOa+8H++5e---+

30. (a)

rq s{futsqroTt61ryme ftrri

(i)

(ii)

Ctro
2 C6H5CHO +

312

-H.N_oH H*,
qrq

. NaOH

----+

(iii) cH3cooH cl2tP


O) q)trEii +frEr gii { srf,t Eti *'m
>

(i)

(ii)

sre {r$qt{s'Tfti"r

++d-ons$ki;+{+.sirr{
nit+Ft stTsiH{
OPFTT

56tUt

10

ful

(a) w+-*,nqfui'

(i)

t curcno erftm.fuqrvftm t
(i0 ffi,FT +16n { olqtffim erra erftr*-mrfi {FT rtm t
(b) B*1 ;1wnt s{fuf{rqrcTf +'ffi ttrflqFffi qtsr,r ffi
(i) qriF-tser{ sTrlqgr
HCN

*'srq s{Frtffir if cHrcocH,

(ii) Nm{qr{
(ii1

(a)

+M sfirt*ur

213

Write the products of the following reactions

(i) (-)-o
+H2N_oH H:
\,_J
(ii) 2 C6H'CHO + conc. NaOH->
(iio cH3cooH cl2lP

(b)

>

Give simple chemical tests to distinguish between the following pairs of

compounds:

(i)
(ii)

Benzaldehyde and Benzoic acid


Propanal and hopanone

OR

(a)

Account for the following

(i)
(ii)
(b)

CH3CHO is more reactive than CHTCOCH3 towards reaction with HCN.

Carboxylic acid is a stronger acid than phenol.

Write the chemical equations to illustrate the following name reactions


(i) Wolff-Kishnerreduction

(ii) Aldolcondensation
(iii) Cannizzaroreaction

56tUt

11

CHEMISTRY MARKING SCHEME


DELHI -2014
SET -56/1/1
Qn

Answers

Marks

Oil in water : milk / vanishing cream (any one)

Water in oil : butter / cold cream (any one)

Hydrogen / Iron

[Co(en)3]3+ : because (en) is a chelating ligand / bidentate ligand

3-hydroxybutanoic acid / 3-hydroxybutan-1-oic acid

o nitrophenol

6.

Solutions with sameosmotic pressure

7.

C6H5NH2<(C2H5)2 NH< C2H5NH2

8.

Amylose

9.

d=11.2 g/cm3
z=4
a=4x10-8 cm
Z x M

d=N

11.2 =

4 x M
6.022x1023 x(4x10-8)3

a x a

M=

. .

M =11.2x6.022x16x10-1

M =107.9gmol-1 or 107.9 u
10

(i) Schottky defect


(ii) Decreases
(iii Alkali metal halides/ Ionic substances having almost similar size of cations and anions
(NaCl/KCl )

11

Tf =

K f x w2 x 1000

w1 x M2
W

2
0.48K = 5.12Kkgmol-1x75 x 256
x 1000

w2 =

0.48 x 75 x 256
5.12 x 1000

w2= 1.8g
12

Solutions which obey Raoults law over the entire range of concentration

A-A or B-B ~A-B interactions


Hmix = 0
1

Vmix = 0
(any one)
13

14

15

(i) Order of reaction is meant for elementary as well as for complex reactions but molecularity is 1
for elementary reactions.
(ii) Order can be zero or fraction but molecularity cannot be zero or fraction.
(or any other difference)
(i) Impurities are more soluble in melt than in solid state of the metal.

(ii) Different components of a mixture are differently adsorbed on an adsorbent

(i) Ca3 P2 + 6H2O3Ca(OH)2 + 2PH3

(ii) Cu + 2H2 SO4CuSO4 + 2H2O+ SO2

(give full credit even if correct products are mentioned)


OR

15

16

17

(i) HI < HBr < HCl < HF

(ii) H2O< H2S < H2Se< H2Te

(i) Tetraamminedichloridochromium (III) ion

(ii) Geometrical isomerism / cis trans

(i) (b) is chiral

OR

(a) undergoes faster SN2


(ii) (a) SN2

(b) SN1
18

(i)

(ii)
19

(a)

Cl

CH2 CH2 CH2 Br


$
%

= Kp 1'n

or

log (x/m)= log K + 1/n log p

(b) Reversible in nature/ stable sol/ solvent loving (or any other)

(c) Associated colloid Soap/ micelles;Multimolecular colloid - S8/ gold sol. (or any other)

20

a) (i)

(ii)

1+1

b)

White phosphorus

It exists as discrete tetrahedral P4unit

Red phosphorus
It exists in the form of polymeric chain.

(i) Because +5 oxidation state is more covalent than +3/ high charge to size ratio / high

OR correct structures.
21

polarizing power
(ii) Because HCl is a mild oxidising agent/ formation of hydrogen gas prevents the formation of

FeCl3 .
(iii) Because of resonance in O3 molecule.
22

SO2 Cl2 SO2+ Cl2


At t = 0s0.4 atm
At t = 100s

0 atm0 atm

(0.4 x) atm

x atm x atm

Pt = 0.4 x + x + x
Pt = 0.4 + x
0.7 = 0.4 + x
x = 0.3
k=
k=
k=
k=
23

2.303
t
2.303
t
2.303
100
.- -

log

)*

)* +),
0.4

log 0.8-0.7
0.4

log 0.1

x 0.6021 = 1.39 x 10-2s-1

(a) carbohydrates, lipids, proteins, enzymes, nucleic acids (any two)

(b) Antiseptics are the chemical substances which are used to kill or prevent the growth of

24

microbes. Eg Dettol / Iodoform / Boric acid/ phenol (or any other correct example)

(c) Becasuse it is unstable at cooking temperature.

(a) Vitamin A

25

(b) Uracil

(c) It suggests that six carbon atoms are in straight chain / CHO (CHOH)4 CH2OH

(i) Concern towards environment / caring / socially aware / team work. (atleast two values)

(ii) Polymers which can be degraded by the action of microorganisms. Eg. PHBV , Nylon -2-

nylon- 6/ any natural polymer


(iii) Addition polymer.

26

(a) HBr H+ + Br-

1
Or

( where R = -CH3)
(b)

27

(a)

(b)

CH3 Br KCN CH3 CN


A
CH3 COOH NH3

LiAlH4

CH3 CONH2
KOH
B

CH3 CH2 NH2


B
Br2

HNO2 CH3 CH2 OH


273K
C

CH3 NH2 CHCl3 CH3 NC


NaOH
C

OR

++

++

27

(i)
NO2

NH2
Sn/HCl

(ii)

CH3 COOH

CH3 CONH2

Br2
CH3 NH2
+KOH

NH3

O
(iii)

NH C- CH3

NH2

(CH3CO)2O

28

( Or by any other suitable method.)

(a) (i) Limiting molar conductivity when concentration approches zero the conductivity is

known as limiting molar conductivity


(ii) Fuel cell are the cells which convert the energy of combustion of fuels to electrical energy.

(b)

1
OR
28

(a) The amount of substance deposited at any electrode during electrolysis is directly

proportional to the quantity of electricity passed through the electrolyte. (aq. Solution or melt)
Charge = Q = 2F
(b) E cell = E0 cell

1
0.059
n

log

[Mg 2+ ]

[Cu2+ ]

E cell = 2.71 E cell = 2.71

0.059
2
. 67

log

0.10
.

log 10

= 2.71 0.0295 = 2.68 V


29

(a) (i)

(ii)
(b) (i) Because of 3d5(half filled) stable configuration of Mn2+

(ii) Because in zinc there is no unpaired electron / there is no contribution from the inner d
electrons.
(iii) Because of comparable energies of 7s, 6d and 5f orbitals

OR
29

(i) Mn , because of presence of 5 unpaired electrons in 3d subshell

(ii) Cu , because enthalpy of atomization and ionisation enthalpy is not compensated by enthalpy

of hydration.
(iii) Mn3+, because Mn2+ is more stable due to its half filled (3d5)configuration

(iv) Eu+2(Eu)

1
1

(v)

30

(a)
(i)
1

(ii)
1

(iii) Cl - CH2 - COOH

(b) (i) Add NaHCO3, benzoic acid will give brisk effervescence whereas benzaldehyde will not
6

give this test. (or any other test)

(ii) Add tollens reagent , propanal will give silver mirror whereas propanone will not give this
test. (or any other test)

1
OR

30

(a) (i) Because the positve charge on carbonyl carbon of CH3 CHO decreases to a lesser extent

due to one electron releasing(+I effect) CH3 group as compared to CH3 COCH3(two electron
releasing CH3 group) and hence more reactive.
(ii) Because carboxylate ion (conjugate base) is more resonance stablized than phenoxide ion.

(b) (i)
1

(ii)
1
(or any other example)
(iii)

1
(or any other example)

CBSE Annual Examination Question Paper 2013


CHEMISTRY (THEORY)
Time allowed: 3 hours]

[Maximum marks: 70

General Instructions:
(i)
All questions are compulsory.
(ii)
Question Nos. 1 to 8 are very short-answer type questions and carry 1 marks
each.
(iii) Question Nos. 9 to 18 are short-answer type questions and carry 2 marks each.
(iv) Question Nos. 19 to 27 are also short-answer type questions and carry 3 marks
each.
(v)
Question No. 28 to 30 are long-answer questions and carry 5 marks.
(vi) Use Log Table, if necessary. Use of calculators is not allowed.
___________________________________________________________________________________________________________
1.

How many atoms constitute one unit cell of a face-centered cubic crystal?

2.

Name the method used for the refining of Nickel metal.

3.

What is the covalency of nitrogen in N2O5?

4.

Write the IUPAC name of

5.

What happens when CH3-Br is treated with KCN?

6.

Write the structure of 3-methyl butanal.

7.

Arrange the following in increasing order of their basic strength in aqueous solution:
CH3.NH2,(CH3)3N,(CH3)2NH

8.

What are three types of RNA molecules which perform different functions?

9.

18g of glucose, C6H12O6 (Molar Mass= 180g mol-1) is dissolved in 1Kg of water in a
sauce pan. At what temperature will this solution boil?

10.

The conductivity of 0.20 M solution of KCl at 298 K is 0.025 S cm-1. Calculate its molar
conductivity.

11.

Write the dispersed phase and dispersion medium of the following colloidal system:
(i)

Smoke

(ii)

Milk

OR

What are lyophilic and lyophobic colloids? Which of these sols can be easily
coagulated on the addition of small amounts of electrolytes?
Material downloaded from http://myCBSEguide.com and http://onlineteachers.co.in
Portal for CBSE Notes, Test Papers, Sample Papers, Tips and Tricks

12.

Write the differences between physisorption and chemisorption with respect to the
following:

13.

(i)

Specificity

(iii)

Reversibility and

(ii)

Temperature dependence

(iv)

Enthalpy change

(a)

Which solution is used for the leaching of silver metal in the presence of air in
the metallurgy of silver?

(b)

Out of C and CO, which is a better reducing agent at the lower temperature
range in the blast furnace to extract iron from the oxide ore?

14.

What happens when


(i)

PCl5 is heated?

(ii)

H3PO3 is heated?

Write the reaction involved.


15.

(a)

Which metal in the first transition series (3d series) exhibits +1 oxidation state
most frequently and why?

(b)

Which of the following cations are colored in aqueous solutions and why?
Sc3+,V3+,Ti4+,Mn2+
(At. nos. Sc = 21, V= 23, Ti = 22, Mn = 25)

16.

Chlorobenzene is extremely less reactive towards a nucleophilic substitution reaction.


Give two reasons for the same.

17.

Explain the mechanism of the following reaction:

18.

How will you convert:

19.

(i)

Propene to Propan-2-ol?

(ii)

Phenol to 2, 4, 6 trinitrophenol?

(a)

What type of semiconductor is obtained when silicon is doped with boron?

(b)

What type of magnetism is shown in the following alignment of magnetic


moments?

(c)

What type of point defect is produced when AgCl is doped with CdCL2?

Material downloaded from http://myCBSEguide.com and http://onlineteachers.co.in


Portal for CBSE Notes, Test Papers, Sample Papers, Tips and Tricks

20.

Determine the osmotic pressure of a solution prepared by dissolving 2.5 102 g of


K2SO4 in 2L of water at 25 C , assuming that it is completely dissociated.
(R=0.0821 L atm K-1mol-1, Molar mass of K2SO4=174g mol-1).

21.

Calculate the emf of the following cell at 298 K:


Fe( s ) Fe 2+ (0.001M ) H + ( g ) H 2 ( g )(1bar ), Pt ( s )

(Givencell
= +0.44V )

22.

How would you account for the following?


(i)

Transition metals exhibit variable oxidation states.

(ii)

Zr (Z=40) and Hf (Z=72) have almost identical radii.

(iii)

Transition metals and their compounds act as catalyst.


OR

Complete the following chemical equations:

23.

24.

(i)

Cr2O72 + 6 Fe 2 + + 14 H +

(ii)

2CrO42 + 2 H +

(iii)

2 MnO4 + 5C2O42 + 16 H +

Write the IUPAC names of the following coordination compounds:


(i)

[Cr(NH3)3Cl3]

(ii)

K3[Fe(CN)6]

(iii)

[CoBr2(en)2]+, (en = ethylenediamine)

Give the structures of A, B and C in the following reactions:


(i)

(ii)
25.

Write the names and structures of the monomers of the following polymers:
(i)

Buna-S

(ii)

Neoprene

(iii)

Nylone-6, 6

Material downloaded from http://myCBSEguide.com and http://onlineteachers.co.in


Portal for CBSE Notes, Test Papers, Sample Papers, Tips and Tricks

26.

After watching a programme on TV about the adverse effects of junk food and soft
drinks on the health of school children, Sonali, a student of Class XII, discussed the
issue with the school principal. Principal immediately instructed the canteen
contractor to replace the fast food with the fibre and vitamins rich food like sprouts,
salad, fruits etc. This decision was welcomed by the parents and the students.
After reading the above passage, answer the following questions:

27.

(a)

What values are expressed by Sonali and the Principal of the school?

(b)

Give two examples of water-soluble vitamins.

(a)

Which one of the following is a food preservative?


Equanil, Morphine, Sodium benzoate

28.

(b)

Why is bithional added to soap?

(c)

Which class of drugs is used in sleeping pills?

(a)

A reaction is second order in A and first order in B.


(i)

Write the differential rate equation.

(ii)

How is the rate affected on increasing the concentration of A three


times?

(iii)

How is the rate affected when the concentration of both A and B are
doubled?

(b)

A first order reaction takes 40 minutes for 30% decomposition. Calculate t1/2
for this reaction.
(Given log 1.428=0.1548)
OR

(a)

For a first order reaction, show that time required for 99% completion is twice
the time required for the completion of 90% of reaction.

(b)

Rate constant k of a reaction varies with temperature T according to the


equation: log k = log A

Ea 1

2.303R T

Where Ea is the activation energy. When a graph is plotted for log k Vs.

1
,a
T

straight line with a slope of -4250 K is obtained. Calculated Ea for the reaction.
(R=8.314 JK-1mol-1)
Material downloaded from http://myCBSEguide.com and http://onlineteachers.co.in
Portal for CBSE Notes, Test Papers, Sample Papers, Tips and Tricks

29.

(a)

(b)

Give reasons for the following:


(i)

Bond enthalpy of F2 is lower than that of Cl2.

(ii)

PH3 has lower boiling point than NH3.

Draw the structures of the following molecules:


(i)

BrF3

(ii)

(HPO3)3

OR
(a)

(b)

Account for the following:


(i)

Helium is used in diving apparatus.

(ii)

Fluorine does not exhibit positive oxidation state.

(iii)

Oxygen shows catenation behavior less than sulphur.

Draw the structures of the following molecules.


(i)

30.

(a)

XeF2

(ii)

H2S2O8

Although phenoxide ion has more number of resonating structures than


Carboxylate ion, Carboxylic acid is a stronger acid than phenol. Give two
reasons.

(b)

How will you bring about the following conversions?


(i)

Propanone to propane

(ii)

Benzoyl chloride to benzaldehyde

(iii)

Ethanal to but-2-enal
OR

(a)

Complete the following reactions:

(i)
(ii)

(iii)
(b)

Give simple chemical tests to distinguish between the following pairs of


compounds:
(i) Ethanal and Propanal

(ii)

Benzoic acid and Phenol

Material downloaded from http://myCBSEguide.com and http://onlineteachers.co.in


Portal for CBSE Notes, Test Papers, Sample Papers, Tips and Tricks

Marking Scheme
Chemistry
Delhi- SET (56/1/1)
1
2

6:6 or 6

1
1

The sum of powers of the concentration terms of the reactants in the rate law expression is
called the order of that chemical reaction.
Or
rate = k[A]p[B]q
Order of reaction = p+q
Due to unbalanced bombardment of the colloidal particles by the molecules of the dispersion
medium.
NO2+

5
6.

2,5-Dimethylhexane -1,3-diol.
(CH3)2CHCOOH < CH3CH(Br)CH2COOH < CH3CH2CH(Br)COOH

1
1

1
1

7.
C6H5N2+Cl

+ KI

C6H5I + KCl + N2

8.

Phenol (or any other correct one)

9.

2
Aryl halides are less ractive towards nucleophilic substitution because of any of the
following reasons with correct explanation:
(i) Resonance effect stabilization
(ii) sp2 hybridization in haloarenes being more electronegative than sp3 in haloalkanes.
(iii) Instability of phenyl cation which is not stabilized by resonance.
(iv) possible repulsion between electron rich nucleophile and electron rich arene
(atleast two reasons to be given )
OR

10
11

(i) CH3I, Because iodine is a better leaving group due to its larger size.
(ii) CH3Cl,the presence of bulky group on the carbon atom in (CH3)2CCl has an inhibiting
effect.
(a) 1-Bromobut-2-ene
(b) CH3CH2CH2CH2Br
Henrys law states that at a constant temperature, the solubility of a gas in a liquid is directly
proportional to the pressure of the gas over the solution.
Applications
(i)
To increase the solubility of CO2 in soft drinks and soda water, the bottle is sealed
under high pressure.
(ii)
Scuba divers must cope with high concentrations of dissolved Nitrogen with
breathing air at high pressure underwater.To avoid this air is diluted with He.
(iii) At high altitudes the partial pressure of oxygen is less than that at the ground level.
Low blood oxygen causes anoxia.
(any two)

1
1
1
1
1

12

k = 2.303 log [ A0 ]
t
[A]
k = 2.303 log 100
40min
70
k = 2.303 x 0.155 = 0.00892min-1
40
t =
0.693
1/2
k
t =
0.693 min
1/2
0.00892
t
1/2 = 77.7min

13

14

Rate constant k of a reaction is defined as the rate of reaction when the concentration of
the reactant(s) is unity. / or Rate constant is the proportionality factor in the rate law.
(i)
Unit for k for a zero order reaction = mol L-1 s-1
(ii)
Unit for k for a first order reaction = s-1

(i)
(ii)

15

Peptide linkage: Peptide linkage is an amide (-CO-NH-) bond formed between


1
COOH and NH2 group in protein formation.
Denaturation: When a protein in its native form, is subjected to physical change like 1
change in temperature or chemical change like change in pH,protein loses its
biological activity. This is called denaturation of protein.

(i) Despite having the aldehyde group, glucose does not give 2,4-DNP test or
Schiffs test.
(ii) It does not form the hydrogensulphite addition product with NaHSO3.
(iii) The pentaacetate of glucose does not react with hydroxylamine indicating
the absence of free CHO group.
(any two)

1+1

16
(i)The lone pair of electrons on N atom in NH3 is directed and not diffused / delocalized as it
is in PH3 due to larger size of P/ or due to availability of d-orbitals in P.
(ii) S2 molecule like O2, has two unpaired electrons in antibonding * orbitals.
1+1

17

SF4
1

XeF4

18.
Biodegradable detergents are those detergents which are easily degraded by the microorganisms and hence are pollution free.
ex. Soap / Sodium laurylsulphate / any other unbranched chain detergent.
(any one)

19

Non Biodegradable Detergents are those detergents which cannot be degraded by the bacteria
easily and hence create pollution. [example not essential]

The solids with intermediate conductivities between insulators and conductors are termed
semiconductors.

(i) n- type semiconductor : It is obtained by doping Si or Ge with a group 15 element


like P. Out of 5 valence electrons , only 4 are involved in bond formation and the fifth
electron is delocalized and can be easily provided to the conduction band. The conduction
is thus mainly caused by the movement of electron.

(ii)p type semi conductor : It is obtained by doping Si or Ge with a group 13


element like Gallium which contains only 3 valence electrons. Due to missing of 4th valence 1
electron,electron hole or electron vacancy is created The movement of these positively
charged hole is responsible for the conduction.

20
Tf = Kf m

No. of moles of glucose =

54 g
180 g mol -1

Molality of Glucose solution =

54 mol x
180

1000
= 1.20mol kg-1
250kg

T f = Kf m
= 1.86 K kg mol-1 x 1.20 mol kg 1
1

= 2.23 K
Temperature at which solution freezes =( 273.15 2.23K = 270.77K or -2.230C
Or (273.000 2.23)K = 270.7 K
21

Lyophilic sols are solvent attracting sols


ex. Gum,gelatine,starch,rubber (any one)

Lyophobic sols are solvent repelling sols


ex. Metal sols,metal sulphides (any one)

Lyophobic sols are readily coagulated because they are not stable.
+

22

(i)

(ii)
(iii)

Froth floatation process: This method is based on the difference in the


wettability of the mineral particles (sulphide ores)and the gangue
particles.The mineral particles become wet by oils while the gangue
particles by water and hence gets separated.
Zone refining: This method is based on the principle that the impurities
are more soluble in the melt than in the solid state of metal.
Refining by Liquation:The method is based on the lower melting point
of the metal than the impurities and tendency of the molten metal to
flow on the sloping surface.

23

(i)3Cl2+6NaOH 5NaCl+NaClO3+3H2O
(ii)4H3PO3  3H3PO4+PH3
(iii)Xe+[PtF6]OR
(i)Ca3P2(s)+ 6H2O(l)3Ca(OH)2(aq) + 2PH3(g)
(ii)Cu2+(aq)+ 4NH3(aq)[Cu(NH3)4]2+(aq)
1x3=3

(iii)2F2(g)+ 2H2O(l)  4H+ (aq) + 4F-(aq) + O2(g)


24

(a) Ligand: The ions or molecules bound to the central atom/ion in the
coordination entity are called ligands.
,

ex. of bidentate ligand- ethane-1,2-diamine or oxalate ion


(or any other)
(b)* In [Ni(CN)4]2-, nickel is Ni2+, (3d8),with strong Ligand like CN-, all the
electrons are paired up in four d-orbitals resulting into dsp2 hybridization
giving square planar structure and diamagnetic character.
In Ni(CO)4:, nickel is in zero valence state , (3d84s2),with strong Ligand like
CO,4s2,electrons are pushed to the d-orbitals resulting into sp3 hybridization
giving tetrahedral shape and diamagnetic in nature.

(or this can be explained by drawing orbital configurations too.)


25

(i) PCC, KMnO4,CrO3 (any one)


(ii) LiAlH4,NaBH4

(any one)

(iii)aqueous Br2

1X 3=3

(or any other suitable reagent)

26

(i)

It is because in aniline the NH2 group is attached directly to the


benzene ring.It results in the unshared electron pair on nitrogen atom to
be in conjugation with the benzene ring and thus making it less
available for protonation.
(or any other suitable reason)

(ii) Methyl amine in water gives OH- ions which react with FeCl3 to give precipitate of
ferric hydroxide/ or
+

CH3NH2 + H2O
Fe3+ + 3OH-

CH3NH3OH-

CH3NH+3 +OH-

Fe (OH)3

(iii)Aniline does not undergo Friedel-Crafts reaction due to salt formation


with aluminium chloride, the Lewis acid.

1X 3=3

(i)

27

Buna-S : 1,3- Butadiene and Styrene

CH2 = CH CH = CH2
(ii)

and

Neoprene:Chloroprene

Cl

CH2 = C CH = CH2
(iii) Nylon-6: Caprolactum
+

28

m =
c

= 7.896 x 10-5 S cm-1 x 1000 cm3L-1


0.00241 molL-1
= 32.76 Scm2 mol-1

m
m0

=32.76 Scm2 mol-1


390.5 Scm2 mol-1

= 0.084 Scm2 mol-1


1

K =
C(1-
=

=C

0.00241 X (0.084)2

=1.7 X 10

-5

or 1.865 x 10

-5

(if is not neglected)

OR

Ag+ + e-  Ag
108 g is deposited by 96500C electric charge
1.45 g of silver is deposited by 96500C x 1.45 g = 1295.6 C
108 g
Quantity of electricity passed = Current x t
t =
1295.6C = 863.7 s
1.5 amp
2+
Cu + 2e  Cu
2 x 96500 C deposits 63.5 g of Cu
1295.6 C deposits 63.5g x 1295.6 C of Cu
2 x 96500 C
= 0.426 g of Cu

1
1

Zn2+ + 2e-  Zn
2 x 96500 C deposits 65.4 g of Zn
1295.6 C deposits 65.4g x 1295.6 C of Zn
2 x 96500 C
1

= 0.44 g of Zn

(i)

29

(or any other suitable method)


Because of larger number of unpaired electrons in their atoms they have
stronger interatomic interaction and hence stronger bonding between
atoms resulting in higher enthalpies of atomisation.

(ii)

Because of their ability to adopt multiple oxidation states and to form


complexes.

(iii)

Because of poorer shielding by 5f electrons than that by 4f , actinoid


contraction is greater than the lanthanoid contraction.

(iv)

Much larger third inonisation energy of Mn( where the required change
is d5 to d4) is mainly responsible for this.
Because of the presence of incomplete d-orbital (3d14s2) in its ground
state.

(v)

1x5=5

OR
3d34s2(Vanadium): Oxidation states +2,+3,+4,+5
Stable oxidation state: +4 as VO2+ ,+5 as VO433d54s2(Manganese): Oxidation states +2,+3,+4,+5,+6,+7
Stable oxidation states: +2 as Mn2+ ,+7 as MnO-4
3d64s2(Iron): Oxidation states +2,+3
Stable oxidation state: +2 in acidic medium, +3 in neutral or in alkaline
medium.

1x3=3

(b) (i)
4FeCr2O4 + 8Na2CO3 + 7O2

8 Na2CrO4 + 2 Fe2O3 + 8 CO2

(ii)
2MnO2 + 4KOH + O2

2K2MnO4 + 2H2O
8

1+1

30

(a)
(i)

(ii) BH3,

H2O2 / OH-,

PCC

(any two)

(iii)
1+1

(NOTE:any two correct answers to be evaluated and 1 marks for each to


be awarded)

(b) (i) Cannizzaro reaction: Aldehydes which do not have an


-hydrogen atom, uhdergo self oxidation and reduction
reaction on treament with concentrated alkali.

(or any other suitable reaction)

(ii) Cross aldol condensation: When aldol condensation is carried out between
two different aldehydes and /or ketones, it is called Cross aldol condensation

(or any other suitable reaction)

(Note: Award full marks for correct chemical equation;award mark if only
statement is written)

OR
(i)

(ii)
(iii)

Because two alkyl groups in ketones reduce the positive charge on


carbon atom of the carbonyl group more effectively than in aldehydes.
/ or sterically, the presence of two relatively large substituents in
ketones hinders the approach of nucleophile to carbonyl carbon than in
aldehydes having only one such substituents.
Beacuase of the absence of hydrogen bonding in aldehydes and 1x3=3
ketones.
Because of the presence of the sp2 hybridised orbitals(or -bond) of
carbonyl carbon.
10

(b) (i)Acetaldehyde and benzaldehyde : Acetaldehyde gives yellow ppt of


Iodoform(CHI3)on addition of NaOH / I2 whereas benzaldehyde does not give
this test.
( or any other suitable test)

(ii) Propanone and propanol : Propanone gives yellow ppt of


1+1
Iodoform(CHI3)on addition of NaOH / I2 whereas propanol does not give this
test. Or / Propanol gives brisk effervesence on adding a piece of Sodium metal
whereas Propanone does not give this test.
( or any other suitable test)

11

Marking Scheme
Chemistry
Delhi- SET (56/1/2)
1

In primary battery the reaction occurs only once and after use over a period of time
becomes dead. Leclanche cell or Dry cell is an example.

Q.3 Set 1

Q.4 Set 1

Q.5 Set 1

6.

1
Pentane -2, 4 dione

7.

Q.7 Set 1
1

8.

Q.8 Set 1

Raoults law states that for a solution of volatile liquids, the partial vapour pressure 1
of each component in the solution is directly proprtional to its mole fraction.
When the solute-solvent interaction is weaker than those between the solute-solute
and solvent-solvent molecules than solution shows positive deviation from Raoults
law because the partial pressure of each component is greater.
ex. mixture of ethanol and acetone or carbondisulphide and acetone behave in this

manner.
When the solute-solvent interaction is stronger than those between the solute-solute
and solvent-solvent molecules than solution shows negative deviation from Raoults
law and the partial vapour pressure of each component is lower.
ex. mixture of chloroform and acetone behave in this manner.
(Note: Explaination with suitable example of any one of the two.)
OR
The extra pressure applied on the solution side that just stops the flow of solvent to
solution through semi-permeable membrane is called osmotic pressure of the
solution.

Here is the osmotic pressure and R is the gas constant.

Thus knowing the quantities w2, T, and V we can calculate the molar mass of the
solute.
1

10

m = x 1000 cm3L-1
c
= 0.0248 S cm-1 x 1000 cm3 L-1
0.20 mol L-1

1
= 24.8 S cm2
0.20 mol

m = 124 Scm2 mol-1


11

The galvanic cell is depicted as:


Zn(s)| Zn2+(aq) || Ag+(aq)|Ag (s)
(i)
Zinc electrode is negatively charged
(ii)
The ions formed i.e Zn2+ and Ag+ in the solution are the carriers of the
current within the cell.
(iii)
At anode: Zn(s)Zn2+(aq) + 2e-

At cathode: 2Ag+(aq)+ 2e-  2Ag(s)

12

Q.14 Set 1

13

Q.15 Set 1

14

(i)
(ii)

1+1

15

C6H5I + KCl + N2
BrCH2-CH2Br

(i) Aryl halides are less ractive towards nucleophilic substitution because of any
of the following reasons
(i) Resonance effect stabilization
(ii) sp2 hybridization in haloarenes and sp3 in haloalkanes.
(iii) Instability of phenyl cation
(iv) possible repulsion

(ii)

Cl

Undergoes SN1 reaction faster because of the stability of secondary


carbocation.

16
17

Q.18 Set 1
Q.16 Set 1

2
2

18

Q.17 Set 1

19
20
21

Q.21 Set 1
Q.20 Set 1
Q.19 Set 1

3
3
3
2

22
23
24
25
26
27
28

Q.23 Set 1
Q.22 Set 1
Q.26 Set 1
Q.27 Set 1
Q.25 Set 1
Q.24 Set 1
(a) Half life of a First order reaction:

3
3
3
3
3
3

*
1/2

1/2
0

(b) 2NH3 (g)


Rate =

N2 (g) +3 H2(g)

-d[NH3] =k[NH3]O = 2.5 x10-4Ms-1


dt

- 1 d[NH3] = + d[N2] = + 1 d[H2]


2
dt
dt
3
dt
Rate of production of N2 = + d[N2] = - 1 [NH3]
dt
2 dt
= 1 x (2.5 x 10-4 Ms-1) = 1.25 x 10-4 Ms-1
2
Rate of production of hydrogen = d[H2] = - 3 [NH3]
dt
2 dt
= 3 x (2.5 x 0-4 Ms-1)
2
= 3.75 x 10-4 Ms-1

or
Rate =

Rate =

-d[NH3] =k[NH3]O = 2.5 x10-4Ms-1


dt

- 1 d[NH3] = + d[N2] = + 1 d[H2]


2
dt
dt
3
dt

Rate of production of N2 = + d[N2] = Rate= 2.5 x10-4Msdt

Rate of production of hydrogen = d[H2] = 3 x Rate


dt
= 3 x (2.5 x 0-4 Ms-1)
= 7.5 x 10-4 Ms-1

(Note: No marks to be deducted for wrong unit in this question,as


there is a misprint in the question in units of k)
or
(a)Factors affecting rate of chemical reaction are:
(i)Concentration of reactants
(ii)Temperature
(iii)Presence of catalyst
(iv)Surface Area
(v)Activation energy
(any four)
(b) k = 0.693
t1/2
=
k
0.693
5730 y
K = 1.21 x 10-4 y-1
t = 2.303
k

log [ A0 ]
[A]

x4=2

log 100
k = 2.303
1.21 x 10-4 y-1
80
k=

log 1.25
2.303
-4 -1
1.21 x 10 y

k=

2.303
x 0.0969
-4 -1
1.21 x 10 y

= 1845 years

29

1x3=3

(or any other suitable method)


(a) (i)Decarboxylation: Carboxylic acids lose carbon dioxide to
form hydrocarbons when their sodium salts are heated with
sodalime.The reaction is known as decarboxylation.
R-COONa

NaOH & CaO

R-H + Na2CO3

Heat
(ii) Cannizzaro reaction: Aldehydes which do not have an

-hydrogen atom, uhdergo self oxidation and reduction


reaction on treament with concentrated alkali.

( or any other suitable reaction)


(Note: Award full marks for correct chemical equation;award
mark if only statement is written)

1+1

OR
(a)
(i)
C
H
O

69.77/12
11.63/1
18.60/16

5.81
11.63
1.16

5.81/1.16
11.63/1.16
1.16/1.16

5
10
1

Empirical formula C5H10O,empirical formula mass 60+10+16=86


1
Hence, Mol formula C5H10O
It is a ketone as it appears from its reactions which on oxidation
1
gives ethanoic and propanoic acids, hence the compound is
1

CH3COCH2CH2CH3
(b)(i)Because the stability of conjuguate base of
monochloroethanoic acid is less due to presence of one electron
withdrawing -Cl group than in dichloroethanoic acid.
(ii)This is because of greater electronegativity of sp2 hybridised
carbon to which carboxyl carbon is attached.
( or any other suitable reason)
30

Q.29 Set 1

1+1

Marking Scheme
Chemistry
Delhi- SET (56/1/3)
1

In which the alignment of domains (moments) is in a compensatory way to give


zero net moment.
m =
where m is molar conductivity , is conductivity
c
c is concentration in mol L-1
Chemisorption

Q.5 Set 1

5
6

Q.4 Set 1
Q.8 Set 1

1
1

Q.6 Set 1

Q.7 Set 1

Q.11 Set 2

1
2

1
1

10
R= ( l / A )
Cell constant, l/A = R/ = R = Resistance x Conductivity
=(1500 ) x (0.146 x 10-3 S cm-1)
= 0.219 cm-1
11

1
1

Q.9 Set 2

12

Q.16 Set 1

2
2

13

Q.17 Set 1

14
15
16
17

Q.9 Set 1
Q.10 Set 1
Q.18 Set 1
Q.14 Set 1

2
2
2
2

18

Q.15 Set 1

19
1x3=3

6x=3

20
21

Q.20 Set 1

(i)

(ii)

(iii)

22

(i)

(ii)
(iii)

Zone refining: This method is based on the principle that the


impurities are more soluble in the melt than in the solid state
of metal.
Vapour phase refining: In this method, the metal is
converted into its volatile compound and collected elsewhere.
It is then decomposed to give pure metal.
Electrolytic refining: In this method, the impure metal is
made to act as anode. A strip of the same metal in pure form
is used as cathode. They are put in a suitable electrolytic bath
containing soluble salt of the same metal.The more basic
metal remains in the solution and the less basic ones go to the 1x3=3
anode mud
Ferric hydroxide sol is positively charged. By adding
potassium chloride, the excess chloride ions neutralize its
positive charge and cause it to coagulate.
The dispersed phase and dispersion medium migrate towards
oppositely charged electrodes (electrophoresis).
The beam of light is scattered by colloidal particles(Tyndall
effect).

1x3=3

23

Q.27 Set 1

24

Q.26 Set 1

25

Q.24 Set 1

26

Q.25 Set 1

27

Q.23 Set 1

28

Q.29 Set 1

29

Q.28 Set 2

30

Q.30 Set 1

You might also like